INSIGHTSIAS IA SIMPLIFYING IAS EXAM PREPARATION

RTM COMPILATIONS PRELIMS 2021

June 2021

www.insightsactivelearn.com | www.insightsonindia.com

Revision Through MCQs (RTM) Compilation (June 2021)

Telegram: https://t.me/insightsIAStips 2 Youtube: https://www.youtube.com/channel/UCpoccbCX9GEIwaiIe4HLjwA

Revision Through MCQs (RTM) Compilation (June 2021)

Table of Contents RTM- REVISION THROUGH MCQS –1st -Jun-2021 ...... 4 RTM- REVISION THROUGH MCQS –2nd -Jun-2021 ...... 8 RTM- REVISION THROUGH MCQS –3rd -Jun-2021 ...... 14 RTM- REVISION THROUGH MCQS –4th -Jun-2021 ...... 19 RTM- REVISION THROUGH MCQS –5th -Jun-2021 ...... 24 RTM- REVISION THROUGH MCQS – 7th -Jun-2021 ...... 30 RTM- REVISION THROUGH MCQS – 8th -Jun-2021 ...... 34 RTM- REVISION THROUGH MCQS – 9th -Jun-2021 ...... 39 RTM- REVISION THROUGH MCQS – 10th -Jun-2021 ...... 44 RTM- REVISION THROUGH MCQS – 11th -Jun-2021 ...... 49 RTM- REVISION THROUGH MCQS – 12th -Jun-2021 ...... 55 RTM- REVISION THROUGH MCQS – 14th -Jun-2021 ...... 59 RTM- REVISION THROUGH MCQS – 15th -Jun-2021 ...... 64 RTM- REVISION THROUGH MCQS – 16th -Jun-2021 ...... 69 RTM- REVISION THROUGH MCQS – 17th -Jun-2021 ...... 74 RTM- REVISION THROUGH MCQS – 18th -Jun-2021 ...... 79 RTM- REVISION THROUGH MCQS – 19th -Jun-2021 ...... 83 RTM- REVISION THROUGH MCQS – 21st -Jun-2021 ...... 88 RTM- REVISION THROUGH MCQS – 22nd -Jun-2021 ...... 93 RTM- REVISION THROUGH MCQS – 23rd-Jun-2021 ...... 98 RTM- REVISION THROUGH MCQS – 24th -Jun-2021 ...... 104 RTM- REVISION THROUGH MCQS – 24th -Jun-2021 ...... 109 RTM- REVISION THROUGH MCQS – 26th -Jun-2021 ...... 113 RTM- REVISION THROUGH MCQS – 28th -Jun-2021 ...... 118 RTM- REVISION THROUGH MCQS – 29th -Jun-2021 ...... 123 RTM- REVISION THROUGH MCQS – 30th -Jun-2021 ...... 127

Telegram: https://t.me/insightsIAStips 3 Youtube: https://www.youtube.com/channel/UCpoccbCX9GEIwaiIe4HLjwA

Revision Through MCQs (RTM) Compilation (June 2021)

RTM- REVISION THROUGH MCQS –1st -Jun-2021 1. Consider the following statements regarding Sedition Law in India. 1. Section 124A of the Indian Penal Code (IPC), which deals with sedition, was drafted by Thomas Babington Macaulay. 2. Mahatma Gandhi was never held under the sedition trial during freedom movement. Which of the above statements is/are correct? (a) 1 only (b) 2 only (c) Both 1 and 2 (d) Neither 1 nor 2 Ans: (a) Explanation: Famous sedition trials during freedom movement: • The first is the trial of Jogendra Chandra Bose in 1891. Bose, the editor of the newspaper, Bangobasi, wrote an article criticizing the Age of Consent Bill for posing a threat to the religion and for its coercive relationship with Indians. • The other most well-known cases are the sedition trials of Bal Gangadhar Tilak and the trial of Mahatma Gandhi in 1922. Mahatma Gandhi for his articles published in Young India.

Refer: https://www.insightsonindia.com/2021/06/01/its-time-to-define-limits-of-sedition- says-sc/

2. Which of the following can be considered as neglected tropical diseases? 1. Chagas disease 2. Hookworm infection 3. Leprosy 4. Lymphatic filariasis Select the correct answer using the code below: (a) 1 and 3 only (b) 2, 3 and 4 only (c) 2 and 4 only (d) 1, 2, 3 and 4 Ans: (d) Explanation: • 13 neglected tropical diseases: ascariasis, Buruli ulcer, Chagas disease, dracunculiasis, hookworm infection, human African trypanosomiasis, Leishmaniasis, leprosy, lymphatic filariasis, onchocerciasis, schistosomiasis, trachoma, and trichuriasis.

Refer: https://www.insightsonindia.com/2021/06/01/january-30-now-world-neglected- tropical-diseases-day/

3. Bhasan Char, an Island recently in news is located in (a) (b) Bay of Bengal (c) South China Sea (d) Sea of Japan Ans: (b) Explanation:

Telegram: https://t.me/insightsIAStips 4 Youtube: https://www.youtube.com/channel/UCpoccbCX9GEIwaiIe4HLjwA

Revision Through MCQs (RTM) Compilation (June 2021)

• Bhasan Char (Floating Island) also known as Char Piya or Thengar Char Island, is an island in Hatiya, Bangladesh. • The island was formed from a build-up of silt in the Bay of Bengal only 20 years ago, and concerns have been consistently raised about Bhasan Char’s exposure to extreme weather and distance from the mainland in emergencies.

Refer: https://www.insightsonindia.com/2021/06/01/thousands-of-rohingya-protest-at- bhashan-char/

4. With reference to 5G mobile network, which of the following statements is/are correct? 1. 5G technology is meant to deliver higher multi-Gbps peak data speeds, ultra-high latency, more reliability and massive network capacity. 2. 5G wireless devices will connect to the Internet and telephone network by radio waves through a local antenna. 3. At present 5G technology is available only in US, China and some European countries. Select the correct answer using the code below: (a) 1 and 2 only (b) 1 and 3 only (c) 2 only (d) 2 and 3 only Ans: (c) Explanation: • S1: 5G wireless technology is meant to deliver higher multi-Gbps peak data speeds, ultra-low latency, more reliability, massive network capacity, increased availability, and a more uniform user experience to more users. Higher performance and improved efficiency empower new user experiences and connects new industries. • S2: Like its predecessors, 5G networks are cellular networks, in which the service area is divided into small geographical areas called cells. All 5G wireless devices in a cell are connected to the Internet and telephone network by radio waves through a local antenna in the cell. • S3: 5G has been deployed in 35+ countries and counting.

o Refer: https://www.insightsonindia.com/2021/06/01/5g-technology/

5. With reference to Global Biodiversity, recently discovered ‘Litoria mira’ is a/an/the (a) Bird (b) Primate (c) Reptile (d) Amphibian

Telegram: https://t.me/insightsIAStips 5 Youtube: https://www.youtube.com/channel/UCpoccbCX9GEIwaiIe4HLjwA

Revision Through MCQs (RTM) Compilation (June 2021)

Ans: (d) Explanation: Litoria mira: • They are new frog species discovered recently in the rainforests of New Guinea. They are chocolate coloured. • The name is inspired by the Latin adjective mirum, which means surprised or strange, stemming from the scientist’s surprise in discovering an undescribed member of the predominately Australian Litoria genus of tree frogs. • Litoria mira can be distinguished from all other Litoria by its unique combination of moderately large size, webbing on hand, relatively short and robust limbs, and small violet patch of skin on the edge of its eyes

Refer: Facts for Prelims: https://www.insightsonindia.com/2021/06/01/insights-daily- current-affairs-pib-summary-1-june-2021/

6. Consider the following statements about Central Board of Direct Taxes (CBDT): 1. It is a statutory body established as per the Central Board of Revenue Act, 1963. 2. It is India’s official financial action task force unit. 3. It is administered by the Department of Financial Services under the Ministry of Finance. Which of the given above statements is/are correct? (a) 1 and 2 only (b) 1 and 3 only (c) 2 and 3 only (d) 1, 2 and 3 Ans: (a) Explanation: About CBDT: • It is a statutory body established as per the Central Board of Revenue Act, 1963. • It is India’s official financial action task force unit. • It is administered by the Department of Revenue under the Ministry of Finance.

Refer: Facts for Prelims: https://www.insightsonindia.com/2021/06/01/insights-daily- current-affairs-pib-summary-1-june-2021/

7. With reference to Asiatic Wild Dogs (Dhole), consider the following statements: 1. It is a highly social animal. 2. It is listed as Endangered on the IUCN Red List. 3. It is protected under Schedule 2 of the Wildlife Protection Act, 1972. Which of the given above statements is/are correct? (a) 1 and 2 only (b) 3 only (c) 2 and 3 only (d) 1, 2 and 3 Ans: (d) Explanation: • S3: In India, the dhole is protected under Schedule 2 of the Wildlife Protection Act, 1972. The creation of reserves under Project Tiger provided some protection for dhole populations sympatric with tigers. In 2014, the Indian

Telegram: https://t.me/insightsIAStips 6 Youtube: https://www.youtube.com/channel/UCpoccbCX9GEIwaiIe4HLjwA

Revision Through MCQs (RTM) Compilation (June 2021)

government sanctioned its first dhole conservation breeding centre at the Indira Gandhi Zoological Park (IGZP) in Visakhapatnam. • S2: Endangered–IUCN. • S1: The dhole is a highly social animal, living in large clans without rigid dominance hierarchies and containing multiple breeding females.

Refer: https://www.insightsonindia.com/wp-content/uploads/2021/04/INSTA-PT-2021- Exclusive-Environment.pdf

8. With reference to Indian Bullfrog, consider the following statements: 1. It is the largest frog found in the Indian Subcontinent. 2. It is listed as Vulnerable on the IUCN Red List. 3. It is protected under Schedule 2 of the Wildlife Protection Act, 1972. Which of the given above statements is/are correct? (a) 1 only (b) 2 only (c) 1 and 2 only (d) 1, 2 and 3 Ans: (a) Explanation: • Scientific name: Hoplobatrachus tigerinus. • IUCN status: Least Concern category. • Habitats: South and South-East Asia. • It is the largest frog found in the Indian Subcontinent. • They often engage in cannibalism by feeding on smaller individuals of their own kind and on other frogs. • Its loud croaking call, attracts the opposite sex, but also predators. • It is protected under Schedule IV of the Wildlife Protection Act of India, 1971.

Refer: https://www.insightsonindia.com/wp-content/uploads/2021/04/INSTA-PT-2021- Exclusive-Environment.pdf

9. In which one of the following State is Valmiki Tiger Reserve (VTR) locted? (a) Odisha (b) Madhya Pradesh (c) Bihar (d) Chhattisgarh Ans: (c) Explanation: • Valmiki Tiger Reserve forms the eastern most limit of the Himalayan Terai forests in India, and is the only tiger reserve of Bihar.

Refer: https://www.insightsonindia.com/wp-content/uploads/2021/04/INSTA-PT-2021- Exclusive-Environment.pdf

10. The ‘Golden Langur Conservation Breeding Programme’ is implemented by which one of the following state? (a) Sikkim (b) West Bengal

Telegram: https://t.me/insightsIAStips 7 Youtube: https://www.youtube.com/channel/UCpoccbCX9GEIwaiIe4HLjwA

Revision Through MCQs (RTM) Compilation (June 2021)

(c) Assam (d) Meghalaya Ans: (c) Explanation: • Assam’s Golden Langur Conservation Breeding Programme has achieved its maiden success with the birth of a female offspring at the Assam State Zoo, widely known as Guwahati Zoo. With funds being provided by (CZA), the project for conservation was started in 2011-12. • Habitat: o Semi evergreen and mixed deciduous forests. o Found in Small regions of western Assam and in the neighbouring foothills of the black mountains of Bhutan. • Protection status: o Schedule Ispeciesin the Wildlife Protection Act (1972). o CITES Appendix I. o Endangeredin IUCN Red List.

Refer: https://www.insightsonindia.com/wp-content/uploads/2021/04/INSTA-PT-2021- Exclusive-Environment.pdf

RTM- REVISION THROUGH MCQS –2nd -Jun-2021

11. Consider the following statements regarding Avian influenza or Bird flu. 1. It is a highly contagious viral disease caused by Influenza Type A viruses. 2. Avian influenza does not affect the egg production in poultry birds. 3. Bird flu can be spread through their droppings while flying. Which of the above statements is/are correct? (a) 1 only (b) 2 and 3 only (c) 1 and 3 only (d) 1, 2 and 3 Ans: (c) Explanation: • What is avian influenza or bird flu? o It is a highly contagious viral disease caused by Influenza Type A viruses which generally affects poultry birds such as chickens and turkeys. There are many strains of the virus – some of them are mild and may merely cause a low egg production or other mild symptoms among chickens, while others are severe and lethal. • How does the bird flu spread? o Wild aquatic birds such as ducks and geese are the natural reservoir of Influenza A viruses and the central players in the ecology of these viruses. • Many birds carry the flu without developing sickness, and shed it in their droppings. Since birds excrete even while flying, they provide “a nice aerosol of influenza virus, shedding it all over the world”

Refer: https://www.insightsonindia.com/2021/06/02/h10n3-bird-flu-strain/

Telegram: https://t.me/insightsIAStips 8 Youtube: https://www.youtube.com/channel/UCpoccbCX9GEIwaiIe4HLjwA

Revision Through MCQs (RTM) Compilation (June 2021)

12. Consider the following statements about National Commission for Protection of Child Rights (NCPCR): 1. It was set up under the Juvenile Justice (Care and Protection of Children) Act, 2015. 2. It works under the aegis of Ministry of Women and Child Development. 3. It is mandated to monitor and regulate in-country and inter-country adoptions. Which of the given above statements is/are correct? (a) 1 and 3 only (b) 2 only (c) 2 and 3 only (d) 1, 2 and 3 Ans: (b) Explanation: About NCPCR: • S1: Set up in March 2007 under the Commission for Protection of Child Rights Act, 2005. • S: It works under the administrative control of the Ministry of Women & Child Development. • S3: The Commission’s Mandate is to ensure that all Laws, Policies, Programmes, and Administrative Mechanisms are in consonance with the Child Rights perspective as enshrined in the Constitution of India and also the UN Convention on the Rights of the Child.

Refer: https://www.insightsonindia.com/2021/06/02/national-commission-for-protection- of-child-rights-ncpcr-2/

13. Under the RTE Act, 2009, the National Commission for Protection of Child Rights (NCPCR) can: 1. inquire into complaints about violation of the law. 2. summon an individual and demand evidence. 3. seek a magisterial enquiry. 4. file a writ petition in the High Court or Supreme Court. 5. recommend interim relief to those affected. Select the correct answer using the code below: (a) 1, 2 and 3 only (b) 1, 2, 3 and 4 only (c) 2 and 5 only (d) 1, 2, 3, 4 and 5 Ans: (d) Explanation: • Under the RTE Act, 2009, the NCPCR can inquire into complaints about violation of the law and can summon an individual, demand evidence, seek a magisterial enquiry as well as file a writ petition in the High Court or Supreme Court. It can also approach the government concerned for prosecution of the offender and recommend interim relief to those affected.

Refer: https://www.insightsonindia.com/2021/06/02/national-commission-for-protection- of-child-rights-ncpcr-2/

Telegram: https://t.me/insightsIAStips 9 Youtube: https://www.youtube.com/channel/UCpoccbCX9GEIwaiIe4HLjwA

Revision Through MCQs (RTM) Compilation (June 2021)

14. With reference to Maintenance and Welfare of Parents and Senior Citizens Act, 2007, consider the following statements: 1. This Act makes it legally obliging for adult children and heirs to provide for parents by way of a monthly allowance. 2. This Act provides an inexpensive and speedy procedure to claim monthly maintenance for parents and senior citizens. 3. According to this Act, parents could mean biological but not adoptive or step- parents. Which of the given above statements is/are correct? (a) 1 and 3 only (b) 2 and 3 only (c) 1 and 2 only (d) 1, 2 and 3 Ans: (c) Explanation: • Overview of the Maintenance and Welfare of Parents and Senior Citizens Act, 2007: • This Act makes it legally obliging for adult children and heirs to provide for parents by way of a monthly allowance. • This Act provides an inexpensive and speedy procedure to claim monthly maintenance for parents and senior citizens. • According to this Act, parents could mean biological, adoptive or step- parents. • Under this Act, there are also provisions to protect the life and property of such persons (elderly).

Refer: https://www.insightsonindia.com/2021/06/02/maintenance-and-welfare-of- parents-and-senior-citizens-act-2007/

15. Consider the following statements about “17+1” initiative: 1. It is a China-led format founded in 2012 in Budapest. 2. It aims to expand cooperation between Beijing and the Central and Southeastern Asia member countries. Which of the given above statements is/are correct? (a) 1 only (b) 2 only (c) Both 1 and 2 (d) Neither 1 nor 2 Ans: (a) Explanation: • The 17+1 initiative is a China-led format founded in 2012 in Budapest with an aim to expand cooperation between Beijing and the Central and Eastern European (CEE) member countries, with investments and trade for the development of the CEE region. • The framework also focuses on infrastructure projects such as bridges, motorways, railway lines and modernisation of ports in the member states.

Telegram: https://t.me/insightsIAStips 10 Youtube: https://www.youtube.com/channel/UCpoccbCX9GEIwaiIe4HLjwA

Revision Through MCQs (RTM) Compilation (June 2021)

• Refer: https://www.insightsonindia.com/2021/06/02/china-and-central-eastern- european-cee-171-mechanism/

16. Consider the following statements about Securities and Exchange Board of India: 1. It is a regulatory body for securities and commodity market in India. 2. It works under the jurisdiction of Ministry of Finance, Government of India. 3. It issues Participatory Notes or P-Notes (PNs) to overseas investors. Which of the given above statements is/are correct? (a) 1 and 3 only (b) 2 only (c) 1, 2 and 3 (d) 1 and 2 only Ans: (d) Explanation: • S3: P-notes are issued by registered foreign portfolio investors (FPIs) to overseas investors who wish to be part of the Indian stock market without registering themselves directly. • S2 and 1: Sebi is the regulatory body for securities and commodity market in India under the jurisdiction of Ministry of Finance , Government of India.

Refer: https://www.insightsonindia.com/2021/06/02/what-is-the-ipo-grey-market/

17. Consider the following statements: As per the National Security Act, the grounds for preventive detention of a person include: 1. acting in any manner prejudicial to the defence of India. 2. strong criticism against government policies and personalities. 3. depictions of an unresponsive or insensitive regime. Which of the statements given above is/are correct? (a) 1 and 3 only (b) 1 only (c) 2 and 3 only (d) 1, 2 and 3 Ans: (b) Explanation: • As per the National Security Act, the grounds for preventive detention of a person include: o acting in any manner prejudicial to the defence of India, the relations of India with foreign powers, or the security of India.

Telegram: https://t.me/insightsIAStips 11 Youtube: https://www.youtube.com/channel/UCpoccbCX9GEIwaiIe4HLjwA

Revision Through MCQs (RTM) Compilation (June 2021)

o regulating the continued presence of any foreigner in India or with a view to making arrangements for his expulsion from India. o preventing them from acting in any manner prejudicial to the security of the State or from acting in any manner prejudicial to the maintenance of public order or from acting in any manner prejudicial to the maintenance of supplies and services essential to the community it is necessary so to do. • Sedition’ is an offence incorporated into the Indian Penal Code (IPC) in 1870. Section 124A of the IPC defines sedition and says: o whoever by words either spoken or written or by signs or by visible representation or otherwise brings or attempts to bring into hatred or contempt, the government established by law; or o whoever by the above means excites or attempts to excite disaffection towards the government established by law, has committed the offence of sedition. • Under the present law, it offers scope to consider as seditious o strong criticism against government policies and personalities o slogans voicing disapprobation of leaders o depictions of an unresponsive or insensitive regime

Refer: https://www.insightsonindia.com/2021/06/02/national-security-act-nsa-1980/

18. Consider the following statements about the United Nations Human Rights Council (UNHRC): 1. It is a specialised agency of the United Nations. 2. It has more than 50 members elected for staggered three-year terms on a regional group basis. 3. It passes binding resolutions on human rights issues through a periodic review of all 193 UN member states. Which of the given above statements is/are not correct? (a) 1 and 2 only (b) 2 and 3 only (c) 1 and 3 only (d) 1, 2 and 3 Ans: (d) Explanation: Here the directive word is not correct!! • S1: It is not a specialised agency of the UN (list of UN Specialized Agencies) • S2: The UNHRC has 47 members serving at any time with elections held to fill up seats every year, based on allocations to regions across the world to ensure geographical representation. • S3: The UNHRC passes non-binding resolutions on human rights issues through a periodic review of all 193 UN member states called the Universal Periodic Review (UPR). • Who are UN Special Rapporteurs? o They are independent experts working on behalf of the United Nations. They work on a country or a thematic mandate specified by the United Nations Human Rights Council.

Telegram: https://t.me/insightsIAStips 12 Youtube: https://www.youtube.com/channel/UCpoccbCX9GEIwaiIe4HLjwA

Revision Through MCQs (RTM) Compilation (June 2021)

o They have mandates to report and advise on human rights (civil, cultural, economic, political, and social) from a thematic or country-specific perspective. o Rapporteurs do not receive any financial compensation for their work from the United Nations.

Refer: facts for prelims: https://www.insightsonindia.com/2021/06/02/insights-daily- current-affairs-pib-summary-2-june-2021/

19. Consider the following statements regarding National Human Rights Commission. 1. It was established by a Government resolution, for the protection and promotion of human rights. 2. It has its own investigating staff for investigation into complaints of human rights violations. 3. It has no power to punish the violators of human rights. Which of the above statements is/are correct? (a) 1 and 2 only (b) 1 and 3 only (c) 2 and 3 only (d) 1, 2 and 3 Ans: (c) Explanation: • NHRC, established in 1993, is an independent statutory body as per the provisions of the Protection of Human Rights Act of 1993. • The Commission has its own investigating staff headed by a Director General of Police for investigation into complaints of human rights violations. • While inquiring into complaints under the Act, the Commission shall have all the powers of a civil court trying a suit under the Code of Civil Procedure, 1908. • It has no power to punish the violators of human rights. It has no power to award any monetary relief.

Refer: facts for prelims: https://www.insightsonindia.com/2021/06/02/insights-daily- current-affairs-pib-summary-2-june-2021/

20. With reference to “Retrofit of Air-conditioning to improve Indoor Air Quality for Safety and Efficiency” (RAISE) initiative, consider the following statements: 1. It is a joint initiative of Bureau of Energy Efficiency (BEE) and the U.S. Agency for International Development’s (USAID) MAITREE Program. 2. It aims to focus on improving indoor air quality (IAQ), thermal comfort, and energy efficiency (EE) in workspaces across the nation. Which of the given above statements is/are correct? (a) 1 only (b) 2 only (c) Both 1 and 2 (d) Neither 1 nor 2 Ans: (b) Explanation: • S1: It is a joint initiative of Energy Efficiency Services Limited (EESL) and the S. Agency for International Development’s (USAID) MAITREE Program.

Telegram: https://t.me/insightsIAStips 13 Youtube: https://www.youtube.com/channel/UCpoccbCX9GEIwaiIe4HLjwA

Revision Through MCQs (RTM) Compilation (June 2021)

• S2: RAISE initiative can potentially alleviate the issue of bad air quality in workspaces across the nation and pioneer ways to make them healthier and greener.

Refer: https://www.insightsonindia.com/wp-content/uploads/2021/04/INSTA-PT-2021- Exclusive-Environment.pdf

RTM- REVISION THROUGH MCQS –3rd -Jun-2021

21. Which of the following statements is true about the Model Tenancy Act (MTA) in India? 1. The new Act seeks to cover urban and as well as rural areas. 2. The new Act will not affect existing tenancies. 3. After enforcement of this Act, no person can let or take on rent any premises except by an agreement in writing. Select the correct answer using the code below: (a) 1 and 2 only (b) 2 and 3 only (c) 1 and 3 only (d) 1, 2 and 3 Ans: (d) Explanation: • S1: The Act seeks to cover urban and as well as rural areas. • S2: The new Act will be applicable prospectively and will not affect existing tenancies. • S3: After enforcement of this Act, no person can let or take on rent any premises except by an agreement in writing.

Refer: https://www.insightsonindia.com/2021/06/03/model-tenancy-act/

22. With reference to India’s decision to levy 2 per cent Digital Services Tax or DST on revenues generated from digital services by non-resident entities and others, which of the following statements is/are correct? 1. It is introduced as a part of the Income Tax Act. 2. The levy applied only on digital advertising services. Select the correct answer using the code given below: (a) 1 only (b) 2 only (c) Both 1 and 2 (d) Neither 1 nor 2 Ans: (d) Explanation: • India’s 2 per cent DST is levied on revenues generated from digital services offered in India. • S1: It is part of the Finance act 2020. • S2: Applicability: o India has expanded the scope of the equalisation levy over the last few years, to tax non-resident digital entities.

Telegram: https://t.me/insightsIAStips 14 Youtube: https://www.youtube.com/channel/UCpoccbCX9GEIwaiIe4HLjwA

Revision Through MCQs (RTM) Compilation (June 2021)

o While the levy applied only to digital advertising services till 2019- 20 at the rate of 6 percent, the government in April last year widened the scope to impose a 2 per cent tax on non-resident e-commerce players with a turnover of Rs 2 crore. o The scope was further widened in the Finance Act 2021-22 to cover e- commerce supply or service when any activity takes place online. o Since May 2021, this also includes any entity that systematically and continuously does business with more than 3 lakh users in India.

Refer: https://www.insightsonindia.com/2021/06/03/digital-tax-in-india/

23. With reference to agriculture in India, how can the technique of `genome sequencing’, often seen in the news, be used in the immediate future? 1. Genome sequencing can be used to identify genetic markers for disease resistance and drought tolerance in various crop plants. 2. This technique helps in reducing the time required to develop new varieties of crop plants. 3. It can be used to decipher the host-pathogen relationships in crops. Select the correct answer using the code given below: (a) 1 only (b) 2 and 3 only (c) 1 and 3 only (d) 1, 2 and 3 Ans: (d) Explanation: • Justification: Genome sequencing is figuring out the order of DNA nucleotides, or bases, in a genome—the order of As, Cs, Gs, and Ts that make up an organism’s DNA. • Justification: Statement 1: Currently available newborn screening (genome) for childhood diseases allows detection of rare disorders that can be prevented or better treated by early detection and intervention. • https://www.scientificamerican.com/article/whole-genome-sequencing-predict- disease/ • Statement 2: Naturally if the gene potential can be identified by screening and sequencing, it will help in better genetic engineering. • Statement 3: This can be reasoned logically based on the above. • https://en.wikipedia.org/wiki/Whole_genome_sequencing

Refer: https://www.insightsonindia.com/2021/06/03/only-delta-is-a-variant-of-concern- who/

24. Consider the following statements regarding United Nations General Assembly (UNGA): 1. It is the main deliberative, policymaking and representative organ of the United Nations (UN). 2. It is the only UN body with universal representation. 3. Its president is elected every two year by assembly to serve a two-year term of office. Which of the given above statements is/are correct? (a) 1 and 2 only (b) 2 and 3 only (c) 1 and 3 only

Telegram: https://t.me/insightsIAStips 15 Youtube: https://www.youtube.com/channel/UCpoccbCX9GEIwaiIe4HLjwA

Revision Through MCQs (RTM) Compilation (June 2021)

(d) 1, 2 and 3 Ans: (a) Explanation: About UNGA: • The General Assembly is the main deliberative, policymaking and representative organ of the UN. • All 193 Member States of the UN are represented in the General Assembly, making it the only UN body with universal representation. • The President of the General Assembly is elected each year by assembly to serve a one-year term of office. • The presidency rotates annually between the five geographic groups: African, Asia-Pacific, Eastern European, Latin American and Caribbean, and Western European and other States.

Refer: https://www.insightsonindia.com/2021/06/03/unga-head/

25. Consider the following statements about Bureau of Indian Standards (BIS): 1. It is the only national body that frames standards in India. 2. It works under the aegis of Ministry of Commerce and Industry, Government of India. Select the correct answer using the code given below: (a) 1 only (b) 2 only (c) Both 1 and 2 (d) Neither 1 nor 2 Ans: (a) Explanation: About the Bureau of Indian Standards (BIS): • BIS is the only national body that frames standards. • It works under the aegis of Ministry of Consumer Affairs, Food & Public Distribution, Government of India. • BIS is responsible for the harmonious development of the activities of standardization, marking and quality certification of goods and for matters connected therewith or incidental thereto.

Refer: https://www.insightsonindia.com/2021/06/03/research-design-standards- organization-rdso/

26. With reference to china’s ‘Artificial Sun’ experiment, consider the following statements: 1. The mission mimics the energy generation process of the sun. 2. This project is part of the International Thermonuclear Experimental Reactor (ITER) facility. 3. As of now, china is the only country that has achieved high plasma temperatures. Which of the given above statements is/are correct? (a) 1 and 2 only (b) 2 and 3 only (c) 1 and 3 only (d) 1, 2 and 3 Ans: (a) Explanation: • S1: The mission mimics the energy generation process of the sun.

Telegram: https://t.me/insightsIAStips 16 Youtube: https://www.youtube.com/channel/UCpoccbCX9GEIwaiIe4HLjwA

Revision Through MCQs (RTM) Compilation (June 2021)

o The reactor consists of an advanced nuclear fusion experimental research device located in Hefei, China. • S2: The EAST project is part of the International Thermonuclear Experimental Reactor (ITER) facility, which will become the world’s largest nuclear fusion reactor when it becomes operational in 2035. o The ITER project includes the contributions of several countries, including India, South Korea, Japan, Russia and the United States. • S3: China is not the only country that has achieved high plasma temperatures. In 2020, South Korea’s KSTAR reactor set a new record by maintaining a plasma temperature of over 100 million degrees Celsius for 20 seconds.

Refer: https://www.insightsonindia.com/2021/06/03/chinese-artificial-sun-experimental- fusion-reactor-sets-new-world-record/

27. The National Disaster Management Authority (NDMA) is headed by the: (a) Prime Minister of India (b) President of India (c) Home Minister of India (d) Environment Minister of India Ans: (a) Explanation: • On March 24, 2020, the Centre, through the National Disaster Management Authority (NDMA) headed by the Prime Minister, invoked the provisions of the Act to streamline the management of the pandemic, empowering district magistrates to take decisions and centralise other decisions on the supply of oxygen and movement of vehicles.

Refer: https://www.insightsonindia.com/2021/06/03/dm-act-is-of-2005-vintage-was-first- enforced-during-pandemic/

28. In which one of the following state is Sakteng wildlife sanctuary located? (a) Arunachal Pradesh (b) Manipur (c) Meghalaya (d) None of the above Ans: (d) Explanation: • Sakteng is based in Eastern Bhutan, or Trashigang Dzongkhag (district) that borders Arunachal Pradesh. • It protects several endemic speciesincluding the eastern blue pine and the black- rumped magpie. • It was created in part to protect the migoi, a yeti-like cryptid whose existence has not been scientifically confirmed, but in which the local population strongly believes.

Telegram: https://t.me/insightsIAStips 17 Youtube: https://www.youtube.com/channel/UCpoccbCX9GEIwaiIe4HLjwA

Revision Through MCQs (RTM) Compilation (June 2021)

• Refer: https://www.insightsonindia.com/wp-content/uploads/2021/04/INSTA-PT-2021- Exclusive-Environment.pdf

29. Consider the following statements: 1. It is located in the Dibrugarh and Tinsukia districts of Assam. 2. It forms the largest stretch of lowland rainforest in the country. 3. It was declared as Elephant Reserve under Project Elephant. The above given statements refers to which one of the following protected area? (a) Dibru-Saikhowa National Park (b) Orang National Park (c) Dehing Patkai National Park (d) Kaziranga National Park Ans: (c) Explanation: • Assam government has decided to upgrade the Dehing Patkai Wildlife Sanctuary into a national park. • About Dehing Patkai Wildlife Sanctuary: o Also known as the Jeypore Rainforest is a part of Dehing Patkai Elephant Reserve . o Located in Dibrugarh and Tinsukia districts, the 111.19 sq km Dehing Patkai was declared a wildlife sanctuary in 2004. o It is home to 47 mammal, 47 reptile, and 310 butterfly species. o It forms the largest stretch of lowland rainforest in the country.

Refer: https://www.insightsonindia.com/wp-content/uploads/2021/04/INSTA-PT-2021- Exclusive-Environment.pdf

30. Which of the following pairs is/are correctly matched? 1. Mansar lake : Haryana 2. Lonar lake : Maharashtra 3. Startsapuk Tso: Sikkim Select the correct answer using the code below: (a) 3 only (b) 2 only (c) 1 and 2 only (d) 1, 2 and 3 Ans: (b) Explanation: • Startsapuk Tso: Ladakh

Telegram: https://t.me/insightsIAStips 18 Youtube: https://www.youtube.com/channel/UCpoccbCX9GEIwaiIe4HLjwA

Revision Through MCQs (RTM) Compilation (June 2021)

• Lonar lake: Maharashtra • Mansar lake: Jammu

Refer: https://www.insightsonindia.com/wp-content/uploads/2021/04/INSTA-PT-2021- Exclusive-Environment.pdf

RTM- REVISION THROUGH MCQS –4th -Jun-2021

31. Consider the following statements about Central Vigilance Commission: 1. It was set up by the Government in 1964 on the recommendations of the First Administrative Reforms Commission. 2. It has the status of an autonomous body, free of control from any Ministry/ Department. 3. It submits its report to the President of India. Which of the given above statements is/are correct? (a) 3 only (b) 2 and 3 only (c) 1 and 2 only (d) 1, 2 and 3 Ans: (b) Explanation: About CVC: • The CVC was set up by the Government in February, 1964 on the recommendations of the Committee on Prevention of Corruption, headed by Shri K. Santhanam. • In 2003, the Parliament enacted CVC Act conferring statutory status on the CVC. • The CVC is not controlled by any Ministry/Department. It is an independent body which is only responsible to the Parliament. • It submits its report to the President of India. • It exercises superintendence over the functioning of the Delhi Special Police Establishment (CBI) insofar as it relates to the investigation of offences under the Prevention of Corruption Act, 1988.

Refer: https://www.insightsonindia.com/2021/06/04/norms-for-employing-retired- officials-defined/

32. Which of the following is/are the key features of the Mahatma Gandhi National Rural Employment Guarantee Act? 1. The State Employment Agency after due verification will issue a Job Card 2. A minimum of 10 job seekers shall apply to sanction a new work under MGNREGA 3. At least one-half beneficiaries shall be women who have registered and requested work under the scheme Select the correct answer using the code below: (a) 3 only (b) 2 and 3 only (c) 2 only (d) 1 and 2 only Ans: (c)

Telegram: https://t.me/insightsIAStips 19 Youtube: https://www.youtube.com/channel/UCpoccbCX9GEIwaiIe4HLjwA

Revision Through MCQs (RTM) Compilation (June 2021)

Explanation: • S2: A minimum of 10 job seekers shall apply to sanction a new work under MGNREGA • S1: The Gram Panchayat after due verification will issue a Job Card. The Job Card will bear the photograph of all adult members of the household willing to work under MGNREGA and is free of cost • S3: At least one-third beneficiaries shall be women who have registered and requested work under the scheme. • Refer: more salient features: https://vikaspedia.in/social-welfare/rural-poverty- alleviation-1/schemes/faqs-on-mgnrega/mahatma-gandhi-national-rural- employment-guarantee-act

Refer: https://www.insightsonindia.com/2021/06/04/caste-categories-for-nregs-pay/

33. The “Balfour Declaration” is sometimes mentioned in the news in the context of the affairs of (a) Afghanistan (b) Palestine (c) Columbia (d) Ans: (b) Explanation: • The Balfour Declaration was issued after Britain gained control with the aim of establishing a home for the Jews in Palestine. However during that period the Arabs were in majority in Palestine.

Refer: https://www.insightsonindia.com/2021/06/04/palestine-flays-indias-abstention- from-vote/

34. Consider the following statements: 1. Venus is the second planet from the sun. 2. Venus is one of just two planets that rotate from east to west. 3. Recently, SpaceX announced ‘Davinci+’ and ‘Veritas’ mission to Venus. Which of the given above statements is/are correct? (a) 1 only (b) 2 only (c) 1 and 2 only (d) 1, 2 and 3 Ans: (c) Explanation: • S1: Venus is the second planet from the sun and the hottest planet in the solar system with a surface temperature of 500C – high enough to melt lead. • S2: Venus is one of just two planets that rotate from east to west. Only Venus and Uranus have this “backwards” rotation. • S3: NASA has announced two new missions (‘Davinci+’ and ‘Veritas’) to Venus.

Refer: https://www.insightsonindia.com/2021/06/04/nasa-announces-two-new-missions- to-venus/

35. With reference to historic missions to Venus, which of the following pairs is/are correctly matched?

Telegram: https://t.me/insightsIAStips 20 Youtube: https://www.youtube.com/channel/UCpoccbCX9GEIwaiIe4HLjwA

Revision Through MCQs (RTM) Compilation (June 2021)

1. Venus Express – NASA mission 2. Magellan – European mission 3. Akatsuki– Japanese spacecraft Select the correct answer using the code below: (a) 1 only (b) 2 and 3 only (c) 3 only (d) 1, 2 and 3 Ans: (c) Explanation: • Historic missions to Venus: o Magellan – a Nasa mission that ended in 1994. o Venus Express– A European mission- focused on atmospheric science. o Akatsuki– Japanese spacecraft- focused on atmospheric science. • Future missions: o The European Space Agency (Esa) is evaluating a Venus mission, called EnVision, alongside two astronomy proposals – Theseus and Spica. Other concepts are also being proposed to Nasa.

Refer: https://www.insightsonindia.com/2021/06/04/nasa-announces-two-new-missions- to-venus/

36. Consider the following statements: 1. Agricultural soils release nitrogen oxides into environment. 2. Cattle release ammonia into environment. 3. Poultry industry releases reactive nitrogen compounds into environment. Which of the statements given above is/are correct? (a) 1 and 3 only (b) 2 and 3 only (c) 2 only (d) 1, 2 and 3 Ans: (d) Explanation: • Agricultural soils contributed to over 70% of N2O emissions from India in 2010, followed by waste water (12%) and residential and commercial activities (6%). Since 2002, N2O has replaced methane as the second largest Greenhouse Gas (GHG) from Indian agriculture. • Cattle account for 80% of the ammonia production, though their annual growth rate is 1%, due to a stable population. • The poultry industry, on the other hand, with an annual growth rate of 6%, recorded an excretion of reactive nitrogen compounds of 0.415 tonnes in 2016.

Refer: https://www.insightsonindia.com/2021/06/04/eighth-global-nitrogen-conference/

37. Consider the following statements regarding the International Nitrogen Initiative Conference: 1. It was set up in 2001 by a shared partnership between the International Geosphere- Biosphere Program (IGBP) and the Intergovernmental Panel on Climate Change (IPCC). 2. It aims to optimize nitrogen’s beneficial role in sustainable food production.

Telegram: https://t.me/insightsIAStips 21 Youtube: https://www.youtube.com/channel/UCpoccbCX9GEIwaiIe4HLjwA

Revision Through MCQs (RTM) Compilation (June 2021)

Which of the given above statements is/are correct? (a) 1 only (b) 2 only (c) Both 1 and 2 (d) Neither 1 nor 2 Ans: (b) Explanation: • The International Nitrogen Initiative (INI) is an international program, set up in 2003 under sponsorship of the Scientific Committee on Problems of the Environment (SCOPE) and from the International Geosphere-Biosphere Program (IGBP). The key aims of the INI are to: o optimize nitrogen’s beneficial role in sustainable food production, and o minimize nitrogen’s negative effects on human health and the environment resulting from food and energy production. • The program is currently a sustained partner of Future Earth.

Refer: https://www.insightsonindia.com/2021/06/04/eighth-global-nitrogen-conference/

38. Consider the following statements: 1. Tarballs are dark-coloured, sticky balls of oil that form when crude oil floats on the ocean surface. 2. Oilzapper technology for bio-remediation of oily sludge and oil spills was developed by DRDO. Which of the given above statements is/are correct? (a) 1 only (b) 2 only (c) Both 1 and 2 (d) Neither 1 nor 2 Ans: (a) Explanation: • S1: Tarballs are dark-coloured, sticky balls of oil that form when crude oil floats on the ocean surface. Tarballs are formed by weathering of crude oil in marine environments. They are transported from the open sea to the shores by sea currents and waves. o Most of the times, the presence of several tarballs indicate an oil spill. However, its annual occurrence on the west coast during the monsoon has led marine biologists and experts to demand an investigation in the matter. • S2: Oilzapper technology o Developed by TERI after seven years of research work and partly supported by the DBT (Department of Biotechnology), Ministry of Science and Technology, Government of India, the Oilzapper is essentially a cocktail of five different bacterial strains that are immobilized and mixed with a carrier material (powdered corncob).

Refer: https://www.insightsonindia.com/2021/06/04/sri-lanka-braces-for-oil-spill-from- sinking-cargo-vessel/

Telegram: https://t.me/insightsIAStips 22 Youtube: https://www.youtube.com/channel/UCpoccbCX9GEIwaiIe4HLjwA

Revision Through MCQs (RTM) Compilation (June 2021)

39. The ‘Atlas Mountains’ are a series of mountain ranges which act as a divide between which of the following regions? (a) The and the Kalahandi desert (b) The and the Libyan desert (c) The Mediterranean basin and the Sahara desert (d) The Gulf of Sudra and the Katanga plateau Ans: (c) Explanation: • Britannica: Atlas Mountains, series of mountain ranges in northwestern Africa, running generally southwest to northeast to form the geologic backbone of the countries of the Maghrib (the western region of the Arab world)—Morocco, Algeria, and Tunisia. They extend from the Moroccan port of Agadir in the southwest, to the Tunisian capital of Tunis in the northeast. Their thick rim rises to form a high sill separating the Mediterranean basin to the north from the Sahara to the south, thus constituting a barrier that hinders, without completely preventing, communication between the two regions. Across, the mountains filter both air masses and human migrations.

• Refer: https://indianexpress.com/article/trending/trending-globally/sahara-snow-algeria- saudi-arabia-7152475/

40. The term ‘polar wandering’, recently seen in the news, refers to which of the following? (a) The ice loss from Greenland and Antarctica. (b) The process of exploration of the polar regions of Earth. (c) The migration of the magnetic poles over Earth's surface through geologic time. (d) The changing pole star, from north to south direction. Ans: (c) Explanation: • Polar wandering is the migration of the magnetic poles over Earth’s surface through geologic time. • It can be used, for example, to measure the degree to which Earth's magnetic poles have been observed to move relative to the Earth's rotation axis. • Also read: https://www.firstpost.com/science/melting-glaciers-due-to-climate- change-caused-earths-axis-to-shift-since-mid-90s-9573721.html

Refer: https://www.britannica.com/science/polar-wandering

Telegram: https://t.me/insightsIAStips 23 Youtube: https://www.youtube.com/channel/UCpoccbCX9GEIwaiIe4HLjwA

Revision Through MCQs (RTM) Compilation (June 2021)

RTM- REVISION THROUGH MCQS –5th -Jun-2021

41. The “Sustainable Development Goals (SDG) India Index” is released by which of the following? (a) National Statistical Office (NSO) (b) National Productivity Council (NPC) (c) National Innovation Foundation (NIF) (d) National Institution for Transforming India (NITI) Ans: (d) Explanation: • The NITI Aayog released the Baseline Report of the Sustainable Development Goals (SDG) India Index, which comprehensively documents the progress made by India's States and Union Territories towards implementing the 2030 SDG targets.

Refer: https://www.insightsonindia.com/2021/06/05/what-is-the-sdg-india-index/

42. Consider the following statements about ‘Industrial Deep Decarbonization Initiative’ (IDDI): 1. It is an initiative of UNEP and International Transport Forum (ITF). 2. India has been a member of IDDI since 2008. Which of the given above statements is/are correct? (a) 1 only (b) 2 only (c) Both 1 and 2 (d) Neither 1 nor 2 Ans: (d) Explanation: What is Industrial Deep Decarbonization Initiative (IDDI)? • It is a global coalition of public and private organisations who are working to stimulate demand for low carbon industrial materials. • In collaboration with national governments, IDDI works to standardise carbon assessments, establish ambitious public and private sector procurement targets, incentivise investment into low-carbon product development and design industry guidelines. • Coordinated by United Nations Industrial Development Organization (UNIDO). • Members: The IDDI is co-led by the UK and India and current members include Germany and Canada.

Refer: https://www.insightsonindia.com/2021/06/05/clean-energy-ministerials-cem- industrial-deep-decarbonization-initiative-iddi/

43. The Clean Energy Ministerial (CEM), a high-level global forum to promote policies and programs that advance the deployment of clean energy technology, emerged at: (a) The Earth Summit on Sustainable Development 2002, Johannesburg (b) The United Nations Framework Convention on Climate Change 2009, Copenhagen (c) The United Nations Conference on Sustainable Development 2012, Rio de Janeiro (d) The World Sustainable Development Summit 2016, New Delhi Ans: (b)

Telegram: https://t.me/insightsIAStips 24 Youtube: https://www.youtube.com/channel/UCpoccbCX9GEIwaiIe4HLjwA

Revision Through MCQs (RTM) Compilation (June 2021)

Explanation: About Clean Energy Ministerial (CEM): • Establishment: • It was established in December 2009 at the UN’s Framework Convention on Climate Change conference of parties in Copenhagen. • CEM is a high-level global forum to promote policies and programs that advance clean energy technology, to share lessons learned and best practices, and to encourage the transition to a global clean energy economy. • 29 countries are part of CEM including India.

Refer: https://www.insightsonindia.com/2021/06/05/clean-energy-ministerials-cem- industrial-deep-decarbonization-initiative-iddi/

44. The ‘Five to 50’ service, sometimes seen in the news, is related to (a) Hospitality Management and Tourism (b) Waste Management (c) Wireless Communication (d) Solar Power Production Ans: (c) Explanation: About OneWeb’s LEO internet programme: • Using LEO satellites OneWeb seeks to offer connectivity across the UK, Alaska, Northern Europe, Greenland, the Arctic Seas and Canada. The company expects the service to be switched on before the end of the year. • It calls this programme the ‘Five to 50’ service of offering internet connectivity to all regions north of 50 degrees latitude.

Refer: https://www.insightsonindia.com/2021/06/05/internet-through-leo-satellites/

45. “They were discovered in 1773 by a German pastor named Johann August Ephraim Goeze. Three years later, the Italian clergyman and scientist Lazzaro Spallanzani discovered that they had superpowers. He added water to sediment from a rain gutter, and looked under a microscope. He found hundreds of little bear-shaped creatures swimming around.” The above paragraph refers to which one of the following micro-animals? (a) Rotifer (b) Loricifera (c) Gastrotrich (d) Tardigrade Ans: (d) Explanation: • Micro-animals are animals so small that they can be visually observed only under a microscope. • Tardigrades are one of nature's smallest animals. They are never more than 1.5 mm long, and can only be seen with a microscope. They are commonly known as "water bears". • Tardigrades were discovered in 1773 by a German pastor named Johann August Ephraim Goeze. Three years later, the Italian clergyman and scientist Lazzaro Spallanzani discovered that they had superpowers. • Spallanzani added water to sediment from a rain gutter, and looked under a microscope. He found hundreds of little bear-shaped creatures swimming

Telegram: https://t.me/insightsIAStips 25 Youtube: https://www.youtube.com/channel/UCpoccbCX9GEIwaiIe4HLjwA

Revision Through MCQs (RTM) Compilation (June 2021)

around. In his book "Opuscoli di Fisica Animale, e Vegetabile", he named them "il Tardigrado", meaning "slow-stepper", because they moved so slowly.

Refer: https://www.insightsonindia.com/2021/06/05/why-is-nasa-sending-water-bears- baby-squid-to-the-international-space-station/

46. Consider the following statements about International Space Station (ISS). 1. It is a habitable artificial satellite in the low Earth orbit. 2. It is the largest artificial object in space. 3. It serves as a microgravity and space environment research laboratory. Which of the above statements is/are correct? (a) 1 and 2 only (b) 2 only (c) 1 and 3 only (d) 1, 2 and 3 Ans: (d) Explanation: • The International Space Station (ISS) is a space station, or a habitable artificial satellite, in low Earth orbit. The ISS is now the largest artificial body in orbit. o The ISS serves as a microgravity and space environment research laboratory in which crew members conduct experiments in biology, human biology, physics, astronomy, meteorology and other fields. o The station is suited for the testing of spacecraft systems and equipment required for missions to the Moon and Mars. o The ISS programme is a joint project among five participating space agencies: NASA, Roscosmos, JAXA, ESA, and CSA. o The ownership and use of the space station is established by intergovernmental treaties and agreements. o The station is divided into two sections, the Russian Orbital Segment (ROS) and the United States Orbital Segment (USOS), which is shared by many nations

Refer: https://www.insightsonindia.com/2021/06/05/why-is-nasa-sending-water-bears- baby-squid-to-the-international-space-station/

47. Consider the following statements: 1. Hail is produced by cumulonimbus clouds. 2. Cumulonimbus clouds are delicate, feathery clouds that are made mostly of ice crystals. Which of the given above statements is/are correct? (a) 1 only (b) 2 only (c) Both 1 and 2 (d) Neither 1 nor 2 Ans: (a) Explanation: • S1: Hail is produced by cumulonimbus clouds, which are generally large and dark and may cause thunder and lightning.

Telegram: https://t.me/insightsIAStips 26 Youtube: https://www.youtube.com/channel/UCpoccbCX9GEIwaiIe4HLjwA

Revision Through MCQs (RTM) Compilation (June 2021)

• S2: More commonly known as thunderclouds, cumulonimbus is the only cloud type that can produce hail, thunder and lightning. The base of the cloud is often flat, with a very dark wall-like feature hanging underneath, and may only lie a few hundred feet above the Earth's surface. • Cirrus clouds are delicate, feathery clouds that are made mostly of ice crystals.

Refer: https://www.insightsonindia.com/2021/06/05/anti-hail-gun-test-by-himachal- pradesh/

48. Which of the following is/are the potential impacts associated with increasing Black Carbon in the atmosphere? 1. It prevents clouds from being formed 2. It accelerates melting of snow and ice 3. It reduces sunlight 4. It affects the plant productivity Select the correct answer using the code below: (a) 1, 2 and 3 only (b) 1, 3 and 4 only (c) 2 and 4 only (d) 1, 2, 3 and 4 Ans: (d) Explanation:

• Black carbon, or soot, is part of fine particulate air pollution (PM2.5) and contributes to climate change. • Black carbon is formed by the incomplete combustion of fossil fuels, wood and other fuels.

• • Black carbon is a short-lived climate pollutant with a lifetime of only days to weeks after release in the atmosphere. During this short period of time, black carbon can have significant direct and indirect impacts on the climate, the cryosphere (snow and ice), agriculture and human health.

Refer: https://www.insightsonindia.com/2021/06/05/strong-policies-on-black-carbon- can-sharply-cut-glacier-melt-world-bank-study/

49. Consider the following statements: 1. As per the right to education (RTE) Act, to be eligible for appointment as a teacher in a state, a person would be required to possess the minimum qualification laid down by the concerned State council of Teacher education.

Telegram: https://t.me/insightsIAStips 27 Youtube: https://www.youtube.com/channel/UCpoccbCX9GEIwaiIe4HLjwA

Revision Through MCQs (RTM) Compilation (June 2021)

2. As per the RTE Act, for teaching primary classes, a candidate is required to pass a Teacher Eligibility Test conducted in accordance with the National Council of Teacher Education guidelines. 3. In India, more than 90 % of teacher education institutions are directly under the State Governments. Which of the statements given above is/are correct? (a) 1 and 2 (b) 2 only (c) 1 and 3 (d) 3 only Ans: (b) Explanation: • S1 is incorrect and S2 is correct because under the Right of Children to Free and Compulsory Education (RTE), Act 2009, National Council of Teacher Education (NCTE), has laid down the minimum educational & professional qualifications for a person to be eligible for an appointment as a teacher for classes I-VIII, which are applicable to all schools imparting elementary education, including the schools under the State Governments and to qualify under a Teacher Eligibility Test (TET). • Teachers Eligibility Test: o Teachers Eligibility Test is the minimum qualification required in India for a person to be eligible for appointment as a teacher for Classes I to VIII. o The test is mandatory for teaching jobs in Indian government schools. o The TET is conducted by both the central and state governments of India. o The test is conducted to meet the objectives of the Right of Children to Free and Compulsory Education Act, 2009. • Why in News? o The Ministry of Education has extended the validity of the Teachers Eligibility Test (TET) qualifying certificate from seven years to life with retrospective effect from 2011.

Refer: facts for prelims: https://www.insightsonindia.com/2021/06/05/insights-daily- current-affairs-pib-summary-5-june-2021/

50. Which one of the following has launched SAGE Portal? (a) NITI Aayog (b) Ministry of Micro, Small and Medium Enterprises (MSME) (c) Ministry of Skill Development and Entrepreneurship (MSDE) (d) None of the above Ans: (d) Explanation: SAGE Portal: • The Minister of Social Justice and Empowerment, launched the SAGE (Seniorcare Aging Growth Engine) initiative and SAGE portal to support India’s elderly persons. • The portal will be a “one-stop access” of elderly care products and services by credible start-ups.

Telegram: https://t.me/insightsIAStips 28 Youtube: https://www.youtube.com/channel/UCpoccbCX9GEIwaiIe4HLjwA

Revision Through MCQs (RTM) Compilation (June 2021)

• The start-ups selected under SAGE will be those which will provide new innovative products and services to elderly persons in various areas like health, travel, finance, legal, housing, and food among others. • The Ministry of Social Justice & Empowerment will act as a facilitator for this scheme. • A fund of uptoRs.1 crore as one-time equity will be granted to each selected start-up.

Refer: facts for prelims: https://www.insightsonindia.com/2021/06/05/insights-daily- current-affairs-pib-summary-5-june-2021/

Telegram: https://t.me/insightsIAStips 29 Youtube: https://www.youtube.com/channel/UCpoccbCX9GEIwaiIe4HLjwA

Revision Through MCQs (RTM) Compilation (June 2021)

RTM- REVISION THROUGH MCQS – 7th -Jun-2021 51. Consider the following statements: 1. The Constitution of India empowers the Parliament to enact a law to provide for matters relating to citizenship. 2. The children of foreign diplomats posted in India can acquire Indian citizenship by birth. 3. All persons acquiring citizenship of India must take an oath of allegiance before they are registered as citizens of India. Which of the given above statements is/are correct? (a) 1 and 3 only (b) 1 only (c) 2 and 3 only (d) 1, 2 and 3 Ans: (a) Explanation: • S2: The children of foreign diplomats posted in India and enemy aliens cannot acquire Indian citizenship by birth. • S1: The Constitution deals with the citizenship from Articles 5 to 11 under Part II. It empowers the Parliament to enact a law to provide for such matters and any other matter relating to citizenship. • S3: All categories of persons acquiring citizenship of India must take an oath of allegiance before they are registered as citizens of India.

Refer: https://www.insightsonindia.com/2021/06/07/npr-slips-valid-for-long-term-visas/

52. Consider the following statements regarding the “Performance Grading Index”: 1. It is a survey mechanism used to measure the level of student participation at universities and colleges in India. 2. It is initiated by the Department of School Education and Literacy (DoSEL). Which of the given above statements is/are correct? (a) 1 only (b) 2 only (c) Both 1 and 2 (d) Neither 1 nor 2 Ans: (b) Explanation: • The Ministry of Education has released the Performance Grading Index for 2019-20. o Performance Grading Index is initiated by the Department of School Education and Literacy (DoSEL). • PGI is a tool to grade all States and UTs on their performance across 70 indicators on school education. • Objective: To encourage States & UTs to adopt best practices like online recruitment and transfer of teachers, electronic attendance of students & teachers.

Refer: https://www.insightsonindia.com/2021/06/07/education-index-ranking/

Telegram: https://t.me/insightsIAStips 30 Youtube: https://www.youtube.com/channel/UCpoccbCX9GEIwaiIe4HLjwA

Revision Through MCQs (RTM) Compilation (June 2021)

53. Consider the following statements: 1. In viral-vectored vaccines, a virus is used to carry the target antigen gene into human cells. 2. Covaxin is an adenovirus viral vector vaccine for COVID-19 jointly developed by Bharat Biotech and the Indian Council of Medical Research. Which of the given above statements is/are correct? (a) 1 only (b) 2 only (c) Both 1 and 2 (d) Neither 1 nor 2 Ans: (a) Explanation: • A vector, in infectious disease biology, is what works as a vehicle to transport a disease-causing agent. o In viral-vectored vaccines, a virus is used to carry the target antigen gene into human cells. o Covishield (the vaccine by Oxford University-AstraZeneca) is based on the viral-vectored platform. • Sputnik V is an adenovirus viral vector vaccine for COVID-19 developed by the Gamaleya Research Institute of Epidemiology and Microbiology. • Covaxin (jointly developed by Bharat Biotech and the Indian Council of Medical Research), is an inactivated vaccine.

Refer: https://www.insightsonindia.com/2021/06/07/more-antibodies-produced-by- covishield-than-covaxin-study/

54. With reference to Organisation for the Prohibition of Chemical Weapons (OPCW), consider the following statements: 1. It is a Paris-based autonomous intergovernmental organization. 2. It prohibits the use of chemical weapons and requires their destruction. 3. It was awarded the 2013 Nobel Peace Prize "for its extensive efforts to eliminate chemical weapons". Which of the given above statements is/are correct? (a) 1 only (b) 2 and 3 only (c) 1 and 2 only (d) 1, 2 and 3 Ans: (b) Explanation: • The Nobel Peace Prize for 2013 has been awarded to the Organisation for the Prohibition of Chemical Weapons (OPCW) “for its extensive efforts to eliminate chemical weapons”. • The Hague-based autonomous body, which works within the framework of the United Nations, was established in 1997 by the Chemical Weapons Convention to carry out its mandate. • The organisation promotes and verifies the adherence to the Chemical Weapons Convention, which prohibits the use of chemical weapons and requires their destruction. Verification consists both of evaluation of declarations by member states and onsite inspections.

Telegram: https://t.me/insightsIAStips 31 Youtube: https://www.youtube.com/channel/UCpoccbCX9GEIwaiIe4HLjwA

Revision Through MCQs (RTM) Compilation (June 2021)

Refer: https://www.insightsonindia.com/2021/06/07/syria-has-likely-used-chemical- weapons-17-times-opcw/

55. The CEO Water Mandate is a special initiative of (a) The UN Secretary-General and the UN Global Compact (b) The UNEP Secretariat and the UN Global Compact (c) The UNFCCC Secretariat and the UN Global Compact (d) None of the above Ans: (a) Explanation: • The CEO Water Mandate is a special initiative of the UN Secretary-General and the UN Global Compact, implemented in partnership with the Pacific Institute.

Refer: https://www.insightsonindia.com/2021/06/07/ntpc-joins-uns-ceo-water-mandate/

56. With reference to UN Global Compact initiative, consider the following statements: 1. It is a non-binding United Nations pact to encourage businesses and firms worldwide to adopt sustainable and socially responsible policies. 2. It is the world’s largest corporate sustainability initiative. Which of the given above statements is/are corcet? (a) 1 only (b) 2 only (c) Both 1 and 2 (d) Neither 1 nor 2 Ans: (c) Explanation: • The UN Global Compact is the world’s largest corporate sustainability initiative, with more than 12,000 corporate participants and stakeholders from more than 140 countries. • It is based on ten principles in the areas of human rights, labour standards, the environment, and anti-corruption. • It is a non-binding United Nations pact to encourage businesses and firms worldwide to adopt sustainable and socially responsible policies, and to report on their implementation. • Launched in 2000. • It is a principle-based framework for businesses, stating ten principles in the areas of human rights, labor, the environment and anti-corruption. • Under the Global Compact, companies are brought together with UN agencies, labor groups and civil society. • Cities can join the Global Compact through the Cities Programme.

Refer: https://www.insightsonindia.com/2021/06/07/ntpc-joins-uns-ceo-water-mandate/

57. The “Tiananmen square massacre” was in news recently, is mentioned in the news in the context of the affairs of: (a) Israel (b) China (c) Russia (d) Germany

Telegram: https://t.me/insightsIAStips 32 Youtube: https://www.youtube.com/channel/UCpoccbCX9GEIwaiIe4HLjwA

Revision Through MCQs (RTM) Compilation (June 2021)

Ans: (b) Explanation: Tiananmen square massacre: • Context: 32nd anniversary of the event was observed recently. • What happened? • On June 4, 1989 Chinese troops cracked down on pro-democracy protesters around Beijing’s Tiananmen Square. No official death toll has ever been released, but rights groups estimate hundreds, if not thousands were killed. • The protests were primarily student-led demonstrations calling for democracy, free speech and a free press in China.

Refer: facts for prelims: https://www.insightsonindia.com/2021/06/07/insights-daily- current-affairs-pib-summary-7-june-2021/

58. Consider the following statements: 1. Since 1974, World Environment Day has been celebrated every year on 5 June. 2. Every World Environment Day is hosted by a different country in which official celebrations take place and this year's host is Pakistan. Which of the given above statements is/are corcet? (a) 1 only (b) 2 only (c) Both 1 and 2 (d) Neither 1 nor 2 Ans: (c) Explanation: • The United Nations Assembly established World Environment Day in 1972, which was the first day of the Stockholm Conference on the human environment. • Since 1974, World Environment Day has been celebrated every year on 5 June, engaging governments, businesses and citizens in an effort to address pressing environmental issues. • The theme of World Environment Day 2021 is ‘Reimagine. Recreate. Restore.’ The theme is based on the UN Decade on Ecosystem Restoration, which will be launched by the United Nations this year. • Pakistan will be the global host for 2021.

Refer: facts for prelims: https://www.insightsonindia.com/2021/06/07/insights-daily- current-affairs-pib-summary-7-june-2021/

59. ‘YounTab scheme’ is sometimes mentioned in the news with reference to (a) social networking (b) online internship (c) digital learning (d) vocational training Ans: (c) Explanation: • Ladakh Lt Governor has launched the YounTab scheme for students in the Union Territory to encourage digital learning. • Under the scheme, 12,300 tablets with pre-loaded online and offline content, including textbooks, video lectures and online class applications, would be distributed to government school students from Class 6th to 12th.

Telegram: https://t.me/insightsIAStips 33 Youtube: https://www.youtube.com/channel/UCpoccbCX9GEIwaiIe4HLjwA

Revision Through MCQs (RTM) Compilation (June 2021)

Refer: facts for prelims: https://www.insightsonindia.com/2021/06/07/insights-daily- current-affairs-pib-summary-7-june-2021/

60. ‘I-Familia’ is a global database to identify missing persons, launched by: (a) Europol (b) INTERPOL (c) Central Bureau of Investigation (d) International Labour Organization Ans: (b) Explanation: I-Familia: • It is a Global database to identify missing persons. • Launched by Interpol. • It can identify missing persons through family DNA and help the police solve cold cases in member countries.

Refer: facts for prelims: https://www.insightsonindia.com/2021/06/07/insights-daily- current-affairs-pib-summary-7-june-2021/

RTM- REVISION THROUGH MCQS – 8th -Jun-2021 61. Consider the following statements: 1. Juvenile Justice (Care and Protection of Children) Act, 2015 mandates setting up Juvenile Justice Boards and Child Welfare Committees in every district. 2. As per the new amendments to JJ Act 2015, no new children’s home can be opened without the sanction of the district magistrates (DMs). Which of the given above statements is/are correct? (a) 1 only (b) 2 only (c) Both 1 and 2 (d) Neither 1 nor 2 Ans: (c) Explanation: • S1: JJ Act, 2015 mandates setting up Juvenile Justice Boards and Child Welfare Committees in every district. Both must have at least one-woman member each. • S2: Child Care Institutions can be government-run, government-aided, privately run or run through government, private or foreign funding. These institutions, while falling under the CWC and the state child protection units had very little oversight and monitoring. Even to receive a license, after an application was made, if the children’s home were to not receive a reply from the government within 3 months’ time, it would be “deemed registered’’ for a period of six months, even without government permission. The new amendment ensures that this can no longer happen and that no new children’s home can be opened without the sanction of the DM. • DM’s are also responsible now for ensuring that CCIs falling in their district are following all norms and procedures. During the NCPCR survey, for instance, it had been CCI’s with large funds, including foreign funding, had been found keeping children in unsanitary conditions in portacabins.

Refer: https://www.insightsonindia.com/2021/06/08/sc-urged-to-stop-illegal-adoption/

Telegram: https://t.me/insightsIAStips 34 Youtube: https://www.youtube.com/channel/UCpoccbCX9GEIwaiIe4HLjwA

Revision Through MCQs (RTM) Compilation (June 2021)

62. Consider the following statements about the Group of Seven (G7): 1. It is an intergovernmental organization made up of the world’s largest developed economies. 2. It has no legislative or authoritative power to enforce the recommended policies and plans it compiles. Which of the given above statements is/are correct? (a) 1 only (b) 2 only (c) Both 1 and 2 (d) Neither 1 nor 2 Ans: (c) Explanation: • The Group of Seven (G-7) is an intergovernmental organization that meets periodically to address international economic and monetary issues. • G-7 countries consist of the U.S., U.K., France, Germany, Italy, Canada, and Japan. • The G-7 was formerly referred to as the G-8 until Russia was suspended from the group in 2014 after illegally annexing Crimea. • The G-7 is not an official, formal entity and, therefore, has no legislative or authoritative power to enforce the recommended policies and plans it compiles.

Refer: https://www.insightsonindia.com/2021/06/08/g7-corporate-tax-deal/

63. With reference to Association of Southeast Asian Nations, consider the following statements: 1. It is an economic union comprising 10 member states in Southeast Asia. 2. It launched the South Asian Free Trade Area in 2006. 3. Its secretariat is based in Indonesia, Jakarta. Which of the given above statements is/are correct? (a) 1 only (b) 2 and 3 only (c) 1 and 3 only (d) 1, 2 and 3 Ans: (c) Explanation: • S1: The Association of Southeast Asian Nations is a regional organization which was established to promote political and social stability amid rising tensions among the Asia-Pacific’s post-colonial states. • S2: The South Asian Association for Regional Cooperation (SAARC) launched the South Asian Free Trade Area in 2006. • S3: ASEAN Secretariat – Indonesia, Jakarta.

Refer: https://www.insightsonindia.com/2021/06/08/china-hosts-asean-foreign- ministers/

64. Consider the following statements about National Asset Reconstruction Company Ltd (NARCL): 1. It was established under the Insolvency and Bankruptcy Code, 2016.

Telegram: https://t.me/insightsIAStips 35 Youtube: https://www.youtube.com/channel/UCpoccbCX9GEIwaiIe4HLjwA

Revision Through MCQs (RTM) Compilation (June 2021)

2. It has regulatory oversight over the Insolvency Professionals and Insolvency Professional Agencies in India. Which of the given above statements is/are correct? (a) 1 only (b) 2 only (c) Both 1 and 2 (d) Neither 1 nor 2 Ans: (d) Explanation: What is NARCL? • Setting up of NARCL, the proposed bad bank for taking over stressed assets of lenders, was announced in the Budget for 2021-22. • The plan is to create a bad bank to house bad loans of ₹500 crore and above, in a structure that will contain an asset reconstruction company (ARC) and an asset management company (AMC) to manage and recover dud assets. • The new entity is being created in collaboration with both public and private sector banks. • How is NARCL different from existing ARCs? How can it operate differently? o The proposed bad bank will have a public sector character since the idea is mooted by the government and majority ownership is likely to rest with state-owned banks. o At present, ARCs typically seek a steep discount on loans. With the proposed bad bank being set up, the valuation issue is unlikely to come up since this is a government initiative. o The government-backed ARC will have deep pockets to buy out big accounts and thus free up banks from carrying these accounts on their books.

Refer: https://www.insightsonindia.com/2021/06/08/national-asset-reconstruction- company-ltd-narcl/

65. With reference to “Sea Snot”, consider the following statements: 1. It is a collection of mucus-like organic matter found in the sea. 2. It is formed when algae are overloaded with nutrients. Which of the given above statements is/are correct? (a) 1 only (b) 2 only (c) Both 1 and 2 (d) Neither 1 nor 2 Ans: (c) Explanation: • Sea snot, sea saliva or marine mucilage is a collection of mucus-like organic matter found in the sea. • It is formed when algae are overloaded with nutrients as a result of water pollution combined with the effects of climate change.

Refer: https://www.insightsonindia.com/2021/06/08/what-is-the-sea-snot-outbreak-in- turkey/

Telegram: https://t.me/insightsIAStips 36 Youtube: https://www.youtube.com/channel/UCpoccbCX9GEIwaiIe4HLjwA

Revision Through MCQs (RTM) Compilation (June 2021)

66. With reference to Sea of Marmara, consider the following statements: 1. It is bordered by Bulgaria, Romania and Turkey. 2. It connects the Black Sea to the Aegean Sea. Which of the given above statements is/are correct? (a) 1 only (b) 2 only (c) Both 1 and 2 (d) Neither 1 nor 2 Ans: (b) Explanation: • Sea of Marmara is the inland sea, entirely within the borders of Turkey. • It connects the Black Sea to the Aegean Sea, thus separating Turkey's Asian and European lands. • The Bosphorus strait connects it to the Black Sea and the Dardanelles strait to the Aegean Sea. The former also separates Istanbul into its Asian and European sides.

• Refer: https://www.insightsonindia.com/2021/06/08/what-is-the-sea-snot-outbreak-in- turkey/

67. The Black Sea is bordered by which of the following countries? 1. Ukraine 2. Turkey 3. Greece 4. Georgia Select the correct answer using the code below: (a) 1, 2 and 4 only (b) 1, 3 and 4 only (c) 1, 2 and 3 only (d) 2, 3 and 4 only Ans: (a) Explanation:

Telegram: https://t.me/insightsIAStips 37 Youtube: https://www.youtube.com/channel/UCpoccbCX9GEIwaiIe4HLjwA

Revision Through MCQs (RTM) Compilation (June 2021)

• Refer: World Physical Map

68. In which one of the following states is Gumti Wildlife Sanctuary located? (a) (b) Nagaland (c) Assam (d) Arunachal Pradesh Ans: (a) Explanation: • Gumti Wildlife Sanctuary is a Wildlife Sanctuary in Tripura.

Refer: https://en.wikipedia.org/wiki/Nature_reserve

69. Consider the following statements regarding ‘Earth Day’: 1. It is an initiative of the World Wide Fund for Nature (WWF). 2. It is a movement in which the participants switch off the lights for one hour on a certain day every year. 3. It is a movement to raise the awareness about protection of the environment Which of the above statements is/are correct? (a) 1 and 2 only (b) 1 and 3 only (c) 3 only (d) 1, 2 and 3 Ans: (c) Explanation: • Earth Day is an annual event celebrated on April 22. Worldwide, various events are held to demonstrate support for environmental protection. First celebrated in 1970, Earth Day events in more than 193 countries are now coordinated globally by the Earth Day Network. • Earth Hour is a worldwide movement organized by the World Wide Fund for Nature (WWF). The event is held annually encouraging individuals, communities, and businesses to turn off non-essential electric lights for one hour, from 8:30 to 9:30 pm on a specific day towards the end of March, as a symbol of commitment to the planet. It was started as a lights-off event in Sydney, Australia, in 2007. Since then, it has grown to engage more than 7,000 cities and towns across 187 countries and territories.

Telegram: https://t.me/insightsIAStips 38 Youtube: https://www.youtube.com/channel/UCpoccbCX9GEIwaiIe4HLjwA

Revision Through MCQs (RTM) Compilation (June 2021)

Refer: https://www.earthday.org/ and https://www.earthhour.org/

70. Consider the following statements about Cartagena Protocol on Biosafety (CPB). 1. It is the first international regulatory framework for safe transfer, handling and use of Living Modified Organisms (LMOs). 2. It was negotiated under the aegis of United Nations Framework Convention on Climate Change (UNFCCC). 3. It establishes a Biosafety Clearing-House (BCH) to facilitate information exchange with special attention to developing countries. Which of the above statements is/are correct? (a) 1 and 3 only (b) 1 and 2 only (c) 2 and 3 only (d) 1, 2 and 3 Ans: (a) Explanation: • The Cartagena Protocol on Biosafety (CPB), the first international regulatory framework for safe transfer, handling and use of Living Modified Organisms (LMOs) was negotiated under the aegis of the Convention on Biological Diversity (CBD). The protocol was adopted on 29th January 2000. The Protocol entered into force on 11 September 2003. • The objective of the Protocol is to contribute to ensuring an adequate level of protection in the field of the safe transfer, handling and use of LMOs resulting from modern biotechnology that may have adverse effects on the conservation and sustainable use of biological diversity, taking also into account risks to human health, and specifically focusing on transboundary movements. • The Protocol establishes a Biosafety Clearing-House (BCH) to facilitate information exchange, and contains provisions on capacity building and financial resources, with special attention to developing countries and those without domestic regulatory systems.

Refer: https://bch.cbd.int/protocol/

RTM- REVISION THROUGH MCQS – 9th -Jun-2021 71. Consider the following statements: 1. The president of India can summon a session of parliament at such a place as he/she thinks fit. 2. The constitution of India provides for three sessions of the parliament in a year but it is not mandatory to conduct all the sessions. 3. There is no minimum number of days that the parliament is required to meet in a year. Which of the above given statements is/are correct? (a) 1 only (b) 2 only (c) 1 and 3 only (d) 2 and 3 only Ans: (c) Explanation:

Telegram: https://t.me/insightsIAStips 39 Youtube: https://www.youtube.com/channel/UCpoccbCX9GEIwaiIe4HLjwA

Revision Through MCQs (RTM) Compilation (June 2021)

• S1 and S2: Article 85(1) of the Constitution empowers the President to summon each House of Parliament to meet at such time and place as he thinks fit, but six months shall not intervene between its last sitting in one Session and the date appointed for its first sitting in the next Session. • S3: This is correct, refer to the explanation above. • http://legislative.gov.in/sites/default/files/coi-4March2016.pdf

Refer: https://www.insightsonindia.com/2021/06/09/monsoon-session-of-parliament- likely-to-begin-in-july/

72. Consider the following statements: 1. The Government of India Act, 1935, contained a provision relating to the summoning of the legislature in India. 2. The Constitution that came into force in 1950 required Parliament to be summoned twice in a year, with a gap of no more than six months between its sessions. Which of the given above statements is/are correct? (a) 1 only (b) 2 only (c) Both 1 and 2 (d) Neither 1 nor 2 Ans: (c) Explanation: • S2: The Constitution that came into force in 1950 required Parliament to be summoned twice in a year, with a gap of no more than six months between its sessions. The First Amendment changed this in 1951, and Article 85 got its present form. o Article 85 only requires that there should not be a gap of more than six months between two sessions of Parliament. • S1: The Government of India Act, 1935, contained a provision relating to the summoning of the legislature in India. It specified that the central legislature had to be summoned to meet at least once a year, and that not more than 12 months could elapse between two sessions.

Refer: https://www.insightsonindia.com/2021/06/09/monsoon-session-of-parliament- likely-to-begin-in-july/

73. With reference to Sixth Schedule of the Constitution of India, consider the following statements: 1. It consists of provisions for the administration of tribal areas in Assam, Meghalaya, Tripura and Mizoram 2. It seeks to safeguard the rights of tribal population through the formation of Autonomous District Councils (ADC). 3. It provides for separate Regional Councils for each area constituted as an autonomous region. Which of the given above statements is/are correct? (a) 1 and 2 only (b) 2 and 3 only (c) 1 and 3 only (d) 1, 2 and 3 Ans: (d)

Telegram: https://t.me/insightsIAStips 40 Youtube: https://www.youtube.com/channel/UCpoccbCX9GEIwaiIe4HLjwA

Revision Through MCQs (RTM) Compilation (June 2021)

Explanation: • S1: The Sixth Schedule consists of provisions for the administration of tribal areas in Assam, Meghalaya, Tripura and Mizoram, according to Article 244 of the Indian Constitution. • S2: Passed by the Constituent Assembly in 1949, it seeks to safeguard the rights of tribal population through the formation of Autonomous District Councils (ADC). ADCs are bodies representing a district to which the Constitution has given varying degrees of autonomy within the state legislature. • S3: Along with ADCs, the Sixth Schedule also provides for separate Regional Councils for each area constituted as an autonomous region. In all, there are 10 areas in the Northeast that are registered as autonomous districts – three in Assam, Meghalaya and Mizoram and one in Tripura. These regions are named as district council of (name of district) and regional council of (name of region).

Refer: https://www.insightsonindia.com/2021/06/09/rengma-nagas-demand- autonomous-council/

74. Consider the following statements: 1. The United Nations Charter established six main organs of the United Nations, including the United Nations Security Council (UNSC). 2. The UNSC is the only UN body with the authority to issue binding resolutions on member states. Which of the given above statements is/are correct? (a) 1 only (b) 2 only (c) Both 1 and 2 (d) Neither 1 nor 2 Ans: (c) Explanation: • S1: The United Nations Charter established six main organs of the United Nations, including the Security Council. It gives primary responsibility for maintaining international peace and security to the Security Council, which may meet whenever peace is threatened. • According to the Charter, the United Nations has four purposes: o to maintain international peace and security; o to develop friendly relations among nations; o to cooperate in solving international problems and in promoting respect for human rights; o and to be a centre for harmonizing the actions of nations. • S2: All members of the United Nations agree to accept and carry out the decisions of the Security Council. While other organs of the United Nations make recommendations to member states, only the Security Council has the power to make decisions that member states are then obligated to implement under the Charter.

Refer: https://www.insightsonindia.com/2021/06/09/un-security-council-endorses- secretary-general-guterres-for-second-term/

Telegram: https://t.me/insightsIAStips 41 Youtube: https://www.youtube.com/channel/UCpoccbCX9GEIwaiIe4HLjwA

Revision Through MCQs (RTM) Compilation (June 2021)

75. Consider the following statements regarding BRICS grouping. 1. Not all BRICS members are the members of G20. 2. Since 2009, the BRICS nations have met annually at formal summits. 3. New Development Bank (NDB) is a multilateral development bank operated by the BRICS countries with each country contributing equally to its equity. Which of the above statements is/are correct? (a) 1 and 2 only (b) 2 and 3 only (c) 1 and 3 only (d) 2 only Ans: (b) Explanation: • BRICS is the acronym coined for an association of five major emerging national economies: Brazil, Russia, India, China and South Africa. • The BRICS members are known for their significant influence on regional affairs; all are members of G20. Since 2009, the BRICS nations have met annually at formal summits. The 12th BRICS summit was hosted virtually by Russian President Vladimir Putin. • The main achievement of BRICS is the New Development Bank, with each country contributing equally to its equity.

Refer: https://www.insightsonindia.com/2021/06/09/brics-opposes-exceptionalism- china/

76. The “Operation Pangea XIV” was in news recently, is conducted by: (a) Wildlife Crime Control Bureau (b) National Tiger Conservation Authority (c) Narcotics Control Bureau (d) None of the above Ans: (d) Explanation: • A record number of fake online pharmacies have been shut down under Operation Pangea XIV targeting the sale of counterfeit and illicit medicines and medical products. • The operation coordinated by INTERPOL involved police, customs and health regulatory authorities from 92 countries.

Refer: facts for prelims: Facts for Prelims: https://www.insightsonindia.com/2021/06/09/insights-daily-current-affairs-pib- summary-9-june-2021/

77. Consider the following pairs: GI product State 1. Zardalu West Bengal 2. Chak-Hao Manipur 3. Arumbavur Wood Kerala Which of the above pairs is/are correctly matched? (a) 1 and 2 only (b) 2 only (c) 2 and 3 only

Telegram: https://t.me/insightsIAStips 42 Youtube: https://www.youtube.com/channel/UCpoccbCX9GEIwaiIe4HLjwA

Revision Through MCQs (RTM) Compilation (June 2021)

(d) 3 only Ans: (b) Explanation: • Sixteen varieties of mangoes including three GI certified varieties exported to Bahrain from West Bengal & Bihar. o These include GI certified Khirsapati & Lakshmanbhog (West Bengal), Zardalu (Bihar). • GI Tags in India 2020

State Product

Jammu & Kashmir Saffron (Mongra, Lachha, Guchhi) Tamil Nadu Kovilpatti Kadalai Mittai Tamil Nadu Thanjavur Pith Works Tamil Nadu Arumbavur Wood Carvings Uttar Pradesh Gorakhpur Terracotta Manipur Chak-Hao Telangana Telia Rumal Jharkhand Sohrai – Khovar Painting Refer: Refer: facts for prelims: Facts for Prelims: https://www.insightsonindia.com/2021/06/09/insights-daily-current-affairs-pib- summary-9-june-2021/

78. Consider the following statements about Geographical Indications (GI) tags: 1. Geographical Indications are part of the intellectual property rights that comes under the Paris Convention for the Protection of Industrial Property. 2. In India, Geographical Indications registration is administered by the Geographical Indications of Goods (Registration and Protection) Act of 1999. 3. The registration of a geographical indication is valid for a period of 20 years. Which of the given above statements is/are correct? (a) 1 and 2 only (b) 2 and 3 only (c) 1 and 3 only (d) 1, 2 and 3 Ans: (a) Explanation: • A GI is primarily an agricultural, natural or a manufactured product (handicrafts and industrial goods) originating from a definite geographical territory. • S1: Geographical Indications are part of the intellectual property rights that comes under the Paris Convention for the Protection of Industrial Property. • S2: In India, Geographical Indications registration is administered by the Geographical Indications of Goods (Registration and Protection) Act of 1999. • S3: The registration of a geographical indication is valid for a period of 10 years.

Refer: Facts for Prelims: https://www.insightsonindia.com/2021/06/09/insights-daily- current-affairs-pib-summary-9-june-2021/

Telegram: https://t.me/insightsIAStips 43 Youtube: https://www.youtube.com/channel/UCpoccbCX9GEIwaiIe4HLjwA

Revision Through MCQs (RTM) Compilation (June 2021)

79. Consider the following statements regarding Energy Efficiency Services Limited (EESL): 1. It was formed under India's Ministry of New and Renewable Energy to facilitate energy efficiency projects. 2. It is the world's largest public Energy Service Company (ESCO). Which of the given above statements is/are correct? (a) 1 only (b) 2 only (c) Both 1 and 2 (d) Neither 1 nor 2 Ans: (b) Explanation: • S2: Energy Efficiency Services Limited (EESL) is an energy service company (ESCO) of the Government of India and is the world's largest public ESCO. • S1: EESL was formed under India's Ministry of Power to facilitate energy efficiency projects. • CESL and Ladakh sign an MoU to make the Union Territory carbon neutral: o Convergence Energy Services Limited (CESL) has signed a Memorandum of Understanding (MoU) with the Administration of Union Territory (UT) of Ladakh, to make it a clean and green UT. o CESL is a wholly owned subsidiary of Energy Efficiency Services Limited (EESL) under Ministry of Power.

Refer: Facts for Prelims: https://www.insightsonindia.com/2021/06/09/insights-daily- current-affairs-pib-summary-9-june-2021/

80. “Surakshit Hum Surakshit Tum Abhiyaan” has been launched by (a) NITI Aayog (b) Ministry of Railways (c) Ministry of Road Transport and Highways (d) Ministry of Health and Family Welfare Ans: (a) Explanation: • NITI Aayog & Piramal Foundation have launched Surakshit Hum Surakshit Tum Abhiyaan in 112 Aspirational Districts. • Campaign Will Provide Covid Home-Care Support to 20 Lakh Citizens.

Refer: Facts for Prelims: https://www.insightsonindia.com/2021/06/09/insights-daily- current-affairs-pib-summary-9-june-2021/

RTM- REVISION THROUGH MCQS – 10th -Jun-2021 81. Consider the following statements: 1. The Election Commission of India (ECI) is an autonomous constitutional authority responsible for administering Union and State election processes in India. 2. The Constitution grants the ECI with the power of direction, superintendence, and control of elections to urban bodies. 3. The tenure of office and the conditions of service of all the Election commissioners is determined by the Parliament. Which of the given above statements is/are correct? (a) 1 only

Telegram: https://t.me/insightsIAStips 44 Youtube: https://www.youtube.com/channel/UCpoccbCX9GEIwaiIe4HLjwA

Revision Through MCQs (RTM) Compilation (June 2021)

(b) 1 and 3 only (c) 2 and 3 only (d) 1, 2 and 3 Ans: (a) Explanation: • S1: The Election Commission of India is an autonomous constitutional authority responsible for administering Union and State election processes in India. • S2: The Constitution grants the ECI with the power of direction, superintendence, and control of elections to Parliament, state legislatures, the office of president of India and the office of vice-president of India. The ECI does not deal with the elections to the urban bodies such as Municipalities and Panchayats in the states and hence, a separate State Election Commission. • S3: The President appoints the Chief Election Commissioner and other election commissioners. The tenure of office and the conditions of service of all the commissioners is determined by the President.

Refer: https://www.insightsonindia.com/2021/06/10/election-commissioner/

82. Consider the following statements: 1. In India, the minimum support price (MSP) is fixed twice a year on the recommendations of the Cabinet Committee on Economic Affairs (CCEA). 2. The Cabinet Committee on Economic Affairs (CCEA) headed by Finance Minister of India. Which of the given above statements is/are correct? (a) 1 only (b) 2 only (c) Both 1 and 2 (d) Neither 1 nor 2 Ans: (d) Explanation: • S1: The MSP is fixed twice a year on the recommendations of the Commission for Agricultural Costs and Prices (CACP). • S2: The Cabinet Committee on Economic Affairs (CCEA) headed by Prime minster.

Refer: https://www.insightsonindia.com/2021/06/10/centre-announces-hike-in-msp-for- paddy-pulses-oilseeds/

83. Consider the following: 1. Areca nut 2. Barley 3. Coffee 4. Finger millet 5. Groundnut 6. Sesamum 7. Turmeric The Cabinet Committee on Economic Affairs has announced the Minimum Support Price for which of the above? (a) 1, 2, 3 and 7 only

Telegram: https://t.me/insightsIAStips 45 Youtube: https://www.youtube.com/channel/UCpoccbCX9GEIwaiIe4HLjwA

Revision Through MCQs (RTM) Compilation (June 2021)

(b) 2, 4, 5 and 6 only (c) 1, 3, 4, 5 and 6 only (d) 1, 2, 3, 4, 5 and 7 Ans: (b) Explanation: • The Centre currently fixes MSPs for 23 farm commodities — 7 cereals (paddy, wheat, maize, bajra, jowar, ragi and barley), 5 pulses (chana, arhar/tur, urad, moong and masur), 7 oilseeds (rapeseed-mustard, groundnut, soyabean, sunflower, sesamum, safflower and nigerseed) and 4 commercial crops (cotton, sugarcane, copra and raw jute).

Refer: https://www.insightsonindia.com/2021/06/10/centre-announces-hike-in-msp-for- paddy-pulses-oilseeds/

84. With reference to Republic of Ecuador, consider the following statements: 1. It is a country in northeastern South America. 2. It shares borders with Colombia and Peru only. 3. It experiences little variation in daylight hours during the course of a year. Which of the given above statements is/are correct? (a) 1 and 2 only (b) 2 and 3 only (c) 1 and 3 only (d) 1, 2 and 3 Ans: (b) Explanation: • S1 and S2: It is a country in northwestern South America, bordered by Colombia on the north, Peru on the east and south, and the Pacific Ocean on the west. It is the westernmost country that lies on the equator. • S3: Because of its location at the equator, Ecuador experiences little variation in daylight hours during the course of a year. Both sunrise and sunset occur each day at the two six o'clock hours.

• Refer: https://www.insightsonindia.com/2021/06/10/legalisation-of-bitcoin-in-el- salvador/

85. Consider the following statements: 1. Addu Atoll is the southernmost atoll of the Maldives. 2. Maldives is situated in the . Which of the given above statements is/are correct? (a) 1 only (b) 2 only (c) Both 1 and 2

Telegram: https://t.me/insightsIAStips 46 Youtube: https://www.youtube.com/channel/UCpoccbCX9GEIwaiIe4HLjwA

Revision Through MCQs (RTM) Compilation (June 2021)

(d) Neither 1 nor 2 Ans: (c) Explanation: • S1: Addu Atoll is the southernmost atoll of the Maldives. • S2: Maldives is a small archipelagic state in South Asia situated in the Indian Ocean. • Apart from its strategic location in the Indian Ocean, Addu is the second largest city in the archipelago, home to over 30,000 people.

• Refer: Facts for Prelims: https://www.insightsonindia.com/2021/06/10/insights-daily- current-affairs-pib-summary-10-june-2021/

86. Consider the following statements about Argostemma quarantena: 1. It is a new plant species belonging to the tea family. 2. It was recently spotted in Cardamom Hills in Kerala. Which of the given above statements is/are correct? (a) 1 only (b) 2 only (c) Both 1 and 2 (d) Neither 1 nor 2 Ans: (d) Explanation: • Argostemma quarantena: • It is a new plant species belonging to the coffee family. • It was recently spotted in Wagamon hills in Kerala. • It has been named Argostemma quarantena to commemorate the millions who have died in the pandemic.

Refer: Facts for Prelims: https://www.insightsonindia.com/2021/06/10/insights-daily- current-affairs-pib-summary-10-june-2021/

87. The “Nagorno-Karabakh” conflict was in news recently, is an ethnic and territorial conflict between which of the following? (a) Israel and Palestine (b) Syria and Israel (c) China and Mongolia (d) Armenia and Azerbaijan Ans: (d) Explanation:

Telegram: https://t.me/insightsIAStips 47 Youtube: https://www.youtube.com/channel/UCpoccbCX9GEIwaiIe4HLjwA

Revision Through MCQs (RTM) Compilation (June 2021)

• Nagorno-Karabakh is part of Azerbaijan, but its population is majority Armenian. As the Soviet Union saw increasing tensions in its constituent republics in the 1980s, Nagorno-Karabakh voted to become part of Armenia – sparking a war which stopped with a ceasefire in 1994. • Since then, Nagorno-Karabakh has remained part of Azerbaijan but is controlled by separatist ethnic Armenians backed by the Armenian government.

Refer: Facts for Prelims: https://www.insightsonindia.com/2021/06/10/insights-daily- current-affairs-pib-summary-10-june-2021/

88. With reference to India’s Dehing Patkai National Park, consider the following statements: 1. It is a dipterocarp-dominated lowland rainforest. 2. It is an ideal habitat for lion-tailed macaque. 3. It is a World Heritage Site. Which of the given above statements is/are correct? (a) 1 only (b) 2 only (c) 1 and 2 only (d) 1, 2 and 3 Ans: (a) Explanation: • S1: It is located in the Dehing patkai landscape which is a dipterocarp- dominated lowland rainforest. • S2: The lion-tailed macaque or the wanderoo, is an Old World monkey endemic to the Western Ghats of South India. • S3: National Paks in Assam: Kaziranga, Manas, Nameri, Orang, Dibru- Saikhowa and Raimona National Park. Kaziranga and Manas are UNESCO World Heritage Sites. They are also tiger reserves along with Nameri and Orang.

Refer: Facts for Prelims: https://www.insightsonindia.com/2021/06/10/insights-daily- current-affairs-pib-summary-10-june-2021/

89. Consider the following statements with reference to Kaziranga National Park and Tiger Reserve: 1. It is the home of the world's most one-horned rhinos. 2. It is home to the highest density of tigers in Assam. 3. It is recognized as an Important Bird Area by International Union for Conservation of Nature (IUCN). Which of the given above statements is/are correct? (a) 1 and 2 only (b) 1 only (c) 2 and 3 only (d) 1, 2 and 3 Ans: (b) Explanation: • S1: The sanctuary, which hosts two-thirds of the world’s great one-horned rhinoceroses, is a World Heritage Site.

Telegram: https://t.me/insightsIAStips 48 Youtube: https://www.youtube.com/channel/UCpoccbCX9GEIwaiIe4HLjwA

Revision Through MCQs (RTM) Compilation (June 2021)

• S2: it is home to the highest density of tigers among protected areas in the world and was declared a Tiger Reserve in 2006 (now the highest tiger density is in Orang National Park, Assam). Also read • S3: It is recognized as an Important Bird Area by BirdLife International.

Refer: Facts for Prelims: https://www.insightsonindia.com/2021/06/10/insights-daily- current-affairs-pib-summary-10-june-2021/

90. In which one of the following States is Raimona National Park located? (a) Arunachal Pradesh (b) Manipur (c) Meghalaya (d) Assam Ans: (d) Explanation: • Raimona National Park is a national park in Assam, India, located in Gossaigaon subdivision of Kokrajhar district. • It was declared to be National Park on 5 June 2021 by the announcement of Assam's Chief Minister Himanta Biswa Sarma on the occasion of World Environment Day at Gandhi Mandap, Guwahati.

Refer: Facts for Prelims: https://www.insightsonindia.com/2021/06/10/insights-daily- current-affairs-pib-summary-10-june-2021/

RTM- REVISION THROUGH MCQS – 11th -Jun-2021

91. A new “Atlantic Charter” is sometimes mentioned in the news in the context of the affairs of the: (a) Israel and Palestine (b) European Union (EU) and United Kingdom (c) United States and the United Kingdom (d) Armenia and Azerbaijan Ans: (c) Explanation: • President Joe Biden and British Prime Minister Boris Johnson recently inspected documents related to the Atlantic Charter, a declaration signed by British Prime Minister Winston Churchill and U.S. President Franklin D. Roosevelt in August 1941. • The two leaders plan to sign what they’re calling a new Atlantic Charter, pledging to “defend the principles, values, and institutions of democracy and open societies.” o The Atlantic Charter was a joint declaration issued during World War II (1939-45) by the United States and Great Britain that set out a vision for the postwar world.

Refer: https://www.insightsonindia.com/2021/06/11/atlantic-charter/

Telegram: https://t.me/insightsIAStips 49 Youtube: https://www.youtube.com/channel/UCpoccbCX9GEIwaiIe4HLjwA

Revision Through MCQs (RTM) Compilation (June 2021)

92. Consider the following statements regarding Parliamentary privileges: 1. The parliamentary privileges do not extend to the president of India. 2. A member of the parliament cannot be arrested during the session of Parliament if he/she is booked in civil case. 3. Courts can inquire into the validity of any proceedings in Parliament on the ground of an alleged irregularity of procedure. Which of the given above statements is/are correct? (a) 1 and 2 only (b) 2 and 3 only (c) 1 and 3 only (d) 1, 2 and 3 Ans: (a) Explanation: • S1: The parliamentary privileges do not extend to the president who is also an integral part of the Parliament. • S2: Members cannot be arrested during the session of Parliament and 40 days before the beginning and 40 days after the end of a session. This privilege is available only in civil cases and not in criminal cases or preventive detention cases. • S3: The courts are prohibited to inquire into the proceedings of a House or its committees.

Refer: https://www.insightsonindia.com/2021/06/11/breach-of-privilege-motion/

93. With reference to Indian Parliament, consider the following statements: 1. In the Lok Sabha, the Speaker nominates a committee of privileges consisting of 15 members as per respective party strengths. 2. In the Rajya Sabha, the Leader of the House is the first level of scrutiny of a privilege motion. Which of the given above statements is/are correct? (a) 1 only (b) 2 only (c) Both 1 and 2 (d) Neither 1 nor 2 Ans: (a) Explanation: • S2: The Speaker/RS chairperson is the first level of scrutiny of a privilege motion. The Speaker/Chair can decide on the privilege motion himself or herself or refer it to the privileges committee of Parliament. • S1: In the Lok Sabha, the Speaker nominates a committee of privileges consisting of 15 members as per respective party strengths. A report is then presented to the House for its consideration. The Speaker may permit a half- hour debate while considering the report. The Speaker may then pass final orders or direct that the report be tabled before the House. A resolution may then be moved relating to the breach of privilege that has to be unanimously passed. In the Rajya Sabha, the deputy chairperson heads the committee of privileges that consists of 10 members.

Refer: https://www.insightsonindia.com/2021/06/11/breach-of-privilege-motion/

Telegram: https://t.me/insightsIAStips 50 Youtube: https://www.youtube.com/channel/UCpoccbCX9GEIwaiIe4HLjwA

Revision Through MCQs (RTM) Compilation (June 2021)

94. With reference to Nagaland, a state in northeastern India, consider the following statements: 1. It became a full-fledged State on December 1, 1963. 2. It is known as the land of the rising sun. 3. As per the India State of Forest Report (ISFR) 2019, the recorded forest area of the state is less than 50% of its geographical area. Which of the given above statements is/are not correct? (a) 1 only (b) 1 and 2 only (c) 2 and 3 only (d) 1, 2 and 3 Ans: (c) Explanation: here the directive word is not correct!! • S1: Nagaland became a full-fledged State on December 1, 1963. • S2: Arunachal Pradesh is known as the land of the rising sun with reference to its position as the eastern most State of India. • S3: The recorded forest area of the state is 8629.30 sq.km which is 52% of its geographical area. • The Forest Cover in the State is 12,486.40 sq km which is 75.31 % of the State's geographical area read more>> https://fsi.nic.in/isfr19/vol2/isfr-2019- vol-ii-nagaland.pdf

Refer: https://www.insightsonindia.com/2021/06/11/nagaland-to-form-panel-on-naga- issue/

95. Consider the following statements regarding Pradhan Mantri Fasal Bhima Yogna (PMFBY): 1. The Scheme covers all Food & Oilseeds crops for which past yield data is available and for which requisite number of Crop Cutting Experiments (CCEs) are conducted being under General Crop Estimation Survey (GCES). 2. The scheme is compulsory for loanee farmers availing Crop Loan /KCC account for notified crops. 3. The scheme is being administered by Ministry of Agriculture. Which of the given above statements is/are correct? (a) 1 and 3 only (b) 2 and 3 only (c) 1 and 2 only (d) 1, 2 and 3 Ans: (d) Explanation: • PMFBY provides a comprehensive insurance cover against failure of the crop thus helping in stabilising the income of the farmers. • S1: The Scheme covers all Food & Oilseeds crops and Annual Commercial/Horticultural Crops for which past yield data is available and for which requisite number of Crop Cutting Experiments (CCEs) are conducted being under General Crop Estimation Survey (GCES).

Telegram: https://t.me/insightsIAStips 51 Youtube: https://www.youtube.com/channel/UCpoccbCX9GEIwaiIe4HLjwA

Revision Through MCQs (RTM) Compilation (June 2021)

• The scheme is implemented by empanelled general insurance companies. Selection of Implementing Agency (IA) is done by the concerned State Government through bidding. • S2: The scheme is compulsory for loanee farmers availing Crop Loan /KCC account for notified crops and voluntary for other others. • S3: The scheme is being administered by Ministry of Agriculture.

Refer: https://www.insightsonindia.com/2021/06/11/what-is-the-beed-model-of-crop- insurance-maharashtra-is-pushing-for/

96. In which one of the following States is Pakhui Tiger Reserve located? (a) Arunachal Pradesh (b) Manipur (c) Meghalaya (d) Nagaland Ans: (a) Explanation: • Pakke Tiger Reserve is also known as Pakhui Tiger Reserve. • This Tiger Reserve has won India Biodiversity Award 2016 in the category of ‘Conservation of threatened species’ for its Hornbill Nest Adoption Programme. • It is bounded by Bhareli or Kameng River in the west and north, and by Pakke River in the east. • Neighbours: Papum Reserve Forest in Arunachal Pradesh, Assam’s Nameri National Park, Doimara Reserve Forest and Eaglenest Wildlife Sanctuary. • The main perennial streams in the area are the Nameri, Khari and Upper Dikorai. West of Kameng River is Sessa Orchid Sanctuary. • It falls within the Eastern Himalaya Biodiversity Hotspot.

Refer: Facts for Prelims: https://www.insightsonindia.com/2021/06/11/insights-daily- current-affairs-pib-summary-11-june-2021/

97. Consider the following statements: 1. Food Safety and Standards Authority of India is a statutory body established under the Ministry of Food Processing Industries (MoFPI). 2. FSSAI has the mandate to regulate manufacture, distribute, sell or import “organic foods” as per the provisions laid under the Environment (Protection) Act, 1986. Which of the given above statements is/are correct? (a) 1 only (b) 2 only (c) Both 1 and 2 (d) Neither 1 nor 2 Ans: (d) Explanation: • S1: Food Safety and Standards Authority of India (FSSAI) is an autonomous statutory body established under the Food Safety and Standards Act, 2006 (FSS Act). o Ministry of Health & Family Welfare, Government of India is the administrative Ministry of FSSAI. o To pursue any food related business, the owner needs to get a certificate and license with the permission of FSSAI.

Telegram: https://t.me/insightsIAStips 52 Youtube: https://www.youtube.com/channel/UCpoccbCX9GEIwaiIe4HLjwA

Revision Through MCQs (RTM) Compilation (June 2021)

• S2: Food Safety and Standards Authority of India (FSSAI) has the mandate to regulate manufacture, distribute, sell or import “organic foods” as per the provisions laid under Section 22 of the Food Safety Standards Act, 2006. Organic foods are regulated by FoodSafety and Standards (Organic Foods) Regulations, 2017 notified under the provisions of the Act.

Refer: Facts for Prelims: https://www.insightsonindia.com/2021/06/11/insights-daily- current-affairs-pib-summary-11-june-2021/

98. Consider the following statements about Gharials: 1. It is locally extinct in Pakistan, Bhutan and Myanmar. 2. It is listed in schedule II of the Wildlife (Protection) Act, 1972 3. It has been listed as critically endangered on the IUCN Red List. Which of the given above statements is/are correct? (a) 1 and 3 only (b) 2 and 3 only (c) 1 and 2 only (d) 1, 2 and 3 Ans: (a) Explanation: • S1: Historically, gharial were found in the river system of India, Pakistan, Bangladesh and southern part of Bhutan and Nepal. Today they survive only in the waters of India and Nepal. Today they survive only in the waters of India and Nepal. Read more>> • S2 and S3: Listed in Schedule I of Wildlife (Protection) Act, 1972 and as Critically Endangered on IUCN Red List

Refer: Facts for Prelims: https://www.insightsonindia.com/2021/06/11/insights-daily- current-affairs-pib-summary-11-june-2021/

99. With reference to communication technologies, what is/are the difference / differences between LTE (Long-Term Evolution) and VoLTE (Voice over Long-Term Evolution)? 1. LTE is commonly marketed as 3G and VoLTE is commonly marketed as advanced 3G. 2. LTE is data-only technology and VoLTE is voice-only technology. Select the correct answer using the code given below. (a) 1 only (b) 2 only (c) Both 1 and 2 (d) Neither 1 nor 2 Ans: (d) Explanation: • VoLTE is voice calls over a 4G LTE network, rather than the 2G or 3G connections. VolTE can transmit data too. VoLTE has up to three times more voice and data capacity than 3G UMTS and up to six times more than 2G GSM. So, both 1 and 2 are wrong. • Cabinet approves 5 MHz spectrum for Railways: o The Union Cabinet has approved the allotment of 5 Mhz spectrum in the premium 700 MHz band to the Railways for captive use in areas of public safety and security services.

Telegram: https://t.me/insightsIAStips 53 Youtube: https://www.youtube.com/channel/UCpoccbCX9GEIwaiIe4HLjwA

Revision Through MCQs (RTM) Compilation (June 2021)

Refer: Facts for Prelims: https://www.insightsonindia.com/2021/06/11/insights-daily- current-affairs-pib-summary-11-june-2021/

100. In which one of the following States is Khonoma Nature Conservation and Tragopan Sanctuary located? (a) Arunachal Pradesh (b) Manipur (c) Meghalaya (d) Nagaland Ans: (d) Explanation: • Khonoma Nature Conservation & Tragopan Sanctuary wins India Biodiversity Awards 2021 • KNCTS in Nagaland won the award under the ‘Sustainable Use of Biological Resources’ category. KNCTS was established on December 11, 1998.

Refer: https://morungexpress.com/khonoma-nature-conservation-tragopan-sanctuary- wins-india-biodiversity-awards-2021

Telegram: https://t.me/insightsIAStips 54 Youtube: https://www.youtube.com/channel/UCpoccbCX9GEIwaiIe4HLjwA

Revision Through MCQs (RTM) Compilation (June 2021)

RTM- REVISION THROUGH MCQS – 12th -Jun-2021

101. With reference to One Nation One Ration Card Scheme, which of the following statements are correct? 1. Under the initiative, eligible beneficiaries would be able to avail their entitled foodgrains under the National Food Security Act (NFSA) from any Fair Price Shop in state across India using the same ration card. 2. For national portability, the state governments have been asked to issue the ration card in bi-lingual format. 3. The integrated Aadhaar Enabled Payment System (AePS) provides the technological platform for the inter-state portability of ration cards. Select the correct answer using the given code below: (a) 1 and 2 only (b) 2 and 3 only (c) 1 and 3 only (d) 1, 2 and 3 Ans: (a) Explanation: • S3: The Integrated Management of Public Distribution System (IM-PDS) portal provides the technological platform for the inter-state portability of ration cards. • S2: For national portability, the state governments have been asked to issue the ration card in bi-lingual format, wherein besides the local langauge, the other language could be Hindi or English. • S1: Under the initiative, eligible beneficiaries would be able to avail their entitled foodgrains under the National Food Security Act (NFSA) from any Fair Price Shop in the country using the same ration card.

Refer: https://www.insightsonindia.com/2021/06/12/one-nation-one-ration-card-scheme- 6/

102. Consider the following statements regarding the International Court of Justice (ICJ): 1. It is one of the six principal organs of the United Nations (UN). 2. It settles disputes between states in accordance with international law and gives advisory opinions on international legal issues. 3. The seat of the Court is at the Peace Palace in The Hague (Netherlands). Select the correct answer using the given code below: (a) 1 and 2 only (b) 2 and 3 only (c) 1 and 3 only (d) 1, 2 and 3 Ans: (d) Explanation: • The International Court of Justice (ICJ) is the principal judicial organ of the United Nations (UN). It was established in June 1945 by the Charter of the United Nations and began work in April 1946.

Telegram: https://t.me/insightsIAStips 55 Youtube: https://www.youtube.com/channel/UCpoccbCX9GEIwaiIe4HLjwA

Revision Through MCQs (RTM) Compilation (June 2021)

• The seat of the Court is at the Peace Palace in The Hague (Netherlands). Of the six principal organs of the United Nations, it is the only one not located in New York (United States of America). • The Court’s role is to settle, in accordance with international law, legal disputes submitted to it by States and to give advisory opinions on legal questions referred to it by authorized United Nations organs and specialized agencies. • The Court is composed of 15 judges, who are elected for terms of office of nine years by the United Nations General Assembly and the Security Council. It is assisted by a Registry, its administrative organ. Its official languages are English and French.

Refer: https://www.insightsonindia.com/2021/06/12/pak-passes-bill-to-let-jadhav- appeal/

103. Consider the following statements: According to the new Vehicle Scrappage Policy, 1. Commercial vehicles of more than 15 years will have to be mandatorily scrapped if they don’t pass the fitness and emission tests. 2. Passenger vehicles of more than 20 years will have to be voluntarily scrapped if they don’t pass the fitness and emission tests. 3. Government departments will have to let go of their vehicles after they are used for 15 years. Which of the given above statements is/are correct? (a) 1 and 2 only (b) 3 only (c) 1 and 3 only (d) 1, 2 and 3 Ans: (c) Explanation: • According to the new policy, commercial vehicles of more than 15 years and passenger vehicles of more than 20 years will have to be mandatorily scrapped if they don’t pass the fitness and emission tests. Also, government departments will have to let go of their vehicles after they are used for 15 years.

Refer: https://www.insightsonindia.com/2021/06/12/vehicle-scrappage-policy-2/

104. Consider the following statements: 1. EnVision is a European Space Agency (ESA) led mission with contributions from ISRO. 2. NASA recently selected ‘DAVINCI+’ and ‘VERITAS’ missions to the planet Venus. Which of the given above statements is/are correct? (a) 1 only (b) 2 only (c) Both 1 and 2 (d) Neither 1 nor 2 Ans: (b) Explanation: • What is EnVision? o EnVision is an ESA led mission with contributions from NASA.

Telegram: https://t.me/insightsIAStips 56 Youtube: https://www.youtube.com/channel/UCpoccbCX9GEIwaiIe4HLjwA

Revision Through MCQs (RTM) Compilation (June 2021)

o It will be launched on an Ariane 6 rocket, the spacecraft will take about 15 months to reach Venus and will take 16 more months to achieve orbit circularisation. o Objectives: To study the planet’s atmosphere and surface, monitor trace gases in the atmosphere and analyse its surface composition. A radar provided by NASA will help to image and map the surface. • Other missions to Venus: o NASA recently selected two missions to the planet Venus- DAVINCI+ and VERITAS. o Previously, an ESA-led mission to Venus called ‘Venus Express’ (2005- 2014) focussed on atmospheric research and pointed to volcanic hotspots on the planet’s surface. o The first spacecraft to Venus was the Soviet Union’s Venera series, followed by NASA’s Magellan Mission that studied Venus from 1990- 1994. o As of now, Japan’s Akatsuki mission is studying the planet from Orbit.

Refer: https://www.insightsonindia.com/2021/06/12/european-space-agencys-envision- mission-to-venus/

105. Consider the following statements about planet Venus: 1. It is the second closest planet to the Sun. 2. It is called the Earth’s twin because of their similar atmosphere. 3. It does not have a moon and rings. Which of the given above statements is/are correct? (a) 1 and 2 only (b) 2 and 3 only (c) 1 and 3 only (d) 1, 2 and 3 Ans: (c) Explanation: • S1: It is the second closest planet to the Sun. • S2: It is called the Earth’s twin because of their similar sizes. o Venus’ thick atmosphere traps heat and is the reason that it is the hottest planet in the solar system, despite coming after Mercury, the closest planet to the Sun. Surface temperatures on Venus can go up to 471 degrees Celsius, which is hot enough to melt lead. • S3: Venus also does not have a moon and no rings.

Refer: https://www.insightsonindia.com/2021/06/12/european-space-agencys-envision- mission-to-venus/

106. Project HIMANK is a project of which of the following? (a) Indian Defense Service of Engineers (IDSE) (b) Military Officers (Corps of Engineers) (c) Border Roads Organisation (BRO) (d) All of the above Ans: (c) Explanation:

Telegram: https://t.me/insightsIAStips 57 Youtube: https://www.youtube.com/channel/UCpoccbCX9GEIwaiIe4HLjwA

Revision Through MCQs (RTM) Compilation (June 2021)

• Project HIMANK, is a project of the Border Roads Organisation (BRO) being implemented in the Ladakh region of Jammu & Kashmir. This project started in 1985. Under this project, BRO is responsible for the construction and maintenance of roads and related infrastructure including the world's highest motorable roads across the Khardung La, Tanglang La and Chang La passes. • Umling La: o At 5.793m (19,005ft) above the sea level, Umling La is one of the highest motorable roads in the world. o Located in the Ladakh region of Jammu and Kashmir. o The 54 km long road passing through Umling La Pass connected the villages of Chisumle and Demchok and took six years to be built by BRO under Project Himank.

Refer: Facts for Prelims: https://www.insightsonindia.com/2021/06/12/insights-daily- current-affairs-pib-summary-12-june-2021/

107. Consider the following statements: 1. The National Security Guard (NSG) is a counter-terrorism unit under the Indian Ministry of Home Affairs (MHA). 2. It was set up in 1985 after the assassination of Prime Minister Indira Gandhi. Which of the given above statements is/are correct? (a) 1 only (b) 2 only (c) Both 1 and 2 (d) Neither 1 nor 2 Ans: (a) Explanation: • S1: The NSG is a counter-terrorism unit that formally came into existence in 1986 by an act of Parliament- ‘National Security Guard Act, 1986’. It operates under the Ministry of Home Affairs and is a task-oriented force. • S2: The Special Protection Group (SPG) was started in 1985 in the wake of the killing of PM Indira Gandhi in 1984.

Refer: Facts for Prelims: https://www.insightsonindia.com/2021/06/12/insights-daily- current-affairs-pib-summary-12-june-2021/

108. Kamlang wildlife sanctuary is located in (a) Assam (b) Arunachal Pradesh (c) Nagaland (d) Manipur Ans: (b) Explanation: • The Kamlang Wildlife Sanctuary is situated in the Lohit District of Arunachal Pradesh. The park is named after the Kamlang River which flows through it.

Refer: http://arunachalforests.gov.in/kamlang_wildlife_sanctuary.html

109. “Caspian Sea” is bordered by which one of the following groups of nations? (a) Russia, Kazakhstan, Turkmenistan and Iran (b) Russia, Kazakhstan, Afghanistan and Azerbaijan

Telegram: https://t.me/insightsIAStips 58 Youtube: https://www.youtube.com/channel/UCpoccbCX9GEIwaiIe4HLjwA

Revision Through MCQs (RTM) Compilation (June 2021)

(c) Russia, Kazakhstan, Uzbekistan and Iran (d) Russia, Kazakhstan, Afghanistan and Iran Ans: (a) Explanation: • Caspian Sea is bordered by following countries:

• Refer: World physical map

110. In which of the following states/UTs you are most likely to find Tropical evergreen forest, semi evergreen, deciduous, Pine and Temperate forests? (a) Arunachal Pradesh (b) Kerala (c) Andaman and Nicobar (d) Karnataka Ans: (a) Explanation: • Situated on the northeastern tip of the country, the state of Arunachal Pradesh is a part of Eastern Himalayan Ranges. The important forests types found in the state are Tropical evergreen, semi evergreen, deciduous, Pine, Temperate, Alpine and grassland.

Refer: India physical map

RTM- REVISION THROUGH MCQS – 14th -Jun-2021

111. Consider the following statements regarding Citizenship (Amendment) Act, 2019: 1. It exempts certain areas in the North-East from this provision. 2. According to the Citizenship Act, a foreigner may register as an Overseas Citizens of India (OCI) if they are of Indian origin or their spouse is of Indian origin. Which of the given above statements is/are correct? (a) 1 only (b) 2 only (c) Both 1 and 2 (d) Neither 1 nor 2 Ans: (c) Explanation: • S1: it exempts certain areas in the North-East from this provision. It would not apply to tribal areas of Assam, Meghalaya, Mizoram and Tripura. This effectively means that Arunachal Pradesh, Nagaland and Mizoram along with

Telegram: https://t.me/insightsIAStips 59 Youtube: https://www.youtube.com/channel/UCpoccbCX9GEIwaiIe4HLjwA

Revision Through MCQs (RTM) Compilation (June 2021)

almost whole of Meghalaya and parts of Assam and Tripura would stay out of the purview of the Citizenship (Amendment) Act. • S2: According to the Citizenship Act, a foreigner may register as an Overseas Citizens of India (OCI) if they are of Indian origin or their spouse is of Indian origin. The Citizenship (Amendment) Act entitles the OCI cardholders to benefits such as the right to travel to India, and to work and study in the country.

Refer: https://www.insightsonindia.com/2021/06/14/union-home-ministry-order-invites- citizenship/

112. Consider the following statements: 1. The Group of 7 (G7) is an informal group of seven countries including Japan and Italy. 2. The Group of 7 (G7) Secretariat is based in Paris, France. Which of the given above statements is/are correct? (a) 1 only (b) 2 only (c) Both 1 and 2 (d) Neither 1 nor 2 Ans: (a) Explanation: • S1: The Group of 7 (G7) is an informal group of seven countries — the United States, Canada, France, Germany, Italy, Japan and the United Kingdom, the heads of which hold an annual summit with European Union and other invitees. • S2: The G7 is not based on a treaty and has no permanent secretariat or office; its presidency rotates annually among the members, with the presiding nation setting the group's priorities, and hosting and organizing its summit.

Refer: https://www.insightsonindia.com/2021/06/14/g7-meet/

113. Arrange the following locations in the direction of North to South: 1. Siachen glacier 2. Nubra valley 3. Khardung la Select the correct answer using the code below: (a) 1-2-3 (b) 1-3-2 (c) 2-1-3 (d) 2-3-1 Ans: (a) Explanation:

• Refer: https://www.insightsonindia.com/2021/06/14/a-year-after-galwan-clash/

Telegram: https://t.me/insightsIAStips 60 Youtube: https://www.youtube.com/channel/UCpoccbCX9GEIwaiIe4HLjwA

Revision Through MCQs (RTM) Compilation (June 2021)

114. Consider the following statements regarding Rare Earth elements. 1. Rare Earth elements are not radioactive in nature. 2. India has the world's largest rare earth elements reserves. Which of the given above statements is/are correct? (a) 1 only (b) 2 only (c) Both 1 and 2 (d) Neither 1 nor 2 Ans: (d) Explanation: • S1: Despite their classification, most of these elements are not really “rare”. One of the Rare Earths, promethium, is radioactive. • S2: China’s Rare Earths deposits account for 80% of identified global reserves.

Refer: https://www.insightsonindia.com/2021/06/14/rare-earth-metals-at-the-heart-of- china-us-rivalry/

115. Recently, our scientists have discovered a 15-meter-tall tree that belongs to the genus of the coffee family which is distinguished by a long stem with a whitish coating on the trunk and oblong-obovate leaves with a cuneate base. In which part of India has it been discovered? (a) Andaman Islands (b) Western Ghats (c) Araku Valley (d) Tropical rain forests of northeast Ans: (a) Explanation: Tree of coffee family discovered in Andaman and Nicobar • A 15 metre tall tree that belongs to genus of the coffee family has recently been discovered in Andaman Islands • The new species Pyrostria laljii, is also the first record of the genus Pyrostria in India. Trees belonging to these species are usually found in Madagascar • The tree is distinguished by a long stem with a whitish coating on the trunk and oblong-obovate leaves with a cuneate base and was first reported from Wandoor forest in South Andaman • Other places where tree could be located are: Jarawa reserve forest, chidia tapu forest • Pyrostria laljii: Critically endangered as per IUCN status

Refer: Facts for Prelims: https://www.insightsonindia.com/2021/06/14/insights-daily- current-affairs-pib-summary-14-june-2021/

116. What is “New Shepherd”, sometimes seen in the news? (a) A first-generation Chinese aircraft carrier (b) An American anti-missile system (c) A launch vehicle developed by SpaceX (d) None of the above Ans: (d) Explanation:

Telegram: https://t.me/insightsIAStips 61 Youtube: https://www.youtube.com/channel/UCpoccbCX9GEIwaiIe4HLjwA

Revision Through MCQs (RTM) Compilation (June 2021)

• New Shepard is a vertical-takeoff, vertical-landing, crew-rated suborbital launch vehicle that is being developed by Blue Origin as a commercial system for suborbital space tourism.

Refer: Facts for Prelims: https://www.insightsonindia.com/2021/06/14/insights-daily- current-affairs-pib-summary-14-june-2021/

117. Which of the following temples is/are famous for Lord Shiva? 1. Srisailam temple 2. Ramanathaswamy temple 3. Ghrishneshwara temple Select the correct answer using the code given below: (a) 1 only (b) 2 and 3 only (c) 1 and 3 only (d) 1, 2 and 3 Ans: (d) Explanation: • Copperplate inscriptions found at Srisailam temple o The Bhramarambha Mallikarjuna Devasthanam, engaged in reviving ancient Ganta Matham near Srisailam temple complex, chanced upon six sets of copperplate inscriptions on Sunday. o There were 18 copper leaves found that could be dated back to somewhere between 14 and 16 centuries. o The inscriptions depict the far and wide patronage by people. o Out of the six sets, four have inscriptions in Sanskrit and Nandi-Nāgarī script and the other two plates are in Telugu script, he added. • A Jyotirlinga or Jyotirlingam, is a devotional representation of the Hindu god Shiva. • The twelve jyotirlingas are: o Somnath in Gir Somnath, Gujarat o Mallikarjuna in Srisailam, Andhra Pradesh o Mahakaleshwar in Ujjain, Madhya Pradesh o Omkareshwar in Khandwa, Madhya Pradesh o Kedarnath in Rudraprayag, Uttarakhand o Bhimashankar in Pune, Maharashtra o Kashi Vishwanath in Varanasi, Uttar Pradesh o Trayambakeshwar in Nashik, Maharashtra o Vaidyanath in Deoghar, Jharkhand o Nageshwara in Dwarka, Gujarat o Ramanathaswamy (Rameshwar) in Rameswaram, Tamil Nadu o Ghrishneshwara in Aurangabad, Maharashtra

Refer: Facts for Prelims: https://www.insightsonindia.com/2021/06/14/insights-daily- current-affairs-pib-summary-14-june-2021/

118. Which of the following is geographically closest to Madagascar? (a) Comoros (b) Mauritius (c) Seychelles

Telegram: https://t.me/insightsIAStips 62 Youtube: https://www.youtube.com/channel/UCpoccbCX9GEIwaiIe4HLjwA

Revision Through MCQs (RTM) Compilation (June 2021)

(d) Maldives Ans: (a) Explanation: • Self-explanatory

• Refer: World physical map

119. Consider the following pairs: City Country 1. Kenya 2. Zanzibar Tanzania 3. Mombasa Which of the given above pairs is/are correctly matched? (a) 1 and 2 only (b) 2 only (c) 1 and 3 only (d) 2 and 3 only Ans: (b) Explanation: • Mogadishu:: Somalia • Zanzibar:: Tanzania • Mombasa:: Kenya

• Refer: World physical map

120. Consider the following statements: 1. The Nuclear Security Summits are periodically held under the aegis of the United Nations. 2. The International Panel on Fissile Materials is an organ of International Atomic Energy Agency. Which of the statements given above is/are correct? (a) 1 only

Telegram: https://t.me/insightsIAStips 63 Youtube: https://www.youtube.com/channel/UCpoccbCX9GEIwaiIe4HLjwA

Revision Through MCQs (RTM) Compilation (June 2021)

(b) 2 only (c) Both 1 and 2 (d) Neither 1 nor 2 Ans: (d) Explanation: • Statement 1: It is wrong. o In his 2009 speech, Obama (Former US President) stated that nuclear terrorism “is the most immediate and extreme threat to global security.” o To mitigate this threat, he urged that “we act with purpose and without delay,” announcing “a new international effort to secure vulnerable nuclear material around the world” that would begin with “a Global Summit on Nuclear Security that the United States will host.” o http://www.nss2016.org/about-nss/history/ • Statement 2: This is also wrong. o Established in 2006, it is a group of independent nuclear experts from 18 countries: Brazil, Canada, China, France, Germany, India, Iran, Japan, the Netherlands, Mexico, Norway, Pakistan, South Korea, Russia, South Africa, Sweden, the United Kingdom, and the United States. o It aims to advance international initiatives to “secure and to sharply reduce all stocks of highly enriched uranium and separated plutonium, the key materials in nuclear weapons, and to limit any further production” o www.fissilematerials.org/ipfm/about.html

Refer: UPSC CSE 2017

RTM- REVISION THROUGH MCQS – 15th -Jun-2021

121. Consider the following statements: 1. Tulu is listed in the Eighth Schedule of the Constitution through 95th Constitutional Amendment Act. 2. Tulu-speaking people are larger in number than speakers of Manipuri and Sanskrit. Which of the given above statements is/are correct? (a) 1 only (b) 2 only (c) Both 1 and 2 (d) Neither 1 nor 2 Ans: (b) Explanation: • S2: The Census (2011) reports 18,46,427 native speakers of Tulu in India. The Tulu-speaking people are larger in number than speakers of Manipuri and Sanskrit, which have the Eighth Schedule status. • S1: The 22 languages which are listed in the Eighth Schedule are Assamese, Bengali, Bodo, Dogri, Gujarati, Hindi, Kannada, Kashmiri, Konkani, Maithili, Malayalam, Manipuri, Marathi, Nepali, Oriya, Punjabi, Sanskrit, Santali, Sindhi, Tamil, Telugu and Urdu. • Tulu and English are not listed in Eighth Schedule of the Indian Constitution.

Telegram: https://t.me/insightsIAStips 64 Youtube: https://www.youtube.com/channel/UCpoccbCX9GEIwaiIe4HLjwA

Revision Through MCQs (RTM) Compilation (June 2021)

Refer: https://www.insightsonindia.com/2021/06/15/the-history-of-tulu-and-the- demand-for-official-language-status/

122. With reference to the provisions made under the National Food Security Act, 2013, consider the following statements: 1. The families coming under the category of ‘below poverty line (BPL)’ only are eligible to receive subsidies food grains. 2. The eldest woman in a household, of age 18 years or above, shall be the head of the household for the purpose of issuance of a ration card. 3. Pregnant women and lactating mothers are entitled to a ‘take-home ration’ of 1600 calories per day during pregnancy and for six months thereafter Which of the statement given above is/are correct? (a) 1 and 2 only (b) 2 only (c) 1 and 3 only (d) 2 and 3 only Ans: (b) Explanation: • S1: Coverage and entitlement under Targeted Public Distribution System (TPDS): Upto 75% of the rural population and 50% of the urban population will be covered under TPDS, with uniform entitlement of 5 kg per person per month. Not restricted to only BPL families. • S3: Nutritional Support to women and children: Children in the age group of 6 months to 14 years and pregnant women and lactating mothers will be entitled to meals as per prescribed nutritional norms under Integrated Child Development Services (ICDS) and Mid-Day Meal (MDM) schemes. Malnourished children up to the age of 6 have been prescribed for higher nutritional norms. o Maternity Benefit: Pregnant women and lactating mothers will also be receiving maternity benefit of Rs. 6,000. • S2: As a step towards women empowerment, the eldest woman of the household of age 18 years or above is mandated to be the head of the household for the purpose of issuing of ration cards under the Act.

Refer: https://www.insightsonindia.com/2021/06/15/national-food-security-act-nfsa/

123. Consider the following statements about Food Corporation of India. 1. It is a statutory body created and run by the Government of India. 2. It is under the jurisdiction of Ministry of Agriculture & Farmers Welfare. Which of the given above statements is/are correct? (a) 1 only (b) 2 only (c) Both 1 and 2 (d) Neither 1 nor 2 Ans: (a) Explanation: • S1: The Food Corporation of India was setup under the Food Corporation's Act 1964 , in order to fulfill following objectives of the Food Policy:

Telegram: https://t.me/insightsIAStips 65 Youtube: https://www.youtube.com/channel/UCpoccbCX9GEIwaiIe4HLjwA

Revision Through MCQs (RTM) Compilation (June 2021)

o Effective price support operations for safeguarding the interests of the farmers. o Distribution of foodgrains throughout the country for public distribution system. o Maintaining satisfactory level of operational and buffer stocks of foodgrains to ensure National Food Security • S2: It is under the jurisdiction of Ministry of Consumer Affairs, Food and Public Distribution, Government of India.

Refer: https://www.insightsonindia.com/2021/06/15/national-food-security-act-nfsa/

124. Consider the following statements about North Atlantic Treaty Organization: 1. It is an intergovernmental military alliance. 2. Only Canada, France, Italy, the United Kingdom and the United States are the original founding members of the NATO. Which of the given above statements is/are correct? (a) 1 only (b) 2 only (c) Both 1 and 2 (d) Neither 1 nor 2 Ans: (a) Explanation: • S1: About North Atlantic Treaty Organization: o It is an intergovernmental military alliance. o Established by Washington treaty. o Treaty that was signed on 4 April 1949. o Headquarters — Brussels, Belgium. o Headquarters of Allied Command Operations — Mons, Belgium. • S2: Its original members were Belgium, Canada, Denmark, France, Iceland, Italy, Luxembourg, the Netherlands, Norway, Portugal, the United Kingdom, and the United States.

Refer: https://www.insightsonindia.com/2021/06/15/nato-summit/

125. Consider the following statements: 1. Russia and the China together possessed over 90% of global nuclear weapons. 2. India has more nuclear warheads than Pakistan. Which of the given above statements is/are correct? (a) 1 only (b) 2 only (c) Both 1 and 2 (d) Neither 1 nor 2 Ans: (d) Explanation: Swedish think tank Stockholm International Peace Research Institute (SIPRI) has released its Year Book 2021. • Key findings: o S2: India possessed an estimated 156 nuclear warheads at the start of 2021, compared with 150 at the start of last year, while Pakistan had 165 warheads, up from 160 in 2020.

Telegram: https://t.me/insightsIAStips 66 Youtube: https://www.youtube.com/channel/UCpoccbCX9GEIwaiIe4HLjwA

Revision Through MCQs (RTM) Compilation (June 2021)

o China’s nuclear arsenal consisted of 350 warheads, up from 320 at the start of 2020. o The nine nuclear armed states — the U.S., Russia, the U.K., France, China, India, Pakistan, Israel and North Korea — together possessed an estimated 13,080 nuclear weapons at the start of 2021. o S1: Russia and the U.S. together possessed over 90% of global nuclear weapons.

Refer: https://www.insightsonindia.com/2021/06/15/stockholm-international-peace- research-institute-sipri-year-book-2021/

126. Consider the following pairs: The rocket system designed/built by 1. New Shephard Amazon 2. Stardust 1.0 bluShift 3. Falcon 9 SpaceX Which of the given above pairs is/are correctly matched? (a) 1 and 3 only (b) 3 only (c) 2 and 3 only (d) 2 only Ans: (c) Explanation: • S1: New Shepard is a crew-rated suborbital launch vehicle that is being developed by Blue Origin as a commercial system for suborbital space tourism. • S2: Stardust 1.0 is manufactured by bluShift, an aerospace company based in Maine that is developing rockets that are powered by bio-derived fuels. • S3: Falcon 9 is a partially reusable two-stage-to-orbit medium-lift launch vehicle designed and manufactured by SpaceX in the United States.

Refer: https://www.insightsonindia.com/2021/06/15/what-is-the-new-shephard-rocket- system/

127. India enacted The Geographical Indications of Goods (Registration and Protection) Act, 1999 in order to comply with the obligations (a) ILO (b) IMF (c) UNCTAD (d) WTO Ans: (d) Explanation: • Article 22 of the TRIPS Agreements (WTO Agreement) defines Geographical Indication as “indications which identify a good as originating in the territory of a Member, or a region or locality in that territory, where a given quality, reputation or other characteristic of the good is essentially attributable to its geographical origin”. • Consequently, upon India joining as a member state of the TRIPS Agreement sui- geneis legislation for the protection of geographical indication was enacted in 1999. Geographical Indications are covered as an element of Intellectual Property Rights under the TRIPS Agreement (WTO Agreement).

Telegram: https://t.me/insightsIAStips 67 Youtube: https://www.youtube.com/channel/UCpoccbCX9GEIwaiIe4HLjwA

Revision Through MCQs (RTM) Compilation (June 2021)

Refer: https://www.insightsonindia.com/2021/06/15/india-and-pakistans-battle-over- basmati/

128. Consider the following statements: 1. National Securities Depository Limited (NSDL) is the regulatory body for securities and commodity market in India. 2. It is under the jurisdiction of Ministry of Finance, Government of India. Which of the given above statements is/are correct? (a) 1 only (b) 2 only (c) Both 1 and 2 (d) Neither 1 nor 2 Ans: (b) Explanation: • S1: The Securities and Exchange Board of India (SEBI) is the regulatory body for securities and commodity market in India. • S2: National Securities Depository Limited (NSDL) is an Indian central securities depository under the jurisdiction of Ministry of Finance. • National Securities Depository Limited: o The enactment of Depositories Act in August 1996 paved the way for establishment of NSDL in August 1996. o It handles most of the securities held and settled in dematerialized form in the Indian capital market. o NSDL works to support the investors and brokers in the capital market of the country. o It aims at ensuring the safety and soundness of Indian marketplaces by developing settlement solutions that increase efficiency, minimize risk and reduce costs.

Refer: facts for prelims: https://www.insightsonindia.com/2021/06/15/insights-daily- current-affairs-pib-summary-15-june-2021/

129. It is a unique Mango from Bhagalpur district of Bihar and it is known for its light yellow skin and special aroma. Also it received the GI tag in 2018. It is (a) Alphonso Mango (b) Chausa Mango (c) Badami Mango (d) Zardalu Mango Ans: (d) Explanation: Jardalu Mango: • Jardalu or Zardalu mango is a unique variety of mango grown in Bhagalpur and adjoining districts of Bihar. • It received the GI tag in 2018.

• Refer: facts for prelims: https://www.insightsonindia.com/2021/06/15/insights-daily- current-affairs-pib-summary-15-june-2021/

Telegram: https://t.me/insightsIAStips 68 Youtube: https://www.youtube.com/channel/UCpoccbCX9GEIwaiIe4HLjwA

Revision Through MCQs (RTM) Compilation (June 2021)

130. Consider the following pairs: Places in news Country 1. Stockholm Canada 2. Brussels Belgium 3. Carbis Bay United Kingdom Which of the given above pairs is/are correctly matched? (a) 1 and 3 only (b) 3 only (c) 2 and 3 only (d) 2 only Ans: (c) Explanation: • S1: Stockholm International Peace Research Institute (SIPRI) is an international institute based in Stockholm, Sweden. • S2: 2021 NATO Summit Held in Brussels, Belgium. • S3: The 47th G7 summit was held on 11–13 June 2021 in Cornwall in the United Kingdom. o Carbis Bay is a village in Cornwall, UK.

Refer: facts for prelims: https://www.insightsonindia.com/2021/06/15/insights-daily- current-affairs-pib-summary-15-june-2021/

RTM- REVISION THROUGH MCQS – 16th -Jun-2021

131. Consider the following statements about Ram Prasad Bismil: 1. He was associated with the Arya Samaj from an early age. 2. He participated in the Muzaffarpur Conspiracy of 1908. 3. He played a crucial role in Ghadar Movement. Which of the given above statements is/are correct? (a) 1 and 2 only (b) 1 only (c) 2 and 3 only (d) 1, 2 and 3 Ans: (b) Explanation: • S2: Events he was associated with: o He participated in the Mainpuri conspiracy of 1918. o On August 9, 1925, Ram Prasad Bismil along with companions Ashfaqulla Khan and others executed the plan of looting the train at Kakori near Lucknow. • S1: He was associated with the Arya Samaj from an early age.

Refer: https://www.insightsonindia.com/2021/06/16/ram-prasad-bismil/

132. Consider the following statements regarding the Arya Samaj: 1. It is a Hindu revivalist movements. 2. It was founded by Dayanand Saraswati in Lahore, Punjab. 3. It believes in the infallible authority of the Vedas.

Telegram: https://t.me/insightsIAStips 69 Youtube: https://www.youtube.com/channel/UCpoccbCX9GEIwaiIe4HLjwA

Revision Through MCQs (RTM) Compilation (June 2021)

Which of the given above statements is/are correct? (a) 1 and 2 only (b) 3 only (c) 2 and 3 only (d) 1, 2 and 3 Ans: (b) Explanation: • S1 and S2: It is a Hindu reform movement that was founded by Dayanand Saraswati in 1875 in Bombay. • S3: The movement believes in the infallible authority of the Vedas. o The central objectives of Arya Samaj is to, “eradicate Ignorance (Agyan), Indigence or Poverty (Abhav) and Injustice (Anayay) from this earth. o Members of the Arya Samaj believe in one God and reject the worship of idols.

Refer: https://www.insightsonindia.com/2021/06/16/ram-prasad-bismil/

133. Consider the following statements about the United Nations Convention on the Law of the Sea (UNCLOS): 1. It was developed by the International Maritime Organization (IMO). 2. It is the globally recognized regime dealing with all matters relating to the law of the sea. Which of the given above statements is/are correct? (a) 1 only (b) 2 only (c) Both 1 and 2 (d) Neither 1 nor 2 Ans: (b) Explanation: • S1: UNCLOS is an international agreement that resulted from the third United Nations Conference on the Law of the Sea (UNCLOS III), which took place between 1973 and 1982. • S2: The Law of the Sea Convention defines the rights and responsibilities of nations with respect to their use of the world's oceans, establishing guidelines for businesses, the environment, and the management of marine natural resources.

Refer: https://www.insightsonindia.com/2021/06/16/supreme-court-closes-criminal-case- against-italian-marines/

134. The ‘Enrica Lexie Case’ was in news recently, is related to which of the following? (a) International patent system (b) Minimum global corporation tax (c) International drugs syndicate (d) None of the above Ans: (d) Explanation: • The Enrica Lexie case is an ongoing international controversy about a shooting off the western coast of India.

Telegram: https://t.me/insightsIAStips 70 Youtube: https://www.youtube.com/channel/UCpoccbCX9GEIwaiIe4HLjwA

Revision Through MCQs (RTM) Compilation (June 2021)

• Refer: https://www.insightsonindia.com/2021/06/16/supreme-court-closes-criminal-case- against-italian-marines/

135. Consider the following statements about the Food and Agriculture Organization (FAO): 1. It is a specialized agency of the United Nations. 2. It is the only multilateral development organization that focuses solely on rural economies and food security. 3. It is headquartered in Rome, Italy. Which of the given above statements is/are correct? (a) 1 and 2 only (b) 2 and 3 only (c) 1 and 3 only (d) 1, 2 and 3 Ans: (c) Explanation: • S1: the Food and Agriculture Organization (FAO) is a specialized agency of the United Nations that leads international efforts to defeat hunger and improve nutrition and food security. • S2: International Fund for Agricultural Development is the only multilateral development organization that focuses solely on rural economies and food security. • S3: FAO is headquartered in Rome, Italy.

Refer: https://www.insightsonindia.com/2021/06/16/food-and-agriculture-organizations- fao-conference/

136. Which one of the following is not published/released by the Food and Agriculture Organization (FAO)? (a) The State of the World's Forests

Telegram: https://t.me/insightsIAStips 71 Youtube: https://www.youtube.com/channel/UCpoccbCX9GEIwaiIe4HLjwA

Revision Through MCQs (RTM) Compilation (June 2021)

(b) The State of Agricultural Commodity Markets (c) The Global Forest Resources Assessment (d) All the above are published by FAO Ans: (d) Explanation: Important reports of FAO: • The State of the World's Forests • The State of Agricultural Commodity Markets • The Global Forest Resources Assessment

Refer: https://www.insightsonindia.com/2021/06/16/food-and-agriculture-organizations- fao-conference/

137. Consider the following statements about ‘Polar-Areas Stellar-Imaging in Polarisation High-Accuracy Experiment’ (PASIPHAE): 1. It is the largest ever collaborative sky surveying project in the history of science. 2. It is spearheaded by the Massachusetts Institute of Technology, USA. Which of the given above statements is/are correct? (a) 1 only (b) 2 only (c) Both 1 and 2 (d) Nether 1 nor 2 Ans: (d) Explanation: • S1: Polar-Areas Stellar-Imaging in Polarisation High-Accuracy Experiment (PASIPHAE) is an international collaborative sky surveying project. o Scientists aim to study the polarisation in the light coming from millions of stars. • S2: PASIPHAE is an international collaboration between the Crete Astrophysics Group, Caltech in the US, IUCAA in India, the South African Astronomical Observatory, and the University of Oslo in Norway.

Refer: https://www.insightsonindia.com/2021/06/16/what-is-pasiphae-and-why-is-it- important/

138. Consider the following statements regarding the United Nations Convention to Combat Desertification (UNCCD): 1. It is the only convention stemming from a direct recommendation of the Rio Conference’s Agenda 21. 2. It is the sole legally binding international agreement linking environment and development to sustainable land management. 3. Its Secretariat is administered by UNFCCC and is located at Geneva, Switzerland. Which of the given above statements is/are correct? (a) 1 and 2 only (b) 2 and 3 only (c) 1 and 3 only (d) 1, 2 and 3 Ans: (a) Explanation: United Nations Convention to Combat Desertification (UNCCD): • Established in 1994.

Telegram: https://t.me/insightsIAStips 72 Youtube: https://www.youtube.com/channel/UCpoccbCX9GEIwaiIe4HLjwA

Revision Through MCQs (RTM) Compilation (June 2021)

• It is the sole legally binding international agreement linking environment and development to sustainable land management. • It is the only convention stemming from a direct recommendation of the Rio Conference’s Agenda 21. • Focus areas: The Convention addresses specifically the arid, semi-arid and dry sub-humid areas, known as the drylands, where some of the most vulnerable ecosystems and peoples can be found. • S3: The permanent secretariat of the Convention was established in Article 23 of the UNCCD. It has been located in Bonn, Germany since January 1999.

Refer: https://www.insightsonindia.com/2021/06/16/high-level-dialogue-on- desertification-land-degradation-and-drought-dldd/

139. Consider the following pairs: Tradition State 1. Shigmo Festival Manipur 2. Chillai Kalan Jharkhand 3. Raja Parba festival Odisha Which of the above pairs is/are correctly matched? (a) 1 and 2 only (b) 3 only (c) 2 and 3 only (d) 1 and 3 only Ans: (b) Explanation • S3: Raja Parba festival of Odisha: o This is a three-day-long festival dedicated to Mother Earth (Bhuma Devi) and womanhood at large. o The first day of the festival is called Pahili Raja, the second is Mithuna Sankranti and the third Bhu daha or Basi Raja. o The fourth and final day is called Vasumati snana (bathing of Mother Earth). o This festival is also associated with the end of the summer season and the arrival of the monsoon. • S2: Chillai Kalan:: Kashmir valley read here>> • S1: Shigmo festival:: Goa read here>>

Refer: facts for prelims: https://www.insightsonindia.com/2021/06/16/high-level- dialogue-on-desertification-land-degradation-and-drought-dldd/

140. The ‘World Giving Index’ (WGI) is released by which of the following? (a) Transparency International (b) Amnesty International (c) Walk Free Foundation (d) Charities Aid Foundation Ans: (d) Explanation: World Giving Index 2021: • The World Giving Index (WGI) is an annual report published by the Charities Aid Foundation. The report is the world’s largest survey of charitable endeavors from around the world. The first edition was released in September 2010.

Telegram: https://t.me/insightsIAStips 73 Youtube: https://www.youtube.com/channel/UCpoccbCX9GEIwaiIe4HLjwA

Revision Through MCQs (RTM) Compilation (June 2021)

Refer: facts for prelims: https://www.insightsonindia.com/2021/06/16/high-level- dialogue-on-desertification-land-degradation-and-drought-dldd/

RTM- REVISION THROUGH MCQS – 17th -Jun-2021

141. Consider the following statements about Public Accounts Committee of Indian Parliament: 1. The PAC is formed every year with a strength of not more than 22 members. 2. As per the panel’s rules, no subject can be deliberated upon till there is a consensus among all members. 3. Its chief function is to examine the audit report of Comptroller and Auditor General (CAG) before it is laid in the Parliament. Which of the given above statements is/are correct? (a) 1 and 3 only (b) 1 and 2 only (c) 2 and 3 only (d) 1, 2 and 3 Ans: (b) Explanation: • S1: The PAC is formed every year with a strength of not more than 22 members of which 15 are from Lok Sabha and 7 from Rajya Sabha. • S2: As per the panel’s rules, no subject can be deliberated upon till there is a consensus among all members. • S3: Its chief function is to examine the audit report of Comptroller and Auditor General (CAG) after it is laid in the Parliament.

Refer: https://www.insightsonindia.com/2021/06/17/public-accounts-committee-pac/

142. Consider the following statements: 1. Tuberculosis (TB) is caused by a bacterium called Mycobacterium tuberculosis. 2. The bacteria that cause TB are spread when an infected person coughs or sneezes. Which of the given above statements is/are correct? (a) 1 only (b) 2 only (c) Both 1 and 2 (d) Neither 1 nor 2 Ans: (c) Explanation: • TB is an infectious disease caused by the bacillus Mycobacterium tuberculosis. • It typically affects the lungs (pulmonary TB) but can also affect other sites. • The disease is spread when people who are sick with pulmonary TB expel bacteria into the air, for example by coughing.

Refer: https://www.insightsonindia.com/2021/06/17/who-global-tuberculosis- programme/

Telegram: https://t.me/insightsIAStips 74 Youtube: https://www.youtube.com/channel/UCpoccbCX9GEIwaiIe4HLjwA

Revision Through MCQs (RTM) Compilation (June 2021)

143. In which one of the following groups are all the four countries Dialogue Partners of ASEAN Defence Ministers Meeting (ADMM) Plus? (a) China, India, Japan and New Zealand (b) Australia, China, Democratic People's Republic of Korea and Japan (c) India, Bangladesh, Russia and the USA (d) Australia, China, India and Sri Lanka Ans: (a) Explanation: • The ADMM-Plus is a platform for ASEAN and its eight Dialogue Partners to strengthen security and defence cooperation for peace, stability, and development in the region. • Eight Dialogue Partners are Australia, China, India, Japan, New Zealand, Republic of Korea, Russia and the USA (collectively referred to as the “Plus Countries”). • It aims to promote mutual trust and confidence between defence establishments through greater dialogue and transparency.

Refer: https://www.insightsonindia.com/2021/06/17/asean-defence-ministers-meeting- plus-admm-plus-3/

144. Consider the following statements about Inland Waterways Authority of India (IWAI): 1. It was constituted under National Waterways Act, 2016. 2. The meetings of the statutory authority is chaired by the Union Home Minister. Which of the given above statements is/are correct? (a) 1 only (b) 2 only (c) Both 1 and 2 (d) Neither 1 nor 2 Ans: (d) Explanation: • Inland Waterways Authority of India (IWAI) is the statutory authority in charge of the waterways in India. • It was constituted under IWAI Act-1985 by parliament of India. • Its headquarters is located in Noida, UP • Parent department:: Ministry of Ports, Shipping and Waterways

Refer: https://www.insightsonindia.com/2021/06/17/inland-vessels-bill/

145. The Deep Ocean Mission of India consists which of the following components? 1. development of ocean climate change advisory services 2. searching for deep sea flora and fauna 3. explore and identify potential sources of hydrothermal minerals 4. grooming experts in the field of ocean biology and engineering Select the correct answer using the code below: (a) 2 and 4 only (b) 1, 3 and 4 only (c) 1, 2 and 3 only (d) 1, 2, 3 and 4 Ans: (d)

Telegram: https://t.me/insightsIAStips 75 Youtube: https://www.youtube.com/channel/UCpoccbCX9GEIwaiIe4HLjwA

Revision Through MCQs (RTM) Compilation (June 2021)

Explanation: The Deep Ocean Mission consists of the following six major component • Development of Technologies for Deep Sea Mining, and Manned Submersible • Development of Ocean Climate Change Advisory Services • Technological innovations for exploration and conservation of deep-sea biodiversity • Deep Ocean Survey and Exploration • Energy and freshwater from the Ocean • Advanced Marine Station for Ocean Biology

Refer: https://www.insightsonindia.com/2021/06/17/india-set-to-launch-deep-sea- mission-2/

146. Consider the following statements: 1. India is the largest producer of bananas in the world with a share of around 55% in total output. 2. In India, Tamil Nadu is the largest producer of bananas in the last five years. Which of the given above statements is/are correct? (a) 1 only (b) 2 only (c) Both 1 and 2 (d) Neither 1 nor 2 Ans: (d) Explanation: • S1: India is the world’s leading producer of bananas with a share of around 25% in total output. • S2: Andhra Pradesh stands at the top in area under cultivation, production and export of bananas in the country. Read more>>

Refer: Facts for Prelims: https://www.insightsonindia.com/2021/06/17/insights-daily- current-affairs-pib-summary-17-june-2021/

147. Consider the following statements 1. National Highways Authority of India (NHAI) has made mandatory use of drones for monthly video recording of National Highway projects. 2. All project documentation, contractual decisions and approvals are now being done through NHAI’s portal ‘Data Lake’ only. Which of the given above statements is/are correct? (a) 1 only (b) 2 only (c) Both 1 and 2 (d) Neither 1 nor 2 Ans: (c) Explanation: • S1: National Highways Authority of India, under Ministry of Road Transport & Highways has made mandatory use of drones for monthly video recording of National Highway projects during all stages of development, construction, operation and maintenance. • Contractors and Concessionaires shall carry out the drone video recording in presence of Team Leader of the Supervision Consultant and upload comparative project videos of the current and last month on NHAI’s portal ‘Data Lake’.

Telegram: https://t.me/insightsIAStips 76 Youtube: https://www.youtube.com/channel/UCpoccbCX9GEIwaiIe4HLjwA

Revision Through MCQs (RTM) Compilation (June 2021)

Refer: Facts for Prelims: https://www.insightsonindia.com/2021/06/17/insights-daily- current-affairs-pib-summary-17-june-2021/

148. Consider the following statements: 1. Longitude of Dholavira’s location is between those of Lothal and Ganweriwala. 2. Latitude of Harappa’s location is between those of Banawali and Rakhigarhi. 3. Mohenjo-Daro situated more southward than Kalibangan. Which of the given above statements is/are not correct? (a) 1 and 2 only (b) 3 only (c) 2 and 3 only (d) 1, 2 and 3 Ans: (a) Explanation: here the directive word is not correct!! • Self-explanatory

• Refer: Facts for Prelims: https://www.insightsonindia.com/2021/06/17/insights-daily- current-affairs-pib-summary-17-june-2021/

149. Which one of the following has launched Adi Prashikshan Portal? (a) NITI Aayog (b) Ministry of Culture (c) Ministry of Social Justice and Empowerment (d) Ministry Of Tribal Affairs Ans: (d) Explanation: • Ministry Of Tribal Affairs Has Launched Adi Prashikshan Portal.

Refer: Facts for Prelims: https://www.insightsonindia.com/2021/06/17/insights-daily- current-affairs-pib-summary-17-june-2021/

150. Consider the following statements: 1. Lothal, in present-day Rajasthan, was an ancient city of the Indus Valley civilization. 2. The archeological findings discovered at lothal was dockyward. Which of the given above statements is/are correct? (a) 1 only (b) 2 only (c) Both 1 and 2 (d) Neither 1 nor 2 Ans: (b)

Telegram: https://t.me/insightsIAStips 77 Youtube: https://www.youtube.com/channel/UCpoccbCX9GEIwaiIe4HLjwA

Revision Through MCQs (RTM) Compilation (June 2021)

Explanation: • S1: Lothal was one of the southernmost cities of the ancient Indus Valley Civilization located in the Bhāl region of the modern state of Gujarāt. • S2: Excavations have revealed the world’s oldest known artificial dock, which was connected to an old course of the Sabarmati River. Other features include the acropolis, the lower town, the bead factory, the warehouses, and the drainage system.

Refer: Facts for Prelims: https://www.insightsonindia.com/2021/06/17/insights-daily- current-affairs-pib-summary-17-june-2021/

Telegram: https://t.me/insightsIAStips 78 Youtube: https://www.youtube.com/channel/UCpoccbCX9GEIwaiIe4HLjwA

Revision Through MCQs (RTM) Compilation (June 2021)

RTM- REVISION THROUGH MCQS – 18th -Jun-2021

151. Consider the following statements 1. The Bureau of Indian Standards (BIS) is the national Standards Body of India working under the aegis of Ministry of Statistics and Programme Implementation (MoSPI). 2. In India, the Bureau of Indian Standard (BIS) operates gold and silver hallmarking scheme. Which of the given above statements is/are correct? (a) 1 only (b) 2 only (c) Both 1 and 2 (d) Neither 1 nor 2 Ans: (b) Explanation: • The Bureau of Indian Standards (BIS) is the national Standards Body of India working under the aegis of Ministry of Consumer Affairs, Food & Public Distribution, Government of India. o BIS is the National Standard Body of India established under the BIS Act 2016 for the harmonious development of the activities of standardization, marking and quality certification of goods and for matters connected therewith or incidental thereto • S2: The Bureau of Indian Standard (BIS) operates gold and silver hallmarking scheme in India.

Refer: https://www.insightsonindia.com/2021/06/18/what-is-hallmarking-of-gold-and- for-whom-is-it-now-mandatory/

152. Consider the following statements: 1. Neutrinos are the second most abundant particles in the world. 2. A neutrino’s spin always points in the opposite direction of its motion. 3. Neutrinos can be produced in the lab. Which of the given above statements is/are correct? (a) 1 and 2 only (b) 2 and 3 only (c) 1 and 3 only (d) 1, 2 and 3 Ans: (d) Explanation: • S1: Detected for the first time in 1959, neutrinos are the second most abundant particles in the world, after photons, or the light particle. • S2: A neutrino’s spin always points in the opposite direction of its motion. • S3: Neutrinos are created by various radioactive decays; during a supernova, by cosmic rays striking atoms etc. o They can also be produced in the lab.

Refer: https://www.insightsonindia.com/2021/06/18/neutrinos/

Telegram: https://t.me/insightsIAStips 79 Youtube: https://www.youtube.com/channel/UCpoccbCX9GEIwaiIe4HLjwA

Revision Through MCQs (RTM) Compilation (June 2021)

153. What are ‘Tiangong-1’ and ‘Tiangong-2’, sometimes mentioned in news? (a) Chines underwater attack submarines (b) Japanese space robots (c) South Korea’s indigenous anti-missile programme (d) None of the above Ans: (d) Explanation: • China has sent two previous space stations into orbit- the Tiangong-1 and Tiangong-2 were trial stations.

Refer: https://www.insightsonindia.com/2021/06/18/three-chinese-astronauts-enter- space-station-after-successful-docking-operation/

154. Recently, Twitter has lost its intermediary status in India over non-compliance of the new IT rules that came into effect on May 26. In this context, consider the following statements: 1. In India, the intermediary status is a registration granted by the Government of India. 2. In India, the intermediaries are protected under Section 79 of the Information Technology Act. Which of the given above statements is/are correct? (a) 1 only (b) 2 only (c) Both 1 and 2 (d) Neither 1 nor 2 Ans: (b) Explanation: • As per Section 2 (1) of the Information Technology Act, an intermediary is a person/entity that receives, stores and transmits information or provides service for transmission of information. • S1: Please note, intermediary status is not a registration granted by the government. • S2: Intermediaries like Twitter are protected under Section 79 of the Information Technology Act that states that they cannot be held liable for the third party content published on their platform as long as they comply with the legal order to take down content from courts or other authorities.

Refer: https://www.insightsonindia.com/2021/06/18/what-loss-of-safe-harbour-means- for-twitter/

155. What is Kerala’s SilverLine project, sometimes mentioned in the news? (a) It is a flagship semi high-speed railway project (b) It is an ambitious Fibre Optic Network (K-FON) project (c) It is a project to create Miyawaki model micro forests in the state (d) None of the above Ans: (a) Explanation: • It is Kerala’s flagship semi high-speed railway project aimed at reducing travel time between the state’s northern and southern ends.

Telegram: https://t.me/insightsIAStips 80 Youtube: https://www.youtube.com/channel/UCpoccbCX9GEIwaiIe4HLjwA

Revision Through MCQs (RTM) Compilation (June 2021)

• It links Kerala’s southern end and state capital Thiruvananthapuram with its northern end of Kasaragod. • The line is proposed to be 529.45 kms long, covering 11 districts. • The project is being executed by the Kerala Rail Development Corporation Limited (KRDCL). KRDCL, or K-Rail, is a joint venture between the Kerala government and the Union Ministry of Railways.

• Refer: facts for prelims: https://www.insightsonindia.com/2021/06/18/insights-daily- current-affairs-pib-summary-18-june-2021/

156. ‘Desertification and Land Degradation Atlas of India’ has been published by which of the following? (a) Department of Science and Technology (b) Department of Land Resources (c) National Disaster Management Authority (d) Indian Space Research Organisation Ans: (d) Explanation: Desertification and Land Degradation Atlas of India: • It has been published by Space Application Centre, ISRO, Ahmedabad. • The Atlas provides state wise area of degraded lands for the time frame 2018-19. • It was released on the occasion of the World Day to Combat Desertification and Drought, observed on 17 June. • The theme for 2021 is “Restoration. Land. Recovery. We build back better with healthy land”.

Refer: facts for prelims: https://www.insightsonindia.com/2021/06/18/insights-daily- current-affairs-pib-summary-18-june-2021/

157. The shy ‘white-bellied heron’ was spotted in the high Himalayas for the first time in India. In this context, consider the following statements 1. It is categorised as ‘critically endangered’ in International Union for Conservation of Nature (IUCN) Red Data Book. 2. It is listed in Schedule I in the Wildlife Protection Act, 1972. 3. In India, it is found only in the Namdapha Tiger Reserve. Which of the given above statements is/are correct? (a) 1 and 2 only (b) 1 only (c) 2 and 3 only (d) 2 only Ans: (b)

Telegram: https://t.me/insightsIAStips 81 Youtube: https://www.youtube.com/channel/UCpoccbCX9GEIwaiIe4HLjwA

Revision Through MCQs (RTM) Compilation (June 2021)

Explanation: • The white-bellied heron is categorised as ‘critically endangered’ in the International Union for Conservation of Nature (IUCN) Red Data Book and is listed in Schedule IV in the Wildlife Protection Act, 1972. • It is one of the rarest birds in the world and is at present found only in Bhutan, Myanmar and the Namdapha Tiger Reserve in Arunachal Pradesh. • It had also been recorded in the adjacent Kamlang Tiger Reserve in Lohit district in camera trap images.

Refer: https://science.thewire.in/environment/rare-white-bellied-heron-spotted-in- arunachal-pradesh/

158. Consider the following statements: 1. It lies in between Lang River on the North side and famous Namdapha National Park on its south. 2. This Wildlife Sanctuary contains all four big cats (Tiger, Leopard, Clouded Leopard and Snow Leopard). 3. Glao or Glow Lake is the most significant water body inside this sanctuary. The above given statements refers to which one of the following protected area? (a) Namdapha Tiger Reserve (b) Kamlang Wildlife Sanctuary (c) Pakhui Wildlife Sanctuary (d) Dibang Wildlife Sanctuary Ans: (b) Explanation: • Kamlang Wildlife Sanctuary is situated in the South-Eastern part of Lohit District of Arunachal Pradesh. The name comes from the River Kamlang, which flows through the Sanctuary and joins Brahmaputra.

Refer: http://arunachalforests.gov.in/kamlang_wildlife_sanctuary.html

159. Consider the following statements: 1. Western hoolock gibbon is listed as ‘Endangered’ under the IUCN Redlist. 2. Eastern hoolock gibbon is listed as ‘Vulnerable’ under the IUCN Redlist. Which of the given above statements is/are correct? (a) 1 only (b) 2 only (c) Both 1 and 2 (d) Neither 1 nor 2 Ans: (c) Explanation: • Western hoolock gibbon: Endangered under the IUCN Redlist. • Eastern hoolock gibbon: Vulnerable under the IUCN Redlist.

Refer: https://india.mongabay.com/2021/04/not-two-only-one-species-of-hoolock-gibbon- in-india-study/

160. What is/are the purpose/purposes of `District Mineral Foundations’ in India? 1. Promoting mineral exploration activities in mineral-rich districts 2. Protecting the interests of the persons affected by mining operations

Telegram: https://t.me/insightsIAStips 82 Youtube: https://www.youtube.com/channel/UCpoccbCX9GEIwaiIe4HLjwA

Revision Through MCQs (RTM) Compilation (June 2021)

3. Authorizing State Governments to issue licences for mineral exploration Select the correct answer using the code given below. (a) 1 and 2 only (b) 2 only (c) 1 and 3 only (d) 1, 2 and 3 Ans: (b) Explanation: • District Mineral Foundation (DMF) is a trust set up as a non-profit body, in those districts affected by the mining works, to work for the interest and benefit of persons and areas affected by mining related operations. It is funded through the contributions from miners. • So, all other statements apart from 2 are wrong. • Setting up of District Mineral Foundations (DMFs) in all districts in the country affected by mining related operations was mandated through the Mines and Minerals (Development & Regulation) Amendment Act, (MMDRA) 2015. • Its manner of operation comes under the jurisdiction of the relevant State Government.

Refer: UPSC CSE 2016

RTM- REVISION THROUGH MCQS – 19th -Jun-2021 161. With reference to Flag Satyagraha, consider the following statements: 1. It was the first Satyagraha movement led by Sardar Vallabhbhai Patel in India. 2. It contributed to the revival of the indigenous artisan crafts and industries. Which of the given above statements is/are correct? (a) 1 only (b) 2 only (c) Both 1 and 2 (d) Neither 1 nor 2 Ans: (d) Explanation: • What is Flag Satyagraha? o The Flag Satyagraha movement by the freedom fighters shook the British government and it infused a new life into the freedom movement. o Also called the Jhanda Satyagraha, it was held in Jabalpur and Nagpur in 1923. o The news of flag hoisting in Jabalpur spread like fire in the country and after flags were hoisted at several places across the country. • Significance: o It is a campaign of peaceful civil disobedience that focused on exercising the right and freedom to hoist the nationalist flag and challenge the legitimacy of the British Rule in India through the defiance of laws prohibiting the hoisting of nationalist flags and restricting civil freedoms. • Outcomes: o The arrest of nationalist protestors demanding the right to hoist the flag caused an outcry across India especially as Gandhi had recently been arrested.

Telegram: https://t.me/insightsIAStips 83 Youtube: https://www.youtube.com/channel/UCpoccbCX9GEIwaiIe4HLjwA

Revision Through MCQs (RTM) Compilation (June 2021)

o Nationalist leaders such as Sardar Vallabhbhai Patel, Jamnalal Bajaj, Chakravarthi Rajagopalachari, Dr. Rajendra Prasad and Vinoba Bhave organised the revolt and thousands of people from different regions traveled to Nagpur and other parts of the Central Provinces to participate in civil disobedience. o In the end, the British negotiated an agreement with Patel and other Congress leaders permitting the protestors to conduct their march unhindered and obtaining the release of all those arrested.

Refer: https://www.insightsonindia.com/2021/06/19/flag-satyagraha/

162. Consider the following statements: 1. In the Indian judicial system, there are no definite rules on recusals by Judges. 2. Judicial disqualification is the act of abstaining from participation in an official action due to a conflict of interest of the administrative officer. Which of the given above statements is/are correct? (a) 1 only (b) 2 only (c) Both 1 and 2 (d) Neither 1 nor 2 Ans: (c) Explanation: • S2: Judicial disqualification, referred to as recusal, is the act of abstaining from participation in an official action such as a legal proceeding due to a conflict of interest of the presiding court official or administrative officer. • S1: There are no definite rules on recusals by Judges. However, in taking oath of office, judges, both of the Supreme Court and of the high courts, promise to perform their duties, to deliver justice, “without fear or favour, affection or ill- will”.

Refer: https://www.insightsonindia.com/2021/06/19/recusal-of-judges-3/

163. Consider the following statements: 1. An election petition can be filed either by a candidate or a voter in the concerned constituency of the State. 2. An Election Petition has to be filed within 45 days from the declaration of the results of the State Assembly elections. 3. Election petition can be filed in the High Court of the State to which the State Assembly elections pertain. Which of the given above statements is/are correct? (a) 1 and 2 only (b) 2 and 3 only (c) 1 and 3 only (d) 1, 2 and 3 Ans: (d) Explanation: • S1: An election petition can be filed either by a candidate or a voter in the concerned constituency of the State. Other than these persons, no one can approach a High Court with an election petition. Voter is a person who was

Telegram: https://t.me/insightsIAStips 84 Youtube: https://www.youtube.com/channel/UCpoccbCX9GEIwaiIe4HLjwA

Revision Through MCQs (RTM) Compilation (June 2021)

entitled to vote at the election to which the election petition relates. Whether he has voted at such election or not is immaterial. (Sec 81(1) explanation of RPA) • S2: An Election Petition has to be filed within 45 days from the declaration of the results of the State Assembly elections. • S3: Election petition can be filed in the High Court of the State to which the State Assembly elections pertain. (80A RPA)

Refer: https://www.insightsonindia.com/2021/06/19/how-are-poll-results-challenged- and-when-courts-have-set-them-aside/

164. The Returning Officers for the State Assembly Elections are appointed by the: (a) President of India (b) Governor (c) State Election Commission (d) Election Commission of India Ans: (d) Explanation: • A Returning Officer is responsible for overseeing the election in a constituency, or sometimes in two constituencies, as directed by the Election Commission (EC). • The EC appoints the Returning Officer and Assistant Returning Officer for a constituency in consultation with the governments of the State or Union Territory as the case may be. • The Returning Officer’s duties include accepting and scrutinising nomination forms, publishing the affidavits of candidates, allotting symbols to the contesting candidates, preparing the list of contesting candidates, preparing the EVMs and VVPATs, training polling personnel, designating counting centres, and counting the votes and declaring the result.

Refer: https://www.insightsonindia.com/2021/06/19/how-are-poll-results-challenged- and-when-courts-have-set-them-aside/

165. Consider the following statements about Ordnance Factory Board (OFB): 1. It is currently a subordinate office of the Ministry of Defence (MoD). 2. Recently, the Union Cabinet has approved a plan to corporatize the Ordnance Factory Board (OFB). Which of the following statements is/are correct? (a) 1 only (b) 2 only (c) Both 1 and 2 (d) Neither 1 nor 2 Ans: (c) Explanation: • The Union Cabinet has approved a plan to corporatize the Ordnance Factory Board (OFB). • Ordnance Factory Board (OFB): o It is an umbrella body for the ordnance factories and related institutions, and is currently a subordinate office of the Ministry of Defence (MoD). o The first Indian ordnance factory was set up in the year 1712 by the Dutch Company as a GunPowder Factory, West Bengal.

Telegram: https://t.me/insightsIAStips 85 Youtube: https://www.youtube.com/channel/UCpoccbCX9GEIwaiIe4HLjwA

Revision Through MCQs (RTM) Compilation (June 2021)

o OFBs will be responsible for different verticals of the products such as the Ammunition and Explosives group will be engaged in production of ammunition while a Vehicles group will engage in production of defence mobility and combat vehicles.

Refer: https://www.insightsonindia.com/2021/06/19/corporatization-of-ordnance-factory- board/

166. “Children and Digital Dumpsites” is released by which of the following? (a) United Nations Children’s Fund (UNICEF) (b) United Nations Educational, Scientific and Cultural Organization (UNESCO) (c) World Health Organization (WHO) (d) World Economic Forum (WEF) Ans: (c) Explanation: • The new report, titled Children and Digital Dumpsites, was recently released by the WHO. • Key findings: o More than 18 million children and adolescents working at e-waste dumpsites in low- and middle-income countries are potentially at the risk of severe health hazards. o They face risk due to discarded electronic devices or e-waste being dumped from high-income countries.

Refer: https://www.insightsonindia.com/2021/06/19/children-and-digital-dumpsites- report/

167. In India, ‘extended producer responsibility’ was introduced as an important feature in which of the following? (a) The Bio-medical Waste (Management and Handling) Rules, 1998 (b) The Recycled plastic (Manufacturing and Usage) Rules, 1999 (c) The e-Waste (Management and Handling) Rules, 2011 (d) The Food Safety and Standard Regulations, 2011 Ans: (c) Explanation: • The e-Waste (Management and Handling) Rules, 2011 recognises producers’ liability for recycling and reducing e-waste in the country. • Extended Producer Responsibility (EPR) is a policy approach under which producers are given a significant responsibility – financial and/or physical – for the treatment or disposal of post-consumer products. • The Central Pollution Control Board (CPCB) has been given the Extended Producer Responsibility (EPR) authorisation under the new e-waste rules

Refer: https://www.insightsonindia.com/2021/06/19/children-and-digital-dumpsites- report/

168. With reference to India State of Forest Report (ISFR) 2019, which of the following states are showing mangrove cover increase as compared to the previous assessment of 2017? 1. Gujarat 2. Maharashtra

Telegram: https://t.me/insightsIAStips 86 Youtube: https://www.youtube.com/channel/UCpoccbCX9GEIwaiIe4HLjwA

Revision Through MCQs (RTM) Compilation (June 2021)

3. Odisha Select the correct answer using the code below: (a) 1 and 2 only (b) 1 only (c) 2 and 3 only (d) 1, 2 and 3 Ans: (d) Explanation: • The Mangrove ecosystems are unique & rich in biodiversity and they provide numerous ecological services. • Mangrove cover has been separately reported in the ISFR 2019 and the total mangrove cover in the country is 4,975 sq km. • An increase of 54 sq Km in mangrove cover has been observed as compared to the previous assessment of 2017. • Top three states showing mangrove cover increase are Gujarat (37 sq km) followed by Maharashtra (16 sq km) and Odisha (8 sq km).4

Refer: https://www.insightsonindia.com/2021/06/19/barrier-to-cyclone-storms-odisha- plans-to-plant-mangroves-along-its-coast/

169. Consider the following statements about Quality Council of India: 1. It was set up as an autonomous body attached to the Department of Industrial Policy & Promotion, Ministry of Commerce and Industry. 2. It is headed by the Minister of Commerce and Industry of India. Which of the given above statements is/are correct? (a) 1 only (b) 2 only (c) Both 1 and 2 (d) Neither 1 nor 2 Ans: (a) Explanation: • S1: The Quality Council of India (QCI) was set up as an autonomous body attached to the Department of Industrial Policy & Promotion, Ministry of Commerce and Industry. • S2: Chairman of QCI is appointed by the Prime Minister on recommendation of the industry to the government. • Indian Certification of Medical Devices (ICMED) Plus Scheme: o Launched by Quality Council of India (QCI). o This is an upgraded ICMED Scheme that was launched for Certification of Medical Devices in 2016.

Refer: facts for prelims: https://www.insightsonindia.com/2021/06/19/insights-daily- current-affairs-pib-summary-19-june-2021/

170. Consider the following statements: 1. The Bab-el-Mandeb strait connects the with the Red Sea. 2. The Gulf of Aden offers the only entrance from the Arabian Sea and the Indian Ocean into the Persian Gulf. Which of the given above statements is/are correct? (a) 1 only

Telegram: https://t.me/insightsIAStips 87 Youtube: https://www.youtube.com/channel/UCpoccbCX9GEIwaiIe4HLjwA

Revision Through MCQs (RTM) Compilation (June 2021)

(b) 2 only (c) Both 1 and 2 (d) Neither 1 nor 2 Ans: (a) Explanation: • S1: The Bab-el-Mandeb strait connects the Gulf of Aden with the Red Sea.

o • S2: The Gulf of Aden is a gulf amidst Yemen to the north,the Arabian Sea and Guardafui Channel to the east, Somalia to the south, and Djibouti to the west. • S3: The Gulf of Oman offers the only entrance from the Arabian Sea and the Indian Ocean into the Persian Gulf.

o Refer: facts for prelims: https://www.insightsonindia.com/2021/06/19/insights-daily- current-affairs-pib-summary-19-june-2021/

RTM- REVISION THROUGH MCQS – 21st -Jun-2021

171. On 21st June, the Sun (a) does not set below the horizon at the Arctic Circle (b) does not set below the horizon at Antarctic Circle (c) shines vertically overhead at noon on the Equator (d) shines vertically overhead at the Tropic of Capricorn Ans: (a) Explanation: • June 21- this day is referred to as the summer solstice, the longest day of the summer season. It occurs when the sun is directly over the Tropic of Cancer. • For that hemisphere, the summer solstice is when the Sun reaches its highest position in the sky and is the day with the longest period of daylight. At the pole, there is continuous daylight around the summer solstice.

Telegram: https://t.me/insightsIAStips 88 Youtube: https://www.youtube.com/channel/UCpoccbCX9GEIwaiIe4HLjwA

Revision Through MCQs (RTM) Compilation (June 2021)

Refer: https://www.insightsonindia.com/2021/06/21/summer-solstice/

172. With reference to Integrated power development scheme, consider the following statements: 1. The scheme is spearheaded by the Energy Efficiency Services Limited (EESL). 2. All Discoms are eligible for financial assistance under the scheme. Which of the given above statements is/are correct? (a) 1 only (b) 2 only (c) Both 1 and 2 (d) Neither 1 nor 2 Ans: (b) Explanation: The Integrated Power Development Scheme: • S1: Power Finance Corporation (PFC) is the Nodal agency for implementation of the scheme. • S2: All Discoms will be eligible for financial assistance under the scheme.

Refer: https://www.insightsonindia.com/2021/06/21/integrated-power-development- scheme-ipds/

173. Consider the following statements about Ebola: 1. It is caused by eating food or drinking water contaminated with the Ebola virus. 2. There is no cure or specific treatment for the Ebola virus disease that is currently approved for market. Which of the given above statements is/are correct? (a) 1 only (b) 2 only (c) Both 1 and 2 (d) Neither 1 nor 2 Ans: (b) Explanation: • S1: Ebola is caused by viruses in the Ebolavirus and Filoviridae family. Ebola is considered a zoonosis, meaning that the virus is present in animals and is transmitted to humans. o Ebola is spread by direct contact with blood or other body fluids (such as: vomit, diarrhea, urine, breast milk, sweat, semen) of an infected person who has symptoms of Ebola or who has recently died from Ebola. • S2: There is no cure or specific treatment for the Ebola virus disease that is currently approved for market, although various experimental treatments are being developed. For past and current Ebola epidemics, treatment has been primarily supportive in nature.

Refer: https://www.insightsonindia.com/2021/06/21/ebola-outbreak/

174. With reference to Republic of the Union of Myanmar, consider the following statements: 1. The Republic of the Union of Myanmar is a member of the East Asia Summit, Non- Aligned Movement, and ASEAN, but not a member of the Commonwealth of Nations. 2. It is bordered in the northwest by the Mizoram, Manipur, Nagaland and Arunachal Pradesh states of India.

Telegram: https://t.me/insightsIAStips 89 Youtube: https://www.youtube.com/channel/UCpoccbCX9GEIwaiIe4HLjwA

Revision Through MCQs (RTM) Compilation (June 2021)

3. The equator passes through water belonging to the Republic of the Union of Myanmar. Which of the given above statements is/are correct? (a) 1 and 3 only (b) 2 and 3 only (c) 1 and 2 only (d) 1, 2 and 3 Ans: (c) Explanation: • S1: Myanmar is a member of the East Asia Summit, Non-Aligned Movement, ASEAN, and BIMSTEC, but it is not a member of the Commonwealth of Nations. • S2: Myanmar is bordered in the northwest by the Chittagong Division of Bangladesh and the Mizoram, Manipur, Nagaland and Arunachal Pradesh states of India. • S3: Of the 13 countries that lie on the equator, seven are in Africa—the most of any continent—and South America is home to three of the nations. The remaining countries are island nations in the Indian and Pacific oceans.

• Refer: https://www.insightsonindia.com/wp- admin/post.php?post=176718&action=edit&classic-editor

175. Consider the following statements: 1. Monoclonal antibodies plays an important role in hemoglobin production. 2. They can be created in the lab by exposing white blood cells to a particular antigen. Which of the given above statements is/are correct? (a) 1 only (b) 2 only (c) Both 1 and 2 (d) Neither 1 nor 2 Ans: (b) Explanation: What are Monoclonal antibodies? • They are artificially created antibodies that aim to aid the body’s natural immune system. • They target a specific antigen — a protein from the pathogen that induces immune response. • How are they created? o Monoclonal antibodies can be created in the lab by exposing white blood cells to a particular antigen.

Telegram: https://t.me/insightsIAStips 90 Youtube: https://www.youtube.com/channel/UCpoccbCX9GEIwaiIe4HLjwA

Revision Through MCQs (RTM) Compilation (June 2021)

o To increase the quantity of antibodies produced, a single white blood cell is cloned, which in turn is used to create identical copies of the antibodies. o In the case of Covid-19, scientists usually work with the spike protein of the SARS-CoV-2 virus, which facilitates the entry of the virus into the host cell.

Refer: https://www.insightsonindia.com/2021/06/21/monoclonal-antibodies/

176. The terms ‘Cry1Ab’ and ‘Cry2Bc’ sometimes mentioned in the news recently are related to (a) New variants of Covid-19 (b) Crypto currencies (c) Sleeping tablets (d) GM crops Ans: (d) Explanation: • Bt cotton remains the only GM crop allowed to be cultivated in the country. o Developed by US giant Bayer-Monsanto, it involves insertion of two genes viz ‘Cry1Ab’ and ‘Cry2Bc’ from the soil bacterium Bacillus thuringiensis into cotton seeds. • This modification codes the plant to produce protein toxic to Heliothis bollworm (pink bollworm) thus making it resistant to their attack. The commercial release of this hybrid was sanctioned by the government in 2002.

Refer: https://www.insightsonindia.com/2021/06/21/sale-of-illegal-htbt-cotton-seeds- doubles/

177. Consider the following statements about the Hubble Space Telescope: 1. It is the only telescope designed to be serviced in space by astronauts. 2. It was built by the United States space agency NASA, with contributions from the German Aerospace Center (DLR). Which of the given above statements is/are correct? (a) 1 only (b) 2 only (c) Both 1 and 2 (d) Neither 1 nor 2 Ans: (a) Explanation: • About the Hubble Space Telescope: o The Hubble Space Telescope is a large telescope in space. NASA launched Hubble in 1990. o It was built by the United States space agency NASA, with contributions from the European Space Agency. o Hubble is the only telescope designed to be serviced in space by astronauts. o Expanding the frontiers of the visible Universe, the Hubble Space Telescope looks deep into space with cameras that can see across the entire optical spectrum from infrared to ultraviolet.

Telegram: https://t.me/insightsIAStips 91 Youtube: https://www.youtube.com/channel/UCpoccbCX9GEIwaiIe4HLjwA

Revision Through MCQs (RTM) Compilation (June 2021)

o The Hubble Space Telescope makes one orbit around Earth every 95 minutes. • About SOFIA: o SOFIA is a modified Boeing 747SP jetliner that flies at altitudes up to 45,000 feet. o It is a joint project of NASA and the German Aerospace Center. o It has an infrared camera that picks up the wavelength unique to water molecules. o SOFIA’s mission is to look at dark and distant objects. The Moon, on the other hand, is so close and bright that it fills the SOFIA guide camera’s entire field of view.

Refer: https://www.insightsonindia.com/2021/06/21/hubble-space-telescope/

178. Consider the following statements: 1. Pink hydrogen is generated through electrolysis powered by nuclear energy. 2. Grey Hydrogen is produced from natural gas, where the emissions are captured using carbon capture and storage. 3. Blue Hydrogen is produced from natural gas where the associated emissions are released to the air. Which of the given above statements is/are correct? (a) 1 only (b) 2 and 3 only (c) 1, 2 and 3 (d) 2 only Ans: (a) Explanation: • Brown hydrogen is produced using coal where the emissions are released to the air • Grey hydrogen is produced from natural gas where the associated emissions are released to the air • Blue hydrogen is produced from natural gas, where the emissions are captured using carbon capture and storage • Green hydrogen is produced from electrolysis powered by renewable electricity. • Pink hydrogen is generated through electrolysis powered by nuclear energy

Refer: https://www.insightsonindia.com/2021/06/21/summit-on-green-hydrogen- initiatives/

179. The Expert Group under the Chairmanship of Ajit Mishra was in news recently, is related to which of the following? (a) Minimum Support Prices (b) Fair and Remunerative Prices (c) Fixation of Minimum Wages (d) Economic Capital Framework Ans: (c) Explanation: Ajit Mishra expert group: • The Central Government has constituted an Expert Group under the Chairmanship of Professor Ajit Mishra, a renowned economist.

Telegram: https://t.me/insightsIAStips 92 Youtube: https://www.youtube.com/channel/UCpoccbCX9GEIwaiIe4HLjwA

Revision Through MCQs (RTM) Compilation (June 2021)

• Its mandate is to provide technical inputs and recommendations on fixation of Minimum Wages and National Floor Wages to the Government.

Refer: facts for prelims: https://www.insightsonindia.com/2021/06/21/insights-daily- current-affairs-pib-summary-21-june-2021/

180. The ‘Biotech-Krishi Innovation Science Application Network’ (Biotech-KISAN) is an initiative of (a) Department of Agriculture Cooperation & Farmers Welfare (b) NITI Aayog (c) Department of Science and Technology (d) Ministry of Skill Development and Entrepreneurship Ans: (c) Explanation: • Biotech-Krishi Innovation Science Application Network (Biotech-KISAN) is a Department of Biotechnology, Ministry of Science and Technology initiative that empowers farmers, especially women farmers. It aims to understand the problems of water, soil, seed and market faced by the farmers and provide simple solutions to them. • It is a pan-India program, following a hub-and-spoke model and stimulates entrepreneurship and innovation in farmers and empowers women farmers. • It identifies and promotes local farm leadership in both genders. Such leadership helps to develop science-based farming besides facilitating the transfer of knowledge.

Refer: facts for prelims: https://www.insightsonindia.com/2021/06/21/insights-daily- current-affairs-pib-summary-21-june-2021/

RTM- REVISION THROUGH MCQS – 22nd -Jun-2021

181. Consider the following statements: 1. The jurisdiction of a High Court can be shifted only through an Act of Parliament. 2. The initiation of the proposal for the transfer of a Judge (including Chief Justice) from one High Court to any other High Court can be made by the President of India. Which of the given above statements is/are correct? (a) 1 only (b) 2 only (c) Both 1 and 2 (d) Neither 1 nor 2 Ans: (a) Explanation: • S1: The jurisdiction of a High Court can be shifted only through an Act of Parliament. o Article 241 states that Parliament may by law constitute a high court for a Union Territory or declare any court in any such territory to be a high court for all or any of the purposes of this Constitution.

Telegram: https://t.me/insightsIAStips 93 Youtube: https://www.youtube.com/channel/UCpoccbCX9GEIwaiIe4HLjwA

Revision Through MCQs (RTM) Compilation (June 2021)

• S2: Article 222 of the Constitution makes provision for the transfer of a Judge (including Chief Justice) from one High Court to any other High Court. • The initiation of the proposal for the transfer of a Judge should be made by the Chief Justice of India whose opinion in this regard is determinative.

Refer: https://www.insightsonindia.com/2021/06/22/shifting-of-jurisdiction-of-a-high- court/

182. Consider the following statements: 1. Under Article 82, the Parliament can enacts a Delimitation Act after every Census. 2. Once the Delimitation Act is in force, President of India sets up the Delimitation Commission. Which of the given above statements is/are correct? (a) 1 Only (b) 2 Only (c) Both 1 and 2 (d) Neither 1 nor 2 Ans: (a) Explanation: • Delimitation is carried out by an independent Delimitation Commission. • The Constitution mandates that its orders are final and cannot be questioned before any court as it would hold up an election indefinitely. • S1: Under Article 82, the Parliament enacts a Delimitation Act after every Census. • S2: Once the Act is in force, the Union government sets up a Delimitation Commission.

Refer: https://www.insightsonindia.com/2021/06/22/delimitation-in-jammu-and- kashmir/

183. Consider the following statements about Chief of Defence Staff (CDS): 1. The CDS is a five-star officer selected from among the serving officers of the Indian Armed Forces. 2. The CDS heads the Department of Military Affairs under the Ministry of Defence, as its secretary. Which of the given above statements is/are correct? (a) 1 only (b) 2 only (c) Both 1 and 2 (d) Neither 1 nor 2 Ans: (b) Explanation: • The CDS is a four-star officer selected from among the serving officers of the Indian Armed Forces. While being "first among equals" among the service chiefs, the CDS is a single-point military advisor to the defence minister. • The CDS is assisted by a deputy, the Vice Chief of the Defence Staff. The CDS heads the Department of Military Affairs under the Ministry of Defence, as its secretary. Apart from heading the DMA, the CDS is the Permanent Chairperson of the Chiefs of Staff Committee (PC-CoSC).

Telegram: https://t.me/insightsIAStips 94 Youtube: https://www.youtube.com/channel/UCpoccbCX9GEIwaiIe4HLjwA

Revision Through MCQs (RTM) Compilation (June 2021)

Refer: https://www.insightsonindia.com/2021/06/22/chief-of-defence-staff-3/

184. With reference to regulation of films and movies in India, consider the following statements: 1. At present, there are no enabling provisions to check film piracy. 2. Currently a certificate issued by the Central Board of Film Certification (CBFC) is valid only for 10 years. Which of the given above statements is/are correct? (a) 1 only (b) 2 only (c) Both 1 and 2 (d) Neither 1 nor 2 Ans: (c) Explanation: The draft Cinematograph (Amendment) Bill 2021: The draft was recently released by the Centre. It seeks to amend the Cinematograph Act of 1952. • Key Provisions: • Revision of certification: This provision will give the Centre “revisionary powers” and enable it to “re-examine” films already cleared by the Central Board of Film Certification (CBFC). • Age-based certification: It seeks to introduce age-based categorisation and classification. It proposes to divide the existing categories (U, U/A and A) into further age-based groups: U/A 7+, U/A 13+ and U/A 16+. • S1: Provision against piracy: At present, there are no enabling provisions to check film piracy. Violation shall be punishable with imprisonment and fine. • S2: Eternal certificate: It proposes to certify films for perpetuity. Currently a certificate issued by the CBFC is valid only for 10 years.

Refer: https://www.insightsonindia.com/2021/06/22/the-draft-cinematograph- amendment-bill-2021/

185. Consider the following statements about National Initiative for School Heads and Teachers Holistic Advancement (NISHTHA): 1. It is the largest teacher’s training programme of its kind in the world. 2. It has been launched under the Central Sector Scheme of Samagra Shiksha in 2019-20. Which of the given above statements is/are correct? (a) 1 only (b) 2 only (c) Both 1 and 2 (d) Neither 1 nor 2 Ans: (a) Explanation: • National Initiative for School Heads and Teachers Holistic Advancement (NISHTHA) is a national mission to improve learning outcomes at the elementary level. This is done through integrated teacher training. o It is the largest teachers’ training programme of its kind in the world. o It has been launched under the Centrally Sponsored Scheme of Samagra Shiksha in 2109-20.

Telegram: https://t.me/insightsIAStips 95 Youtube: https://www.youtube.com/channel/UCpoccbCX9GEIwaiIe4HLjwA

Revision Through MCQs (RTM) Compilation (June 2021)

Refer: https://www.insightsonindia.com/2021/06/22/nishtha-teachers-training- programme/

186. The term ‘gain of function research’ is sometimes mentioned in media in reference to: (a) Human space travel (b) Ocean thermal energy (c) Bioremediation (d) None of the above Ans: (d) Explanation: • In virology, gain-of-function research involves deliberately altering an organism in the lab, altering a gene, or introducing a mutation in a pathogen to study its transmissibility, virulence and immunogenicity. • It is believed that this allows researchers to study potential therapies, vaccine possibilities and ways to control the disease better in future. “Gain-of-function research involves manipulations that make certain pathogenic microbes more deadly or more transmissible. This is done by genetically engineering the virus and by allowing them to grow in different growth mediums, a technique called as serial passage,”

Refer: https://www.insightsonindia.com/2021/06/22/what-is-gain-of-function/

187. “Jaan Hai To Jahaan Hai” Campaign was in news recently, is launched by: (a) Ministry of Health and Family Welfare (b) Ministry of Social Justice and Empowerment (c) Ministry of Home Affairs (d) Ministry of Minority Affairs Ans: (d) Explanation: • “Jaan Hai To Jahaan Hai” Awareness Campaign: • Launched by the Ministry of Minority Affairs. • It is a nationwide awareness campaign to create awareness on Corona vaccination in rural and remote areas of the country and also to “Crush and Curb” the rumours and apprehensions regarding the on-going vaccination drive.

Refer: Facts for Prelims: https://www.insightsonindia.com/2021/06/22/insights-daily- current-affairs-pib-summary-22-june-2021/

188. Consider the following statements about Black Softshell Turtle: 1. It is a freshwater species. 2. It is listed as Endangered on the IUCN Red List. 3. In India, hunting of Black Softshell Turtle is prohibited under Schedule I of the Wildlife Protection Act of 1972. Which of the given above statements is/are correct? (a) 1 and 2 only (b) 1 only (c) 2 and 3 only (d) 1, 2 and 3 Ans: (b) Explanation: Black Softshell Turtle:

Telegram: https://t.me/insightsIAStips 96 Youtube: https://www.youtube.com/channel/UCpoccbCX9GEIwaiIe4HLjwA

Revision Through MCQs (RTM) Compilation (June 2021)

• It is a freshwater species. • International Union for Conservation of Nature had in 2021 listed the turtle as ‘critically endangered’. • It does not enjoy legal protection under the Indian Wildlife (Protection) Act of 1972. • Until sightings along the Brahmaputra River’s drainage in Assam, the black softshell turtle was thought to be ‘extinct in the wild’ and confined only to ponds of temples in northeastern India and Bangladesh.

Refer: facts for prelims: https://www.insightsonindia.com/2021/06/22/insights-daily- current-affairs-pib-summary-22-june-2021/

189. Consider the following statements about Hayagriva Madhava Temple: 1. It is situated on the Nilachal Hill in western part of Guwahati city in Assam. 2. It is one of the oldest of the 51 Shakti Pithas. Which of the given above statements is/are correct? (a) 1 only (b) 2 only (c) Both 1 and 2 (d) Neither 1 nor 2 Ans: (d) Explanation: • Why in News? • Hayagriva Madhava Temple Committee in Assam has signed a memorandum of understanding with two green NGOs, the Assam State Zoo cum Botanical Garden and the Kamrup district administration for long-term conservation of the rare freshwater black softshell turtle (Nilssonia nigricans). • Hayagriva Madhava Temple is situated on the Monikut hill. The hill is situated in Hajo of Kamrup District in Assam, India. o The present temple structure was constructed by the King Raghudeva Narayan in 1583. o According to some historians the King of Pala dynasty constructed it in 10th century. o It is a stone temple and it enshrines an image of Hayagriva Madhava. • Kamakhya Temple: o It is a Sakta temple dedicated to the mother goddess Kamakhya. o It is one of the oldest of the 51 Shakti Pithas. o Situated on the Nilachal Hill in western part of Guwahati city in Assam, India.

Refer: facts for prelims: https://www.insightsonindia.com/2021/06/22/insights-daily- current-affairs-pib-summary-22-june-2021/

190. The Genetic Engineering Appraisal Committee is constituted under the (a) Food Safety and Standards Act, 2006 (b) Geographical Indications of Goods (Registration and Protection) Act, 1999 (c) Environment (Protection) Act, 1986 (d) Wildlife (Protection) Act, 1972 Ans: (c) Explanation:

Telegram: https://t.me/insightsIAStips 97 Youtube: https://www.youtube.com/channel/UCpoccbCX9GEIwaiIe4HLjwA

Revision Through MCQs (RTM) Compilation (June 2021)

• The Genetic Engineering Appraisal Committee (GEAC) is the apex body constituted in the Ministry of Environment and Forests under ‘Rules for Manufacture, Use, Import, Export and Storage of Hazardous Microorganisms/Genetically Engineered Organisms or Cells 1989’, under the Environment Protection Act, 1986. • It is the apex body to accord notified under Rules 1989, for approval of activities involving large scale use of hazardous microorganisms and recom¬binants in research and industrial production from the environ¬mental angle. The GEAC is also responsible for ap¬proval of proposals relating to release of genetically engineered organisms and products into the environment, including experimen¬tal field trials (Biosafety Research Level trial-I and II known as BRL-I and BRL-II).

Refer: https://geacindia.gov.in/about-geac-india.aspx

RTM- REVISION THROUGH MCQS – 23rd-Jun-2021

191. The economic cost of food grains to the Food Corporation of India is Minimum Support Price and bonus (if any) paid to the farmers plus (a) transportation cost only (b) interest cost only (c) procurement incidentals and distribution cost (d) procurement incidentals and charges for godowns Ans: (c) Explanation: • The economic cost of food grains procured by the FCI is a total of MSP and bonus (if any) paid to the farmers plus the procurement incidentals and distribution cost. See https://www.indiabudget.gov.in/es2004- 05/chapt2005/chap513.pdf • The procurement incidentals are the initial costs incurred during procurement of foodgrains. The distribution costs include freight, handling charges, storage costs in godowns, losses during transit etc. • Why in news: o Centre amends Food Security rules to prevent ration leakage, corruption. o The government said that this amendment has been made as an attempt to take forward the reform process envisaged under Section 12 of the National Food Security Act (NFSA), 2013 by way of improving the transparency of the operation of the Targeted Public Distribution System (TPDS) under NFSA.

Refer: https://www.insightsonindia.com/2021/06/23/food-security-assistance-to-state- government-rules-2015-amended/

192. Suicide worldwide in 2019 report was recently released by (a) Amnesty International (b) International Association for Suicide Prevention (c) World Health Organization

Telegram: https://t.me/insightsIAStips 98 Youtube: https://www.youtube.com/channel/UCpoccbCX9GEIwaiIe4HLjwA

Revision Through MCQs (RTM) Compilation (June 2021)

(d) Walk Free Foundation Ans: (c) Explanation: • Suicide worldwide in 2019 report was recently released by World Health Organization.

Refer: https://www.insightsonindia.com/2021/06/23/suicide-worldwide-in-2019/

193. Consider the following statements regarding the Nipah virus: 1. It is a bat-borne virus that causes Nipah virus infection. 2. It can be transmitted directly from human-to-human. 3. There is no treatment or vaccine available for either people or animals. Which of the given above statements is/are correct? (a) 1 and 2 only (b) 2 and 3 only (c) 1 and 3 only (d) 1, 2 and 3 Ans: (d) Explanation: • S1: Nipah virus is a bat-borne virus that causes Nipah virus infection in humans and other animals, a disease with a high mortality rate. • S2: Nipah virus can be transmitted to humans from animals (such as bats or pigs), or contaminated foods and can also be transmitted directly from human-to-human. • S3: There is no treatment or vaccine available for either people or animals. The primary treatment for humans is supportive care.

Refer: https://www.insightsonindia.com/2021/06/23/possible-antibodies-against-nipah- virus-detected-in-bats-from-mahabaleshwar-cave/

194. Consider the following statements about the United Nations Human Rights Council: 1. It is an inter-governmental body within the United Nations system. 2. It has the ability to discuss all thematic human rights issues and situations that require its attention throughout the year. Which of the given above statements is/are correct? (a) 1 only (b) 2 only (c) Both 1 and 2 (d) Neither 1 nor 2 Ans: (c) Explanation: • The Human Rights Council is an inter-governmental body within the United Nations system made up of 47 States responsible for the promotion and protection of all human rights around the globe. • It has the ability to discuss all thematic human rights issues and situations that require its attention throughout the year. It meets at the UN Office at Geneva.

Refer: https://www.insightsonindia.com/2021/06/23/at-unhrc-grave-concerns-raised- over-xinjiang/

Telegram: https://t.me/insightsIAStips 99 Youtube: https://www.youtube.com/channel/UCpoccbCX9GEIwaiIe4HLjwA

Revision Through MCQs (RTM) Compilation (June 2021)

195. Consider the following statements: 1. The Great Barrier Reef is the largest coral reef system in the world. 2. The reef is located in the Solomon Sea. Which of the given above statements is/are correct? (a) 1 only (b) 2 only (c) Both 1 and 2 (d) Neither 1 nor 2 Ans: (a) Explanation: • The Great Barrier Reef is the largest coral reef system in the world. The reef is located off the coast of Queensland, Australia, in the Coral Sea.

• Refer: https://www.insightsonindia.com/2021/06/23/unesco-to-downgrade-status-of- great-barrier-reef-australia-blames-political-interference/

196. Consider the following pairs: Places in news Country 1. City of Bam Australia 2. Xinjiang China 3. City of Budapest Austria Which of the given above pairs is/are correct? (a) 1 and 2 only (b) 2 only (c) 2 and 3 only (d) 1 and 3 only Ans: (b) Explanation: • From Australia's Great Barrier Reef to the city of Budapest [ Hungary’s capital, is bisected by the River Danube] and the watery wonderland that is Venice, several beauty spots risk losing their treasured UN World Heritage status because of environmental damage, excess development or overtourism.

Refer: https://www.insightsonindia.com/2021/06/23/unesco-to-downgrade-status-of- great-barrier-reef-australia-blames-political-interference/

197. Consider the following statements about the Land for Life Award: 1. It is the United Nation's highest environmental honour. 2. It is organised by UNCCD every two years.

Telegram: https://t.me/insightsIAStips 100 Youtube: https://www.youtube.com/channel/UCpoccbCX9GEIwaiIe4HLjwA

Revision Through MCQs (RTM) Compilation (June 2021)

Which of the given above statements is/are correct? (a) 1 only (b) 2 only (c) Both 1 and 2 (d) Neither 1 nor 2 Ans: (b) Explanation: • About the Land for Life Award: o Launched at the UNCCD COP (Conference of Parties) 10 in 2011. o It is considered as the world’s highest reward regarding land conservation and restoration. o It is organised by UNCCD every two years. o This year’s theme was “Healthy Land, Healthy Lives”. • The Champions of the Earth award o It is the United Nation’s highest environmental honour. o The United Nations Environment Programme established Champions of the Earth in 2005 as an annual awards programme to recognize outstanding environmental leaders from the public and private sectors, and from civil society.

Refer: https://www.insightsonindia.com/2021/06/23/un-land-conservation-award/

198. Consider the following statements about United Nations Convention to Combat Desertification: 1. It is the sole legally binding international agreement linking environment and development to sustainable land management. 2. The UNCCD Secretariat is administered by World Bank. Which of the given above statements is/are correct? (a) 1 only (b) 2 only (c) Both 1 and 2 (d) Neither 1 nor 2 Ans: (a) Explanation: • About United Nations Convention to Combat Desertification: o Established in 1994. o It is the sole legally binding international agreement linking environment and development to sustainable land management under the UN. o It is the only convention stemming from a direct recommendation of the Rio Conference’s Agenda 21. o Focus areas: The Convention addresses specifically the arid, semi-arid and dry sub-humid areas, known as the drylands, where some of the most vulnerable ecosystems and peoples can be found. • S2: The permanent secretariat of the Convention was established in Article 23 of the UNCCD. It has been located in Bonn, Germany since January 1999.

Refer: https://www.insightsonindia.com/2021/06/23/un-land-conservation-award/

Telegram: https://t.me/insightsIAStips 101 Youtube: https://www.youtube.com/channel/UCpoccbCX9GEIwaiIe4HLjwA

Revision Through MCQs (RTM) Compilation (June 2021)

199. Consider the following statements: 1. Tea grows well in hot and humid climate. 2. Kerala is the largest producer of tea in India. 3. Tea Board of India is a statutory body under the Ministry of Commerce. Which of the given above statements is/are correct? (a) 1 only (b) 2 and 3 only (c) 1 and 3 only (d) 1, 2 and 3 Ans: (c) Explanation: • S1: Tea is a tropical and sub-tropical plant and grows well in hot and humid climate. • S2: West Bengal is the largest producer of tea in India.

o • S3: Tea Board of India is a statutory body under the Ministry of Commerce.

Refer: facts for prelims: https://www.insightsonindia.com/2021/06/23/insights-daily- current-affairs-pib-summary-23-june-2021/

200. Consider the following statements: 1. Natural rubber is a native of warm humid Amazon forests. 2. Natural rubber is naturally suited for the colder conditions in the Northeast. 3. Assam is the largest producers of rubber in India. Which of the given above statements is/are correct? (a) 1 only (b) 2 and 3 only (c) 1 and 2 only (d) 1, 2 and 3 Ans: (a) Explanation: • Natural rubber is a native of warm humid Amazon forests and is not naturally suited for the colder conditions in the Northeast, which is one of the largest producers of rubber in India. • Kerala accounts for more than 90 percent of the total rubber production in the country. • World’s 1st GM rubber sapling:

Telegram: https://t.me/insightsIAStips 102 Youtube: https://www.youtube.com/channel/UCpoccbCX9GEIwaiIe4HLjwA

Revision Through MCQs (RTM) Compilation (June 2021)

o World’s first genetically modified (GM) rubber sapling was recently planted at the Rubber Board’s Sarutari research farm on the outskirts of Guwahati in Assam. o It was developed at the Kerala-based Rubber Research Institute of India (RRII). o With additional copies of the gene MnSOD (manganese-containing superoxide dismutase) inserted in it, the GM rubber is expected to tide over the severe cold conditions during winter, which is a major factor affecting the growth of rubber saplings. o MnSOD gene used in the GM rubber was taken from the rubber plant itself.

Refer: facts for prelims: https://www.insightsonindia.com/2021/06/23/insights-daily- current-affairs-pib-summary-23-june-2021/

Telegram: https://t.me/insightsIAStips 103 Youtube: https://www.youtube.com/channel/UCpoccbCX9GEIwaiIe4HLjwA

Revision Through MCQs (RTM) Compilation (June 2021)

RTM- REVISION THROUGH MCQS – 24th -Jun-2021

201. With reference to first-past-the-post (FPTP) voting method, consider the following statements: 1. It is also known as the simple majority system. 2. In this voting method, the candidate with the highest number of votes in a constituency is declared the winner. 3. This system is used in India in direct elections to the Lok Sabha and State Legislative Assemblies. Which of the given above statements is/are correct? (a) 1 and 2 only (b) 2 and 3 only (c) 1 and 3 only (d) 1, 2 and 3 Ans: (d) Explanation: • The first-past-the-post (FPTP) system is also known as the simple majority system. • In this voting method, the candidate with the highest number of votes in a constituency is declared the winner. • This system is used in India in direct elections to the Lok Sabha and State Legislative Assemblies. • While FPTP is relatively simple, it does not always allow for a truly representative mandate, as the candidate could win despite securing less than half the votes in a contest. In 2014, the National Democratic Alliance led by the Bharatiya Janata Party won 336 seats with only 38.5% of the popular vote. Also, smaller parties representing specific groups have a lower chance of being elected in FPTP.

Refer: https://www.insightsonindia.com/2021/06/24/what-is-ranked-choice-voting- which-made-its-debut-in-new-york-mayoral-polls/

202. Consider the following statements about ‘Information Fusion Centre for Indian Ocean Region’ (IFC-IOR): 1. It is the single point center linking all the coastal radar chains networks along the Indian coastline and in some neighbouring countries. 2. It will serve countries that have white (commercial ships) and Grey (military vessels) Shipping Information Exchange agreements with India. Which of the given above statements is/are correct? (a) 1 only (b) 2 only (c) Both 1 and 2 (d) Neither 1 nor 2 Ans: (a) Explanation: • The IFC-IOR was set up in 2018 to coordinate with regional countries on maritime issues and act as a regional repository of maritime data. • It presently has linkages with 21 partner countries and 22 multi-national agencies across the globe.

Telegram: https://t.me/insightsIAStips 104 Youtube: https://www.youtube.com/channel/UCpoccbCX9GEIwaiIe4HLjwA

Revision Through MCQs (RTM) Compilation (June 2021)

• It is located in Gurugram, India. • The Information Fusion Centre will serve countries that have White Shipping Information Exchange agreements with India. o India already has bilateral White Shipping Agreements with 36 countries.

Refer: https://www.insightsonindia.com/2021/06/24/information-fusion-centre-for- indian-ocean-region-ifc-ior/

203. Recently, which one of the following nation’s ‘Tax Inspectors without Borders’ (TIWB) programme launched in partnership with India? (a) Nepal (b) Pakistan (c) Bhutan (d) Myanmar Ans: (c) Explanation: • Bhutan’s Tax Inspectors Without Borders (TIWB) programme launched in partnership with India. o This programme is expected to be of about 24 months’ duration. o The focus of the programme will be in the area of International Taxation and Transfer Pricing.

Refer: https://www.insightsonindia.com/2021/06/24/tax-inspectors-without-borders- tiwb-programme/

204. The ‘Tax Inspectors Without Borders’ (TIWB) programme is a joint initiative of: 1. The United Nations Development Programme (UNDP) 2. The Organisation for Economic Cooperation and Development (OECD) 3. The World Customs Organization (WCO) Select the correct answer using the code below: (a) 1 and 2 only (b) 2 and 3 only (c) 1 and 3 only (d) 1, 2 and 3 Ans: (a) Explanation: About Tax Inspectors Without Borders (TIWB) programme: • It is a joint initiative of the United Nations Development Programme (UNDP) and the Organisation for Economic Cooperation and Development (OECD). • The objective of the TIWB Initiative is to enable sharing of tax audit knowledge and skills with tax administrations in developing countries through a targeted, real time “learning by doing” approach. • TIWB is focused on promoting hands-on assistance by sending Experts to build audit and audit-related skills pertaining to specific international tax matters and the development of general audit skills within developing tax administrations.

Refer: https://www.insightsonindia.com/2021/06/24/tax-inspectors-without-borders- tiwb-programme/

Telegram: https://t.me/insightsIAStips 105 Youtube: https://www.youtube.com/channel/UCpoccbCX9GEIwaiIe4HLjwA

Revision Through MCQs (RTM) Compilation (June 2021)

205. Consider the following statements about Competition Commission Of India: 1. It is a constitutional body of the Government of India. 2. It consists of a Chairperson and not more than ten Members appointed by the Central Government. Which of the given above statements is/are correct? (a) 1 only (b) 2 only (c) Both 1 and 2 (d) Neither 1 nor 2 Ans: (d) Explanation: • Competition Commission of India is the competition regulator in India. • It is a statutory body of the Government of India responsible for enforcing The Competition Act, 2002 and promoting competition throughout India and to prevent activities that have an appreciable adverse effect on competition in India. • CCI consists of a Chairperson and 6 Members appointed by the Central Government.

Refer: https://www.insightsonindia.com/2021/06/24/cci-probes-google-for-unfair- business-practices/

206. Consider the following statements about the Scientific Advisory Board of the United Nations Secretary-General: 1. It is an intergovernmental body of the United Nations. 2. It is composed of representatives appointed by governments. 3. It mainly produces reports that contribute to the work of the United Nations Framework Convention on Climate Change (UNFCCC). Which of the given above statements is/are not correct? (a) 1 and 3 only (b) 1 only (c) 2 and 3 only (d) 1, 2 and 3 Ans: (d) Explanation: here the directive word is not correct!! • S1: The UN Secretary-General announced the creation of the Scientific Advisory Board on 24 September 2013, during the inaugural meeting of the High-Level Political Forum on Sustainable Development. • S2: It will comprise renowned scientists representing various fields of natural, social and human sciences. • S3: The central function of the Board will be to provide advice on science, technology and innovation (STI) for sustainable development to the UN Secretary-General and to Executive Heads of UN organizations.

Refer: https://www.insightsonindia.com/2021/06/24/climate-crisis-to-hit-sooner-than- feared/

207. Consider the following statements: 1. The Suez Canal is a natural sea-level waterway running north to south across the Isthmus of Suez. 2. The Canal connects the and the Red Sea.

Telegram: https://t.me/insightsIAStips 106 Youtube: https://www.youtube.com/channel/UCpoccbCX9GEIwaiIe4HLjwA

Revision Through MCQs (RTM) Compilation (June 2021)

Which of the given above statements is/are correct? (a) 1 only (b) 2 only (c) Both 1 and 2 (d) Neither 1 nor 2 Ans: (b) Explanation: • The Suez Canal is an artificial sea-level waterway running north to south across the Isthmus of Suez in Egypt, to connect the Mediterranean Sea and the Red Sea. • The canal separates the African continent from Asia.

• Refer: facts for prelims: https://www.insightsonindia.com/2021/06/24/insights-daily- current-affairs-pib-summary-24-june-2021/

208. Consider the following statements about Pygmy hog (Porcula salvania): 1. It is endemic to the Western Ghats of India. 2. It is listed as ‘Critically Endangered’ on the IUCN Red List of Threatened Species. 3. It is one of the very few mammals that build its own home. Which of the given above statements is/are correct? (a) 1 and 2 only (b) 2 and 3 only (c) 1 and 3 only (d) 1, 2 and 3 Ans: (b) Explanation: About Pygmy hogs: • They are the world’s rarest and smallest wild pigs. • S1: The pygmy hog is native to dense alluvial grasslands in the southern foothills of the Himalayas. o Endemic to India, they are restricted to very few locations around Manas National Park in north-western Assam. • With just around 250 animals in the wild, the pygmy hog is one of the world’s most threatened mammals. • S2: Currently listed as ‘Critically Endangered’ on the IUCN Red List of Threatened Species.

Telegram: https://t.me/insightsIAStips 107 Youtube: https://www.youtube.com/channel/UCpoccbCX9GEIwaiIe4HLjwA

Revision Through MCQs (RTM) Compilation (June 2021)

• The pygmy hog is designated as a Schedule I species in India under the Wildlife Protection Act, 1972. • S3: The pygmy hog is one of the very few mammals that build its own home, or nest, complete with a ‘roof’, but maybe more important than its own attributes, is the fact that it is an indicator species. Its presence reflects the health of its primary habitat, the tall, wet grasslands of the region

Refer: facts for prelims: https://www.insightsonindia.com/2021/06/24/insights-daily- current-affairs-pib-summary-24-june-2021/

209. In which one of the following States is Barnadi Wildlife Sanctuary located? (a) Assam (b) Odisha (c) Nagaland (d) Kerala Ans: (a) Explanation: • The smallest member of the pig family, the pygmy hog (porcula salvania), is a critically endangered species. Once found along a narrow strip of tall and wet grassland plains on the Himalayan foothills – from Uttar Pradesh to Assam, through Nepal’s terai areas and Bengal’s duars – it was thought to have become extinct in the 1960s. But in 1971 it was “re-discovered” with a small population in the Barnadi Wildlife Sanctuary. o It is situated on the foothills of Himalayas bordering Bhutan in the north and in Udalguri district & Baksa District of Assam. This sanctuary is named after the river Bornadi which flows on its western border.

Refer: facts for prelims: https://www.insightsonindia.com/2021/06/24/insights-daily- current-affairs-pib-summary-24-june-2021/

210. With reference to India’s Manas National Park, consider the following statements: 1. It is spread over four districts. 2. It is one of the natural habitats of red giant flying squirrel. 3. It has the largest population of the endangered Bengal florican to be found anywhere in India. Which of the given above statements is/are correct? (a) 1 and 2 only (b) 3 only (c) 2 and 3 only (d) 1, 2 and 3 Ans: (b) Explanation: • S1: The park area falls in the following districts: Chirang, Baksa in the autonomous territorial region, i.e BTR in the state of Assam in India. • S2: The red giant flying squirrel is often seen in Namdapha National Park. o It is endemic to the park and critically endangered. • S3: It has the largest population of the critically endangered Bengal florican to be found anywhere.

Telegram: https://t.me/insightsIAStips 108 Youtube: https://www.youtube.com/channel/UCpoccbCX9GEIwaiIe4HLjwA

Revision Through MCQs (RTM) Compilation (June 2021)

Refer: facts for prelims: https://www.insightsonindia.com/2021/06/24/insights-daily- current-affairs-pib-summary-24-june-2021/

RTM- REVISION THROUGH MCQS – 24th -Jun-2021

211. Consider the following statements: 1. The Electoral Trusts Scheme, 2013 was notified by the Election Commission of India. 2. The sole object of the electoral trust is to distribute funds only to the eligible political parties in India. Which of the given above statements is/are correct? (a) 1 only (b) 2 only (c) Both 1 and 2 (d) Neither 1 nor 2 Ans: (b) Explanation: About Electoral Trusts Scheme, 2013: • Electoral Trust is a non-profit organization formed in India for orderly receiving of the contributions from any person. • The scheme was notified by the Central Board of Direct Taxes (CBDT). • Objectives of the Scheme: To lay down a procedure for grant of approval to an electoral trust which will receive voluntary contributions and distribute the same to the political parties. • The sole object of the electoral trust is to distribute the contributions received by it to the political party, registered under section 29A of the Representation of the People Act, 1951. • These Electoral Trust companies are not allowed to accept contributions from foreign citizens or companies. • The trust shall also maintain a list of persons from whom contributions have been received and to whom the same have been distributed.

Refer: https://www.insightsonindia.com/2021/06/25/electoral-trust-declares-donation-of- rs-3-crore-through-electoral-bonds/

212. Mission Karmayogi was in news recently, is related to which of the following? (a) Agriculture Reforms (b) Women empowerment (c) Socio-Economic empowerment of tribal people (d) Reform in Indian Bureaucracy Ans: (d) Explanation: About Mission Karmayogi: • The ‘National Programme for Civil Services Capacity Building – Mission Karmayogi’ was launched to effect a transformational shift from rule based training to role-based capacity development of all civil services in the country so as to enhance citizen experience for government services and improve availability of competent workforce.

Refer: https://www.insightsonindia.com/2021/06/25/task-force-for-mission-karmayogi/

Telegram: https://t.me/insightsIAStips 109 Youtube: https://www.youtube.com/channel/UCpoccbCX9GEIwaiIe4HLjwA

Revision Through MCQs (RTM) Compilation (June 2021)

213. In the context of India which one of the following is the characteristic appropriate for Bureaucracy? (a) An agency for widening the scope for parliamentary democracy (b) An agency for strengthening the structure of federalism (c) An agency for facilitating the political stability and economic growth (d) An agency for the implementation of public policy Ans: (d) Explanation: • Public policy formulation and policy implementation are two distinct but closely interrelated functions of the government. Public policy is laid down by the legislature or the political authorities, who are vested with the power of giving policy the requisite legal authority i.e. legitimacy. The policy implementation aspect is supposed to be in the domain of the executive, i.e., the bureaucracy or the administrative arm of the government. • Read more>>

Refer: https://www.insightsonindia.com/2021/06/25/task-force-for-mission-karmayogi/

214. Consider the following statements: 1. Peter Pan syndrome is a mental health condition that can affect one’s quality of life. 2. The World Health Organization does not recognise Peter Pan Syndrome as a health disorder. Which of the given above statements is/are correct? (a) 1 only (b) 2 only (c) Both 1 and 2 (d) Neither 1 nor 2 Ans: (c) Explanation: what is Peter Pan syndrome? • The syndrome is named after a 1900s fictional character. Peter Pan is a care- free young boy, who never grows up. The character was created by Scottish novelist James Matthew Barrie. • It is said that people who develop similar behaviours — of living life carefree, finding responsibilities challenging in adulthood, and basically, “never growing up” — suffer from Peter Pan Syndrome. • Please note that this syndrome has not been recognised by WHO as a health disorder. • It affects people who do not want or feel unable to grow up, people with the body of an adult but the mind of a child. They don’t know how to or don’t want to stop being children and start being mothers or fathers.

Refer: https://www.insightsonindia.com/2021/06/25/peter-pan-syndrome-pps/

215. Consider the following statements about Shanghai Cooperation Organisation (SCO): 1. It is an intergovernmental military alliance. 2. China is the supreme decision-making body in the SCO. 3. The SCO Secretariat is located in Tashkent. Which of the given above statement is/are not correct? (a) 1 and 2 only (b) 3 only

Telegram: https://t.me/insightsIAStips 110 Youtube: https://www.youtube.com/channel/UCpoccbCX9GEIwaiIe4HLjwA

Revision Through MCQs (RTM) Compilation (June 2021)

(c) 2 and 3 only (d) 1, 2 and 3 Ans: (d) Explanation: here the directive word is not correct!! • S1: the SCO has repeatedly stressed that it is not a military alliance with treaty obligations. o It is a permanent intergovernmental international organization. • S2: The Heads of State Council (HSC) is the supreme decision-making body in the SCO. It meets once a year and adopts decisions and guidelines on all important matters of the organisation. • S3: The organisation has two permanent bodies — the SCO Secretariat based in Beijing and the Executive Committee of the Regional Anti-Terrorist Structure (RATS) based in Tashkent.

Refer: https://www.insightsonindia.com/2021/06/25/sco-meet/

216. With reference to India’s Chandrayaan-2 mission, consider the following statements: 1. It comprised an Orbiter, Lander and Rover to explore the unexplored North Pole of the Moon. 2. The primary objective of mission was to demonstrate the ability to soft-land on the lunar surface and operate a robotic rover on the surface. Which of the given above statements is/are correct? (a) 1 only (b) 2 only (c) Both 1 and 2 (d) Neither 1 nor 2 Ans: (b) Explanation: • S1: It comprised an Orbiter, Lander and Rover to explore the unexplored South Pole of the Moon. • S2: The primary objective of Chandrayaan 2 was to demonstrate the ability to soft-land on the lunar surface and operate a robotic rover on the surface.

Refer: https://www.insightsonindia.com/2021/06/25/chandrayaan-2-5/

217. Consider the following statements about the Antarctic Treaty: 1. The primary purpose of the Antarctic Treaty is to completely prohibit military activities in the Antarctic region. 2. The Treaty applies to the area south of 60° South Latitude, including all ice shelves and islands. Which of the given above statements is/are correct? (a) 1 only (b) 2 only (c) Both 1 and 2 (d) Neither 1 nor 2 Ans: (b) Explanation: • The primary purpose of the Antarctic Treaty is to ensure "in the interests of all mankind that Antarctica shall continue forever to be used exclusively for peaceful purposes and shall not become the scene or object of international

Telegram: https://t.me/insightsIAStips 111 Youtube: https://www.youtube.com/channel/UCpoccbCX9GEIwaiIe4HLjwA

Revision Through MCQs (RTM) Compilation (June 2021)

discord." To this end it prohibits military activity, except in support of science; prohibits nuclear explosions and the disposal of nuclear waste; promotes scientific research and the exchange of data; and holds all territorial claims in abeyance. The Treaty applies to the area south of 60° South Latitude, including all ice shelves and islands.

Refer: https://www.insightsonindia.com/2021/06/25/antarctic-treaty/

218. Consider the following statements: 1. The Black Sea is bordered by Bulgaria, Georgia, Romania, Russia, Turkey, and Ukraine. 2. The Black Sea ultimately drains into the Mediterranean Sea, via the Strait of Kerch and the Aegean Sea. Which of the given above statements is/are correct? (a) 1 only (b) 2 only (c) Both 1 and 2 (d) Neither 1 nor 2 Ans: (a) Explanation; • S1: The Black Sea is bordered by Bulgaria, Georgia, Romania, Russia, Turkey, and Ukraine. • S2: The Black Sea ultimately drains into the Mediterranean Sea, via the Turkish Straits and the Aegean Sea. The Bosporus Strait connects it to the small Sea of Marmara which in turn is connected to the Aegean Sea via the Strait of the Dardanelles. To the north, the Black Sea is connected to the Sea of Azov by the Kerch Strait.

• Refer: Facts for Prelims: https://www.insightsonindia.com/2021/06/25/insights-daily- current-affairs-pib-summary-25-june-2021/

219. Consider the following pairs 1. Nokrek Bio-sphere Reserve: Garo Hills 2. Loktak Lake: Barail Range 3. Namdapha National Park: Dafla Hills Which of the above pairs is/are correctly matched? (a) 1 only (b) 2 and 3 only (c) 1, 2 and 3 (d) None Ans: (a) Explanation:

Telegram: https://t.me/insightsIAStips 112 Youtube: https://www.youtube.com/channel/UCpoccbCX9GEIwaiIe4HLjwA

Revision Through MCQs (RTM) Compilation (June 2021)

• Nokrek National Park, the core area of Nokrek Biosphere Reserve, is a national park located approximately 2 km from Tura Peak in West Garo Hills district of Meghalaya, India. UNESCO added this National park to its list of Biosphere Reserves in May 2009. • Logtak Lake is in Manipur; Barail Range is in Assam. • Though Namdapha National Park and Dafla Hill both are in Arunachal Pradesh, the two are separate entities.

Refer: http://www.megforest.gov.in/wildlife_parks.html

220. Garibi hatao slogan was introduced in which of the following Five Year Plan? (a) Fourth (b) Fifth (c) Second (d) First Ans: (b) Explanation: • Garibi Hatao (“Remove poverty”) was the theme and slogan of Indira Gandhi’s 1971 election campaign. • The slogan and the proposed anti-poverty programs that came with it were designed to give Gandhi an independent national support, based on rural and urban poor, which would allow her to by-pass the dominant rural castes both in and out of state and local government. • The programs created through garibi hatao, though carried out locally, were funded, developed, supervised, and staffed by Government officials in New Delhi and Congress Party officials. It was part of the 5th Five-Year Plan.

RTM- REVISION THROUGH MCQS – 26th -Jun-2021

221. With reference to cultural history of India, consider the following statements : 1. Kabir Das' writings had a great influence on the Bhakti movement. 2. The major part of Kabir’s work was collected by the Guru Hargobind. 3. Kabir Das’ ideologies were greatly influenced by Vaishnava saint Swami Ramananda. 4. Kabir Das and Data Ganj Bakhsh are contemporaries. Which of the statements given above are correct? (a) 1 and 3 only (b) 2 and 4 only (c) 1, 2 and 3 (d) 1, 2, 3 and 4 Ans: (a) Explanation: • S1 and S2: Kabir Das' writings had a great influence on the Bhakti movement and includes titles like Kabir Granthawali, Anurag Sagar, Bijak, and Sakhi Granth. The major part of his work was collected by the fifth Sikh guru--Guru Arjan Dev. • S3: Swami Ramananda influence: Kabir Das’ ideologies were greatly influenced by Vaishnava saint Swami Ramananda who accepted Kabir as his disciple.

Telegram: https://t.me/insightsIAStips 113 Youtube: https://www.youtube.com/channel/UCpoccbCX9GEIwaiIe4HLjwA

Revision Through MCQs (RTM) Compilation (June 2021)

• S4: Data Ganj Bakhsh was an 11th-century Persian Sunni Muslim mystic and theologian.

Refer: https://www.insightsonindia.com/2021/06/26/sant-kabir-das-jayanti/

222. Currently, the “Enforcement Directorate” is operating under the jurisdiction of the: (a) Ministry of Home Affairs (b) Ministry of Parliamentary Affairs (c) Ministry of Finance (d) Ministry of Personnel, Public Grievances and Pensions Ans: (c) Explanation: About Enforcement Directorate: • The origin of this Directorate goes back to 1st May, 1956, when an ‘Enforcement Unit’ was formed, in the Department of Economic Affairs, for handling Exchange Control Laws violations under Foreign Exchange Regulation Act, 1947 (FERA ’47). • In the year 1957, this Unit was renamed as ‘Enforcement Directorate’. • Currently, it is part of the Department of Revenue, Ministry of Finance. • The Organization is mandated with the task of enforcing the provisions of two special fiscal laws – Foreign Exchange Management Act, 1999 (FEMA) and Prevention of Money Laundering Act, 2002 (PMLA).

Refer: https://www.insightsonindia.com/2021/06/26/ed-transfers-assets-worth- %e2%82%b98441-50-cr-to-banks/

223. Consider the following statements about ‘Gujarat International Finance Tec-City’ (GIFT City): 1. It is a business district near Ahmedabad in Gujarat, India. 2. It is India's first operational Greenfield smart city and international financial services centre. 3. The city is located on the banks of the Sabarmati River. Which of the given above statements is/are correct? (a) 1 and 2 only (b) 1 and 3 only (c) 2 and 3 only (d) 1, 2 and 3 Ans: (d) Explanation: • S1 and S2: It is a business district near Ahmedabad in Gujarat, India. It is India's first operational Greenfield smart city and international financial services center. • S3: The city is located on the banks of the Sabarmati River.

Refer: https://www.insightsonindia.com/2021/06/26/gujarat-international-maritime- arbitration-centre-gimac/

224. Consider the following statements about the International Labour Organisation (ILO): 1. It was established as an agency for the League of Nations following World War I. 2. It became the first specialised agency of the United Nations (UN) in the year 1946. 3. It is the only tripartite U.N. agency.

Telegram: https://t.me/insightsIAStips 114 Youtube: https://www.youtube.com/channel/UCpoccbCX9GEIwaiIe4HLjwA

Revision Through MCQs (RTM) Compilation (June 2021)

Which of the given above statements is/are correct? (a) 1 and 2 only (b) 2 and 3 only (c) 1 and 3 only (d) 1, 2 and 3 Ans: (d) Explanation: • Established as an agency for the League of Nations following World War I. • Established by the Treaty of Versailles in 1919. • It became the first specialised agency of the United Nations (UN) in the year 1946. • It got the Nobel Peace Prize in 1969. • It is the only tripartite U.N. agency. It brings together governments, employers and workers. • Headquarters: Geneva, Switzerland.

Refer: https://www.insightsonindia.com/2021/06/26/india-completes-term-as-chair-of- ilos-governing-body/

225. Consider the following statements about Financial Action Task Force (FATF): 1. It is an inter-governmental body established in 1989 on the initiative of the Organisation for Economic Co-operation and Development (OECD). 2. Initially it was established to examine and develop measures to combat terrorist financing. Which of the given above statements is/are correct? (a) 1 only (b) 2 only (c) Both 1 and 2 (d) Neither 1 nor 2 Ans: (d) Explanation: • S1: The Financial Action Task Force (FATF) is an inter-governmental body established in 1989 on the initiative of the G7. • S2: Initially it was established to examine and develop measures to combat money laundering. o In October 2001, the FATF expanded its mandate to incorporate efforts to combat terrorist financing, in addition to money laundering. o In April 2012, it added efforts to counter the financing of proliferation of weapons of mass destruction.

Refer: https://www.insightsonindia.com/2021/06/26/pakistan-to-remain-on-fatf-grey-list- 3/

226. Consider the following statements regarding ‘Maritime State Development Council’ (MSDC): 1. It is an apex advisory body for the development of the Maritime sector in India. 2. It monitors the development of minor ports, captive ports and private ports in the Maritime States. Which of the given above statement is/are correct? (a) 1 only

Telegram: https://t.me/insightsIAStips 115 Youtube: https://www.youtube.com/channel/UCpoccbCX9GEIwaiIe4HLjwA

Revision Through MCQs (RTM) Compilation (June 2021)

(b) 2 only (c) Both 1 and 2 (d) Neither 1 nor 2 Ans: (c) Explanation: • MSDC is an apex advisory body for the development of the Maritime sector and aims to ensure integrated development of Major and non-Major Ports. The MSDC was constituted in May 1997 to assess in consultation with State Governments, the future development of existing and new Minor Ports by the respective Maritime States either directly or through captive users and private participation. Further, MSDC also monitors the development of minor ports, captive ports and private ports in the Maritime States with a view to ensure their integrated development with Major Ports and to assess the requirements of other infrastructure requirements like roads/rail/IWT and make suitable recommendations to the concerned Ministers.

Refer: Facts for Prelims: https://www.insightsonindia.com/2021/06/26/insights-daily- current-affairs-pib-summary-26-june-2021/

227. The ‘Project Seabird’ was in news recently, is related to which of the following? (a) ship breaking/recycling (b) construction of naval base (c) indigenous submarine construction (d) deep ocean exploration Ans: (b) Explanation: Project Seabird: • It is the largest naval infrastructure project for India. • The project aims at creation of a naval base at Karwar on the west coast of India.

Refer: Facts for Prelims: https://www.insightsonindia.com/2021/06/26/insights-daily- current-affairs-pib-summary-26-june-2021/

228. Which of the following is geographically closest to Arunachal Pradesh? (a) Lhasa (b) Nyingchi (c) Shigatse (d) Nagqu Ans: (b) Explanation: • China has operationalised its first fully electrified bullet train in the remote Himalayan region of Tibet. • It connects the provincial capital Lhasa and Nyingchi, a strategically located Tibetan border town close to Arunachal Pradesh. • This line is a section of the Sichuan-Tibet Railway. • The rail line crosses the Brahmaputra river locally called Yarlung Zangbo 16 times.

Telegram: https://t.me/insightsIAStips 116 Youtube: https://www.youtube.com/channel/UCpoccbCX9GEIwaiIe4HLjwA

Revision Through MCQs (RTM) Compilation (June 2021)

• Refer: Facts for Prelims: https://www.insightsonindia.com/2021/06/26/insights-daily- current-affairs-pib-summary-26-june-2021/

229. Consider the following statements with reference to river Brahmaputra, 1. The source of river Brahmaputra is the same as that of Teesta but it flows through Assam. 2. It flows as Jamuna River before emptying into the Bay of Bengal. 3. The ‘Zayu River’ is a tributary of Brahmaputra River. Which of the statements given above is/are correct? (a) 2 only (b) 1 and 2 only (c) 3 only (d) 1, 2 and 3 Ans: (c) Explanation: • S1: Teesta River originates from the Pahunri glacier. Brahmaputra originates in Angsi glacier/ Chemayungdung Glacier. Teesta is a tributary of Brahmaputra. • S2: It flows southwest through the Assam Valley as Brahmaputra and south through Bangladesh as the Jamuna. In the vast Ganges Delta, it merges with the Padma, the popular name of the river Ganges in Bangladesh, and finally, after merging with Padma, it becomes the Meghna and from here, it flows as Meghna river before emptying into the Bay of Bengal. • S3: The Lohit River or Zayu River is a river in India and China. o It is a tributary to the Brahmaputra River. o The river rises in Tibet Autonomous Region, in the Kangri Garpo range, where it is known as the Zayü River.

• Refer: Facts for Prelims: https://www.insightsonindia.com/2021/06/26/insights-daily- current-affairs-pib-summary-26-june-2021/

Telegram: https://t.me/insightsIAStips 117 Youtube: https://www.youtube.com/channel/UCpoccbCX9GEIwaiIe4HLjwA

Revision Through MCQs (RTM) Compilation (June 2021)

230. ‘Maguri Motapung Beel Wetland’ is located in: (a) Arunachal Pradesh (b) Assam (c) Meghalaya (d) Manipur Ans: (b) Explanation: • Maguri Motapung Beel is less than 10 km south of the more famous Dibru- Saikhowa National Park and part of the Dibru-Saikhowa Biosphere Reserve. • The wetland derives its name from ‘Magur’, local word for the catfish Clarius batrachus, once found here in abundance. Motapung is a village nearby, and Beel is the Assamese word for wetland.

Refer: Facts for Prelims: https://www.insightsonindia.com/2020/07/20/insights-daily- current-affairs-pib-summary-20-july-2020/

RTM- REVISION THROUGH MCQS – 28th -Jun-2021

231. With reference to India’s Smart Cities mission, consider the following statements: 1. The Mission is operated as a Centrally Sponsored Scheme. 2. The Union Ministry of Urban Development is responsible for implementing the mission in collaboration with the state governments. Which of the given above statements is/are correct? (a) 1 only (b) 2 only (c) Both 1 and 2 (d) Neither 1 nor 2 Ans: (c) Explanation: • S1: It is a Centrally Sponsored Scheme. • S2: The Union Ministry of Urban Development is responsible for implementing the mission in collaboration with the state governments of the respective cities.

Refer: https://www.insightsonindia.com/2021/06/28/india-smart-cities-awards-2020/

232. Consider the following statements about Pradhan Mantri Awas Yojana (URBAN): 1. It is a flagship Mission of Government of India being implemented by Ministry of Housing and Urban Affairs (MoHUA). 2. It covers the entire urban area consisting of Statutory Towns and Notified Planning Areas except Special Area Development Authorities. Which of the given above statements is/are correct? (a) 1 only (b) 2 only (c) Both 1 and 2 (d) Neither 1 nor 2 Ans: (a)

Telegram: https://t.me/insightsIAStips 118 Youtube: https://www.youtube.com/channel/UCpoccbCX9GEIwaiIe4HLjwA

Revision Through MCQs (RTM) Compilation (June 2021)

Explanation: • S1: Pradhan Mantri Awas Yojana – Urban (PMAY-U), a flagship Mission of Government of India being implemented by Ministry of Housing and Urban Affairs (MoHUA), was launched on 25th June 2015. • S2: The Mission covers the entire urban area consisting of Statutory Towns, Notified Planning Areas, Development Authorities, Special Area Development Authorities, Industrial Development Authorities or any such authority under State legislation which is entrusted with the functions of urban planning & regulations.

Refer: https://www.insightsonindia.com/2021/06/28/pradhan-mantri-awas-yojana- urban-7/

233. The Gujarat Prohibition Act (1949) was in news recently, is related to which of the following? (a) prevention of begging in the State (b) promotion and enforcement of alcohol prohibition in the State (c) secure exclusive state control over forests (d) none of the above Ans: (b) Explanation: The Gujarat Prohibition Act, 1949: • Introduced by the then Bombay province as Bombay Prohibition Act, 1949 to overhaul the law relating to intoxicating drugs and narcotics total prohibition. • It is an Act relating to the promotion and enforcement of alcohol prohibition in the Bombay State. • The Bombay state was divided into the states of Maharashtra and Gujarat in 1960. • Gujarat adopted the prohibition policy in 1960 and subsequently chose to enforce it with greater rigidity. • In 2011, it renamed the Act as Gujarat Prohibition Act.

Refer: https://www.insightsonindia.com/2021/06/28/the-gujarat-prohibition-act-1949/

234. Consider the following statements about African Swine Fever (ASF): 1. The organism which causes ASF is a DNA virus of the Asfarviridae family. 2. It is not a threat to human health and cannot be transmitted from pigs to humans. 3. As of now, there is no approved vaccine. Which of the given above statements is/are correct? (a) 1 and 3 only (b) 2 and 3 only (c) 1 and 2 only (d) 1, 2 and 3 Ans: (d) Explanation: • S1: The organism which causes ASF is a DNA virus of the Asfarviridae family. • S2: ASF is not a threat to human health and cannot be transmitted from pigs to humans. • S3: As of now, there is no approved vaccine, which is also a reason why animals are culled to prevent the spread of infection.

Telegram: https://t.me/insightsIAStips 119 Youtube: https://www.youtube.com/channel/UCpoccbCX9GEIwaiIe4HLjwA

Revision Through MCQs (RTM) Compilation (June 2021)

Refer: https://www.insightsonindia.com/2021/06/28/african-swine-fever-2/

235. Consider the following statements about International Atomic Energy Agency: 1. It was set up as the world's "Atoms for Peace" organization in 1957 within the United Nations family. 2. It inhibit the use of nuclear energy for any military purpose, including nuclear weapons. 3. It works only with its Member States. Which of the given above statements is/are correct? (a) 1 and 2 only (b) 1 only (c) 2 only (d) 2 and 3 only Ans: (a) Explanation: • S2: The International Atomic Energy Agency is an international organization that seeks to promote the peaceful use of nuclear energy, and to inhibit its use for any military purpose, including nuclear weapons. • S1: The IAEA is the world's centre for cooperation in the nuclear field. It was set up as the world's "Atoms for Peace" organization in 1957 within the United Nations family. • S3: The Agency works with its Member States and multiple partners worldwide to promote the safe, secure and peaceful use of nuclear technologies.

Refer: https://www.insightsonindia.com/2021/06/28/nuclear-site-images-wont-be- shared-with-iaea-iran/

236. Recently, the Corporate Affairs Ministry has expanded the turnover and borrowing thresholds for Small and Medium sized Companies (SMC). In this context, consider the following statements: 1. The Corporate Affairs Ministry has increased the turnover threshold for SMCs to Rs 50 crore from Rs 10 crore. 2. SMCs are permitted to avail a number of exemptions under the Company (Accounting Standards) Rules 2021 to reduce the complexity of regulatory filings for smaller firms. Which of the given above statements is/are correct? (a) 1 only (b) 2 only (c) Both 1 and 2 (d) Neither 1 nor 2 Ans: (b) Explanation: • The Corporate Affairs Ministry has expanded the turnover and borrowing thresholds for Small and Medium sized Companies (SMC). • What is the change? o The Corporate Affairs Ministry has increased the turnover threshold for SMCs to Rs 250 crore from Rs 50 crore, and the borrowing threshold to Rs 50 crore from Rs 10 crore.

Telegram: https://t.me/insightsIAStips 120 Youtube: https://www.youtube.com/channel/UCpoccbCX9GEIwaiIe4HLjwA

Revision Through MCQs (RTM) Compilation (June 2021)

o SMCs are permitted to avail a number of exemptions under the Company (Accounting Standards) Rules 2021 to reduce the complexity of regulatory filings for smaller firms.

Refer: https://www.insightsonindia.com/2021/06/28/how-small-and-medium-companies- will-benefit-from-higher-thresholds/

237. Recently, the researcher’s from which of the following country have claimed to found an ancient human skull that could belong to an altogether new species of humans? (a) Israel (b) Germany (c) India (d) China Ans: (d) Explanation: Dragon Man: • Researchers from China have claimed to have found an ancient human skull that could belong to an altogether new species of humans. • The skull was found in north-east China’s Harbin city. • It has been dubbed the “Dragon Man” or Homo longi, a name that has been derived from the Long Jiang or Dragon river in the Heilongjiang province of China where the city of Harbin is located. • They could be over 146,000 years old.

Refer: facts for prelims: https://www.insightsonindia.com/2021/06/28/insights-daily- current-affairs-pib-summary-28-june-2021/

238. Consider the following statements about Bengal monitor or common Indian monitor: 1. It is carnivorous and poisonous. 2. It is protected under Schedule I of WPA. 3. It is listed under the ‘Least Concern’ category on the International Union for Conservation of Nature Red List. Which of the given above statements is/are correct? (a) 1 and 3 only (b) 2 only (c) 2 and 3 only (d) 1, 2 and 3 Ans: (c) Explanation: • The Bengal monitor or common Indian monitor (Varanus bengalensis) is a large lizard that is mainly terrestrial. • The monitors are carnivorous and non-poisonous. • The monitor lizard is protected under Schedule I of WPA but is regularly killed for its meat, blood and oil. • The species is listed under the ‘Least Concern’ category on the International Union for Conservation of Nature Red List.

Refer: facts for prelims: https://www.insightsonindia.com/2021/06/28/insights-daily- current-affairs-pib-summary-28-june-2021/

Telegram: https://t.me/insightsIAStips 121 Youtube: https://www.youtube.com/channel/UCpoccbCX9GEIwaiIe4HLjwA

Revision Through MCQs (RTM) Compilation (June 2021)

239. Consider the following statements about Bankim Chandra Chattopadhyay: 1. He is notable for his play Nil Darpan (1860). 2. He brought out a monthly magazine called Bangadarshan in 1872. 3. He wrote the national song Vande Mataram. Which of the given above statements is/are correct? (a) 1 and 2 only (b) 2 and 3 only (c) 1 and 3 only (d) 1, 2 and 3 Ans: (b) Explanation: Bankim Chandra Chattopadhyay: • He was a Bengali poet and writer. • He wrote the national song Vande Mataram. • His novel Anandamath — which was set in the background of the Sanyashi Bidroho (rebellion of monks in late 18th century) — is considered to be one of key works on Bengal’s nationalism. • His first Bengali fiction is called ‘Durgeshnondini’ published in 1865. • He also wrote other famous novels like Kapalkundala in 1866, Mrinalini in 1869, Vishbriksha in 1873, Chandrasekhar in 1877, Rajani in 1877, Rajsimha in 1881 and Devi Chaudhurani in 1884. • He brought out a monthly magazine called Bangadarshan in 1872. • His first fiction to appear in print was the English novel Rajmohan’s Wife. • Nil Darpan or The Indigo Planting Mirror was a Bengali play written by Dinabandhu Mitra in 1858-'59.

Refer: facts for prelims: https://www.insightsonindia.com/2021/06/28/insights-daily- current-affairs-pib-summary-28-june-2021/

240. Consider the following statements: 1. Vande Mataram is a poem written in Bengali by Bankim Chandra Chatterjee. 2. The poem was first sung by Rabindranath Tagore in the 1896 session of the Indian National Congress. Which of the given above statements is/are correct? (a) 1 only (b) 2 only (c) Both 1 and 2 (d) Neither 1 nor 2 Ans: (b) Explanation: • S1: Vande Mataram is a poem written in Sanskrit by Bankim Chandra Chatterjee. • The poem was first sung by Rabindranath Tagore in the 1896 session of the Indian National Congress.

Refer: facts for prelims: https://www.insightsonindia.com/2021/06/28/insights-daily- current-affairs-pib-summary-28-june-2021/

Telegram: https://t.me/insightsIAStips 122 Youtube: https://www.youtube.com/channel/UCpoccbCX9GEIwaiIe4HLjwA

Revision Through MCQs (RTM) Compilation (June 2021)

RTM- REVISION THROUGH MCQS – 29th -Jun-2021

241. Consider the following statements about Attorney General for India (AGI): 1. AGI is a part of the Union Executive. 2. AGI holds office during the pleasure of the President. 3. AGI has the right of audience in all Courts in India. Which of the given above statements is/are correct? (a) 1 and 2 only (b) 2 and 3 only (c) 1 and 3 only (d) 1, 2 and 3 Ans: (d) Explanation: • S1: The Attorney General for India is the central government’s chief legal advisor, and its primary lawyer in the Supreme Court of India. He is a part of the Union Executive. • S2: He is appointed by the President of India under Article 76(1) of the Constitution and holds office during the pleasure of the President. • S3: The Attorney General has the right of audience in all Courts in India as well as the right to participate in the proceedings of the Parliament, though not to vote.

Refer: https://www.insightsonindia.com/2021/06/29/attorney-general-3/

242. Consider the following statements about United Nations Office on Drugs and Crime: 1. It is a member of the United Nations Development Group. 2. It is mandated by its member States to assist in the implementation of both United Nations Convention against Corruption (UNCAC) and United Nations Convention against Transnational Organized Crime (UNTOC). Which of the given above statements is/are correct? (a) 1 only (b) 2 only (c) Both 1 and 2 (d) Neither 1 nor 2 Ans: (c) Explanation: • S1: It is a member of the United Nations Development Group. • S2: The United Nations Office on Drugs and Crime (UNODC) is mandated by its Member States to assist in the implementation of both United Nations Convention against Corruption (UNCAC) and United Nations Convention against Transnational Organized Crime (UNTOC), which along with the UN Drug Conventions of 1961, 1971 and 1988 underpin all the operational work of UNODC.

Refer: https://www.insightsonindia.com/2021/06/29/international-day-against-drug- abuse-and-illicit-trafficking-2021/

Telegram: https://t.me/insightsIAStips 123 Youtube: https://www.youtube.com/channel/UCpoccbCX9GEIwaiIe4HLjwA

Revision Through MCQs (RTM) Compilation (June 2021)

243. In the context of contribution of PC Mahalanonobis to Statistics, consider the following statements: 1. He is known as the father of Indian Statistics. 2. He established the Indian Statistical Institute (ISI) in 1931 in Bombay. 3. He helped in the establishment of Central Statistical Organisation (CSO) and the National Sample Survey (NSS). Which of the given above statements is/are correct? (a) 1 and 2 only (b) 2 and 3 only (c) 1 and 3 only (d) 1, 2 and 3 Ans: (c) Explanation: • S1: He is referred to as the chief architect of Indian statistical system as well as father of statistical science in India. • S2: He established the Indian Statistical Institute (ISI) in 1931 in Kolkata. The institute founded the journal Sankhya, along the lines of Karl Pearson’s Biometrika. In 1959 it was made an autonomous body of Ministry of Statistics & Programme Implementation. • S3: He also helped in the establishment of Central Statistical Organisation (CSO), the National Sample Survey (NSS) and the Annual Survey of Industries (ASI).

Refer: https://www.insightsonindia.com/2021/06/29/statistics-day-3/

244. The ‘Cyber Capabilities and National Power’ report is released by which of the following? (a) World Economic Forum (b) International Telecommunication Union (c) International Atomic Energy Agency (d) International Institute for Strategic Studies Ans: (d) Explanation: • The Cyber Capabilities and National Power Report was recently released by the International Institute for Strategic Studies (IISS). • The report analyses the cyber ecosystem of each state and how it intersects with international security, economic competition and military affairs.

Refer: https://www.insightsonindia.com/2021/06/29/cyber-capabilities-and-national- power-report-iiss/

245. Consider the following statements about World Intellectual Property Organization (WIPO): 1. It is one of the 15 specialized agencies of the United Nations (UN). 2. It is responsible for administering and monitoring the operation of the TRIPS (Trade- Related Aspects of Intellectual Property Rights) Agreement. Which of the given above statements is/are correct? (a) 1 only (b) 2 only (c) Both 1 and 2

Telegram: https://t.me/insightsIAStips 124 Youtube: https://www.youtube.com/channel/UCpoccbCX9GEIwaiIe4HLjwA

Revision Through MCQs (RTM) Compilation (June 2021)

(d) Neither 1 nor 2 Ans: (a) Explanation: • S1: The World Intellectual Property Organization (WIPO) is one of the 15 specialized agencies of the United Nations. • S2: The Agreement on Trade-Related Aspects of Intellectual Property Rights (TRIPS) is an international legal agreement between all the member nations of the World Trade Organization (WTO). o The TRIPS Council is responsible for administering and monitoring the operation of the TRIPS Agreement.

Refer: https://www.insightsonindia.com/2021/06/29/us-digital-millennium-copyright- act/

246. Consider the following statements: 1. The Baihetan Dam is the world’s second-biggest hydroelectric dam. 2. The dam is being constructed on Jinsha River, a tributary of the Mekong River. Which of the given above statements is/are correct? (a) 1 only (b) 2 only (c) Both 1 and 2 (d) Neither 1 nor 2 Ans: (a) Explanation: Baihetan Dam: • It is the world’s second-biggest hydroelectric dam under construction in southwestern China. • The dam is being constructed on Jinsha river, a tributary of the Yangtze. • The 289-m-tall Baihetan Dam will have 16 generating units with a capacity of 1 million kilowatts each.

Refer: facts for prelims: https://www.insightsonindia.com/2021/06/29/insights-daily- current-affairs-pib-summary-29-june-2021/

247. The term “2-deoxy-D-glucose” was in news recently, is related to which of the following? (a) anti-septic cream (b) anti-Covid drug (c) some cosmetic product (d) agriculture pesticide Ans: (b) Explanation: • Dr Reddy’s Laboratories has announced the commercial launch of drug 2-deoxy- D-glucose (2-DG). • It is an anti-Covid drug developed by the Defence Research and Development Organisation (DRDO). • The drug is approved for emergency use as an adjunct therapy to the standard of care in the treatment of coronavirus patients in hospital settings.

Refer: facts for prelims: https://www.insightsonindia.com/2021/06/29/insights-daily- current-affairs-pib-summary-29-june-2021/

Telegram: https://t.me/insightsIAStips 125 Youtube: https://www.youtube.com/channel/UCpoccbCX9GEIwaiIe4HLjwA

Revision Through MCQs (RTM) Compilation (June 2021)

248. With reference to ‘Electronic Vaccine Intelligence Network’ (eVIN), consider the following statements: 1. It is an indigenously developed technology. 2. It monitors the temperature of the cold chain through a smartphone application. 3. It supports the central government’s Universal Immunization Programme. Which of the given above statements is/are correct? (a) 1 and 2 only (b) 2 and 3 only (c) 1 and 3 only (d) 1, 2 and 3 Ans: (d) Explanation: • Electronic Vaccine Intelligence Network is an indigenously developed technology that digitises vaccine stocks and monitors the temperature of the cold chain through a smartphone application. The innovative eVIN was first launched across 12 states in 2015 to support better vaccine logistics management at cold chain points. eVIN supports the central government’s Universal Immunization Programme by providing real-time information on vaccine stocks and flows, and storage temperatures across all cold chain points across states and UTs. • CoWIN: o CoWIN is an extension of an electronic vaccine intelligence network, eVIN, which is used to collect real-time feedback of the vaccination programmes. o It is a cloud-based IT solution for planning, implementing, monitoring and evaluating COVID-19 vaccination in the country.

Refer: facts for prelims: https://www.insightsonindia.com/2021/06/29/insights-daily- current-affairs-pib-summary-29-june-2021/

249. Consider the following statements about Agni-P (Prime) missile: 1. It is a new-generation nuclear-capable ballistic missile. 2. It is a canisterised missile with range of more than 5000 km. 3. It has improved parameters including manoeuvring and accuracy compared to earlier Agni class of missiles. Which of the given above statements is/are correct? (a) 1 and 2 only (b) 2 and 3 only (c) 1 and 3 only (d) 1, 2 and 3 Ans: (c) Explanation: Agni-P (Prime) missile: • It is new-generation nuclear-capable ballistic missile. • It was successfully test-fired by the Defence Research and Development Organisation (DRDO). • Agni-P is a new-generation advanced variant of Agni class of missiles. It is a canisterised missile with range capability between 1,000 and 2,000 km. • Canisterisation of missiles reduces the time required to launch the missile while improving its storage and mobility.

Telegram: https://t.me/insightsIAStips 126 Youtube: https://www.youtube.com/channel/UCpoccbCX9GEIwaiIe4HLjwA

Revision Through MCQs (RTM) Compilation (June 2021)

• Agni-P has improved parameters including manoeuvring and accuracy compared to earlier Agni class of missiles • The longest of the Agni series, Agni-V, an Inter-Continental Ballistic Missile (ICBM) with a range of more than 5,000 km, has already been tested several times and validated for induction.

Refer: facts for prelims: https://www.insightsonindia.com/2021/06/29/insights-daily- current-affairs-pib-summary-29-june-2021/

250. Consider the following statements: 1. Fukuoka Prize is given annually to distinguished people to foster and increase awareness of Asian cultures. 2. Noted journalist P. Sainath has been selected as one of the three recipients of the Fukuoka Prize for 2021. Which of the given above statements is/are correct? (a) 1 only (b) 2 only (c) Both 1 and 2 (d) Neither 1 nor 2 Ans: (c) Explanation: • Noted journalist P. Sainath has been selected as one of the three recipients of the Fukuoka Prize for 2021. • The Fukuoka Prize is given annually to distinguished people to foster and increase awareness of Asian cultures, and to create a broad framework of exchange and mutual learning among the Asian people. • Eleven Indians have received the Fukuoka Prize so far. • The prize was established in 1990.

Refer: facts for prelims: https://www.insightsonindia.com/2021/06/29/insights-daily- current-affairs-pib-summary-29-june-2021/

RTM- REVISION THROUGH MCQS – 30th -Jun-2021

251. With reference to Parliamentary Privilege’s, consider the following statements: 1. According to the Constitution, the powers, privileges and immunities of Parliament and MP's are to be defined by Committee of Privileges. 2. The Speaker of Lok Sabha or Rajya Sabha (RS) Chairperson is the first level of scrutiny of a privilege motion. Which of the given above statements is/are correct? (a) 1 only (b) 2 only (c) Both 1 and 2 (d) Neither 1 nor 2 Ans: (b) Explanation: • S1: According to the Constitution, the powers, privileges and immunities of Parliament and MP's are to be defined by Parliament. No law has so far been

Telegram: https://t.me/insightsIAStips 127 Youtube: https://www.youtube.com/channel/UCpoccbCX9GEIwaiIe4HLjwA

Revision Through MCQs (RTM) Compilation (June 2021)

enacted in this respect. In the absence of any such law, it continues to be governed by British Parliamentary conventions. • S2: The Speaker/RS chairperson is the first level of scrutiny of a privilege motion. o The Speaker/Chair can decide on the privilege motion himself or herself or refer it to the privileges committee of Parliament. o If the Speaker/Chair gives consent under relevant rules, the member concerned is given an opportunity to make a short statement.

Refer: https://www.insightsonindia.com/2021/06/30/parliamentary-privileges-2/

252. Consider the following statements: 1. The ‘One Nation One Ration Card’ scheme is aimed at reducing the inter-state migration and regional disparities in India. 2. It is being implemented under the Integrated Management of PDS (IMPDS.) Which of the given above statements is/are correct? (a) 1 only (b) 2 only (c) Both 1 and 2 (d) Neither 1 nor 2 Ans: (b) Explanation: • S1: ONORC was also launched with the purpose that no poor person should be deprived of getting subsidised food grains under the food security scheme when they shift from one place to another. o The reform especially empowers the migratory population mostly labourers, daily wagers, urban poor like rag pickers, street dwellers, temporary workers in organised and unorganised sectors, domestic workers etc, who frequently change their place of dwelling to be self reliant in food security. • S2: It is being implemented under the Integrated Management of PDS (IMPDS).

Refer: https://www.insightsonindia.com/2021/06/30/one-nation-one-ration-card-onorc/

253. Consider the following statements about the Drugs Controller General of India (DCGI): 1. It controls the prices of pharmaceutical drugs in India. 2. It is responsible for approval of licenses of specified categories of drugs in India. Which of the given above statements is/are correct? (a) 1 only (b) 2 only (c) Both 1 and 2 (d) Neither 1 nor 2 Ans: (b) Explanation: • S1: The National Pharmaceutical Pricing Authority (NPPA) is a government regulatory agency that controls the prices of pharmaceutical drugs in India. • S2: Under the Drug and Cosmetics Act, the regulation of manufacture, sale and distribution of Drugs is primarily the concern of the State authorities while the Central Authorities are responsible for approval of New Drugs, Clinical Trials in the country, laying down the standards for Drugs, control over the quality of

Telegram: https://t.me/insightsIAStips 128 Youtube: https://www.youtube.com/channel/UCpoccbCX9GEIwaiIe4HLjwA

Revision Through MCQs (RTM) Compilation (June 2021)

imported Drugs, coordination of the activities of State Drug Control Organisations and providing expert advice with a view of bring about the uniformity in the enforcement of the Drugs and Cosmetics Act. • Drug Controller General of India is responsible for approval of licenses of specified categories of Drugs such as blood and blood products, I. V. Fluids, Vaccine and Sera.

Refer: https://www.insightsonindia.com/2021/06/30/govt-gives-nod-for-cipla-to-import- modernas-vaccine/

254. With reference to Organization of the Petroleum Exporting Countries (OPEC), consider the following statements: 1. It is an intergovernmental organization of more than 25 Member countries. 2. It is headquartered in Vienna, Austria. 3. According to its statutes, OPEC membership is open to any country that is a substantial exporter of oil and shares the ideals of the organization. Which of the given above statements is/are correct? (a) 1 and 2 only (b) 2 and 3 only (c) 1 and 3 only (d) 1, 2 and 3 Ans: (b) Explanation: • S1 and S2: OPEC, which describes itself as a permanent intergovernmental organization, was created in Baghdad in September 1960 by founding members Iran, Iraq, Kuwait, Saudi Arabia, and Venezuela. The headquarters of the organization are in Vienna, Austria, where the OPEC Secretariat, the executive organ, carries out OPEC’s day-to-day business. • S3: According to its statutes, OPEC membership is open to any country that is a substantial exporter of oil and shares the ideals of the organization.

Refer: https://www.insightsonindia.com/2021/06/30/opec-and-rising-oil-prices/

255. With reference to Currency Swap Agreement (CSA), consider the following statements: 1. It is an arrangement between two friendly countries to involve in trading in their own local currencies. 2. As per the arrangements, both countries pay for import and export trade at the predetermined rates of exchange. Which of the given above statements is/are correct? (a) 1 only (b) 2 only (c) Both 1 and 2 (d) Neither 1 nor 2 Ans: (c) Explanation: What is this Currency Swap Arrangement (CSA)? • It is an arrangement between two friendly countries to involve in trading in their own local currencies. • As per the arrangements, both countries pay for import and export trade at the predetermined rates of exchange, without bringing in third country currency like the US Dollar.

Telegram: https://t.me/insightsIAStips 129 Youtube: https://www.youtube.com/channel/UCpoccbCX9GEIwaiIe4HLjwA

Revision Through MCQs (RTM) Compilation (June 2021)

Refer: https://www.insightsonindia.com/2021/06/30/lanka-banking-on-1-bn-india-swap- deal/

256. Consider the following statements: 1. Recently, the National Ganga Council (NGC) has released an updated atlas of glacial lakes that are part of the Ganga river basin. 2. The Atlas is available on Bhuvan portal of National Remote Sensing Centre (NRSC), ISRO. Which of the given above statements is/are correct? (a) 1 only (b) 2 only (c) Both 1 and 2 (d) Neither 1 nor 2 Ans: (b) Explanation: • Ministry of Jal Shakti has released an updated atlas of glacial lakes that are part of the Ganga river basin. • About 4,707 glacial lakes have been mapped in the Ganga basin. • Glacial lakes with water spread area greater than 0.25 ha were mapped using Resourcesat-2 Linear Imaging Self Scanning Sensor-IV satellite data. • The Atlas is available on Bhuvan portal of National Remote Sensing Centre (NRSC), ISRO.

Refer: Facts for Prelims: https://www.insightsonindia.com/2021/06/30/insights-daily- current-affairs-pib-summary-30-june-2021/

257. Consider the following pairs: GI tags State 1. Kaji Nemu Odisha 2. Hmaram Mizoram 3. Khola Chilli Andhra Pradesh 4. Kadaknath Madhya Pradesh Which of the above pairs is/are correctly matched? (a) 2 and 4 only (b) 1, 2 and 3 only (c) 1 and 4 only (d) 2, 3 and 4 only Ans: (a) Explanation: • Kaji Nemu::Assam • Khola Chilli:: Goa • Kadaknath or Kali Masi is a breed of chicken originating from the Jhabua and Dhar districts of western Madhya Pradesh. o It received GI tag in 2017. o It is popular for its black meat which is famous for its meat quality, texture, taste, and excellent medicinal values. o It is also famous for its low cholesterol and high protein content.

Refer: Facts for Prelims: https://www.insightsonindia.com/2021/06/30/insights-daily- current-affairs-pib-summary-30-june-2021/

Telegram: https://t.me/insightsIAStips 130 Youtube: https://www.youtube.com/channel/UCpoccbCX9GEIwaiIe4HLjwA

Revision Through MCQs (RTM) Compilation (June 2021)

258. Consider the following pairs : Place of Pilgrimage: Location 1. Srisailam : Nallamala Hills 2. Omkareshwar : Satmala Hills 3. Pushkar : Mahadeo Hills Which of the above pairs is/are correctly matched? (a) 1 only (b) 2 and 3 only (c) 1 and 2 only (d) 1 and 3 only Ans: (a) Explanation: • The town (as well as the mandal) of Srisailam is reputed for the shrine of Lord Mallikarjuna on the flat top of Nallamala Hills. • Omkareshwar is situated on the Mandhata hills in Madhya Pradesh, known in ancient times as Shivpuri. The temple dates back to the Krita Yuga. River Narmada flows here around the Mandhata hill in the form of an Om (Omkaram). • The Mahadeo Hills are a range of hills in Madhya Pradesh state of central India. But, Pushkar lies in Ajmer, Rajasthan.

Refer: UPSC CSE 2015

259. With reference to ‘dugong’, a mammal found in India, which of the following statements is/are correct? 1. It is a herbivorous marine animal. 2. It is found along the entire coast of India. 3. It is given legal protection under Schedule 1 of the Wildlife (Protection) Act, 1972. Select the correct answer using the code given below. (a) 1 and 2 (b) 2 only (c) 1 and 3 (d) 3 only Ans: (c) Explanation: • The dugong is the only living representative of the once-diverse family Dugongidae; its closest modern relative, Steller’s sea cow (Hydrodamalis gigas), was hunted to extinction in the 18th century. The dugong is the only strictly marine herbivorous mammal, as all species of manatee use fresh water to some degree. • In India, it is found off the Gujarat Coast (Gulf of Kutch), Tamil Nadu coast (Gulf of Mannar), Palk Bay and Andaman and Nicobar Islands.

Refer: UPSC CSE 2015

260. In India, markets in agricultural products are regulated under the (a) Essential Commodities Act, 1955 (b) Agricultural Produce Market Committee Act enacted by States (c) Agricultural Produce (Grading and Marking) Act, 1937 (d) Food Products Order, 1956 and Meat and Food Products Order, 1973 Ans: (b)

Telegram: https://t.me/insightsIAStips 131 Youtube: https://www.youtube.com/channel/UCpoccbCX9GEIwaiIe4HLjwA

Revision Through MCQs (RTM) Compilation (June 2021)

Explanation: • Agricultural Markets in most parts of the Country are established and regulated under the State APMC Acts. The whole geographical area in the State is divided and declared as a market area wherein the markets are managed by the Market Committees constituted by the State Governments. Once a particular area is declared a market area and falls under the jurisdiction of a Market Committee, no person or agency is allowed freely to carry on wholesale marketing activities.

Refer: UPSC CSE 2015

Telegram: https://t.me/insightsIAStips 132 Youtube: https://www.youtube.com/channel/UCpoccbCX9GEIwaiIe4HLjwA